bstrandable NCLEX Respiratory System

Réussis tes devoirs et examens dès maintenant avec Quizwiz!

The nurse is caring for a patient who has peripheral arterial disease. Which of the following statements by the patient indicates understanding of how to manage the pain of peripheral arterial disease

" I will sit with my legs down"

Which is the best explanation by a nurse for why a physician did not prescribe antibiotics for influenza

" Influenza is caused by viruses"

A nurse provides instructions to a client at risk for thrombophelbitis regarding measures to prevent its occurrence. Which statement by the client indicates an understanding of the measures to prevent thrombophlebitis?

"I should avoid sitting or standing in one position for long periods of time."

The physician has prescribed NPH insulin for a client with diabetes mellitus. Which statement indicates that the client knows when the peak action of the insulin occurs?

"I will eat a snack around three o'clock each afternoon."

The physician has prescribed Novalog insulin for a client with diabetes mellitus. Which statement indicates that the client knows when the peak action of the insulin occurs?

"I will make sure I eat breakfast within 10 minutes of taking my insulin."

While a nurse is reinforcing preoperative teaching for a client scheduled for a right pneumonectomy, the client tells the nurse, "I cough all the time, and I'm really afraid it will hurt when I cough after the surgery." Which of the following is an appropriate nursing response? "After the surgeon removes the lung, you will not need to cough." "I'll make sure you get a cough suppressant to keep you from straining the incision when you cough." "Intravenous pain medication will keep you free of pain when you cough after the surgery." "I will show you how to splint your incision while coughing."

"I will show you how to splint your incision while coughing."

The nurse in charge is teaching a client with emphysema how to perform pursed-lip breathing. The client asks the nurse to explain the purpose of this breathing technique. Which explanation should the nurse provide?

"It helps prevent early airway collapse."

Which statement by the nurse when explaining the purpose of positive end-expiratory pressure (PEEP) to the family members of a patient with ARDS is correct?

"PEEP prevents the lung air sacs from collapsing during exhalation." rationale: By preventing alveolar collapse during expiration, PEEP improves gas exchange and oxygenation. PEEP will not prevent the fibrotic changes that occur with ARDS, push more air into the lungs, or change the fraction of inspired oxygen (FIO2) delivered to the patient.

The nurse is evaluating patient teaching for Mitral Valve prolapse. The patient shows understanding of the prognosis of MVP by stating

"There are often no symptoms"

Which of the following are causes for concern following a thoracentesis? (Select all that apply.)

*Dyspnea *Fever *Hypotension **Dyspnea can indicate pneumothorax or reaccumulation of fluid. Fever can indicate infection. Hypotension can indicate intrathoracic bleeding. Localized bloody drainage contained on a dressing and soreness around puncture site, are expected findings. An SaO2 of 95% indicated good oxygenation.

Which of the following can cause a low pulse oximetry reading? (Select all that apply.)

*Nail polish *Inadequate peripheral circulation *Edema **Nail polish, inadequate peripheral circulation, and edema can all generate a low reading. Hypothermia, rather than hyperthermia, and a decreased Hgb level rather, than an increased Hgb level, can result in a low reading.

A nurse is caring for a client who is scheduled for a thoracentesis at the bedside. Which of the following items should the nurse ensure is in client's room? (Select all that apply.)

*Oxygen equipment *Pulse oximeter *Thoracentesis tray **Equipment to supply oxygen, a pulse oximeter, and a thoracentesis tray are all necessary for a thoracentesis. An incentive spirometer and suture removal kit are not indicated for this type of procedure.

Signs and Symptoms of TB

- early stages pt is asymptomatic - Active disease may initally manifest with fatigue, malaise, anorexia, unexplained weightloss, low-grade fevers and night sweats. - cough develops with mucoid or mucopurulent sputum. -Sometimes more acute onset of symptoms: high fever, chills generalized flu like symptoms, pleuric pain and a productive cough.

Identify initial assessment findings for a patient with EARLY STAGE LEFT sided heart failure

- fatigue - breathlessness - dizziness - confusion as a result of tissue hypoxia from the diminished CO

Latent TB infection (LTBI)

- pts not sick because the organisms are not active. - they are asymptomatic and can NOT spread the disease -drug therapy help to prevent an infection from developing into the active disease. -Drug therapy for LTBI is only one drug instead of a cocktail of drugs (INH)

Causes of Pulmonary embolism

- usually caused by DVT that has been freed and moved to the lungs. Can be air or tumor tissue. - heart failure, over hydration with IV fluids, hypoalbuminemia, altered capillary permeability of lungs, malignancies of lymph system, resp distress syndrome.

Types of Pneumonia

-Community- Acquired Pneumonia- lower resp tract infectio with onset in the community or during the first 2 days of hospitalization. -Hospital- Acquired, Ventilator-Associated andealth Care- Associated Pneumonia- occurs 48 hrs or longer after hospital admission -Aspiration Pneumonia- pts that are at risk for aspiration -Opportunistic Pneumonia- immunocomprised pts

S/E of Anticholinergics

-Red as a Beet= vasodilation -Mad as a hatter= nervous, disorientation -Hot as a Hare= tachycardia -Blind as a Bat = enlarged pupils

Risk factors for Lung Cancer

-SMOKING!!!!!!! -inhaled carcinogens- asbestos, radon, nickel, iron and iron oxides, uranium, arsenic and air pollution. -Genetics -Hormones

Risk factors for pneumonia

-age, immobility, malnutrition, smoking, URTI, chronic diseases, immunosuppressed

TB- Pathophysiology

-gram positive, actid fast bacillus -spread by respiratory droplets. ( not easily spread) Need to be within 6 inches for 20 hrs a week. -Replicates slowly and spread via the lymphatic system. - find favorable environments for growth in upper lobes of the lungs, kidneys, epiphyses of the bone, cerebral cortex, and adrenal glands.

3 criteria for removal of chest tube

-less than 75 cc/day -reinflation of the lung -decreased symptoms

Drugs that increase effects of Methylzanthines

-steroids -erythromycin(antibiotic) -betablocker

S/E of albuterol

-tremors -hypokalemia -anxious -headache -increased HR -increased BP -dizzy **asthma pt should never be on Beta blocker

Emphysema path

...

A nurse performs an ABG sampling at 0930 on a client who has a heparin drip infusing. At which of the following times will it be appropriate for the nurse to discontinue holding pressure on the puncture site?

0950

A nurse perfoms an ABG sampling at 0930 on a client who has a heparin drip infusing. At which of the following times will it be appropriate for the nurse to discontinue holding pressure on the puncture site?

0950 *Immediately after arterial puncture, hold direct pressure over the site for at least 5 min. The pressure must be maintained for at least 20 min if the client is receiving anticoagulant therapy. Ensure that bleeding has stopped prior to removing direct pressure.

The amount of fluid remove during a thoracentesis is limited to ____ at a time to prevent cardiovascular collapse.

1 Liter

A client uses a metered-dose inhaler (MDI) to aid in management of his asthma. Which action by the client indicates to the nurse that he needs further instruction regarding its use? Select all that apply. 1. Activation of the MDI is not coordinated with inspiration. 2. The client inspires rapidly when using the MDI. 3. The client holds his breath for 3 seconds after inhaling with the MDI. 4. The client shakes the MDI after use. 5. The client performs puffs in rapid succession.

1, 2, 3, 4, 5. Utilization of an MDI requires coordination between activation and inspiration; deep breaths to ensure that medication is distributed into the lungs, holding the breath for 10 seconds or as long as possible to disperse the medication into the lungs, shaking up the medication in the MDI before use, and a sufficient amount of time between puffs to provide an adequate amount of inhalation medication.

The nurse is teaching the client how to use a metered-dose inhaler (MDI) to administer a corticosteroid. Which of the following client actions indicates that he is using the MDI correctly? Select all that apply. 1. The inhaler is held upright. 2. The head is tilted down while inhaling the medicine. 3. The client waits 5 minutes between puffs. 4. The mouth is rinsed with water following administration. 5. The client lies supine for 15 minutes following administration.

1, 4. The client should shake the inhaler and hold it upright when administering the drug. The head should be tilted back slightly. The client should wait about 1 to 2 minutes between puffs. The mouth should be rinsed following the use of a corticosteroid MDI to decrease the likelihood of developing an oral infection. The client does not need to lie supine; instead, the client will likely to be able to breathe more freely if sitting upright.

A client experiencing a severe asthma attack has the following arterial blood gas: pH 7.33; Pco2 48; Po2 58; HCO3 26. Which of the following orders should the nurse perform first? 1. Albuterol (Proventil) nebulizer. 2. Chest x-ray. 3. Ipratropium (Atrovent) inhaler. 4. Sputum culture.

1. The arterial blood gas reveals a respiratory acidosis with hypoxia. A quick-acting bronchodilator, albuterol, should be administered via nebulizer to improve gas exchange. Ipratropium is a maintenance treatment for bronchospasm that can be used with albuterol. A chest x-ray and sputum sample can be obtained once the client is stable.

Asthma signs/symptoms and diagnostics:

1. Altered LOC/confused = early sign of respiratory failure. 2. Wheezing 3. Cough 4. SOB - 5. Tachypnea and tachycardia 6. chest tightness 7. increased mucous production

What are the three methods that pneumonia causing organisms can reach the lungs?

1. Aspiration 2. Inhalation 3. Hematogenous spread

What are the 4 classifications of pneumonia by causative agent?

1. Bacterial 2. Viral 3. Fungal 4. Chemical

What are the 2 classifications for acquiring pneumonia?

1. Community 2. Hospital

What are some nursing interventions to help someone with pneumonia?

1. Coughing and deep breathing 2. IS 3. Humidification of airway 4. Provide fluids 5. Suction airway if necessary 6. Assess breath sounds before and after coughing

Key asthma factors:

1. Genetics (a lot of them are allergy based). 2. Triggers are usually different for everyone a. Non allergy based triggers: Irritants: cold dry air, or fine airborne particles, b.emotions, even stress, c.medications: NSAIDS, nonselective beta blockers can 3. Airway hyperresponsiveness: exercise or URI 4. Allergic - allergens: inflammatory/histamine response - IgE mediated response involving antigens and antibodies.

What are the s/s of respiratory acidosis? (4)

1. HA 2. Confusion 3. sleepy (difficult to arouse) 4. hypoxia

What are the 5 defining characteristics of ARDS?

1. Hypoxemia that persists even with 100% O2 2. Decreased Lung compliance 3. Dyspnea 4. Non-cardiac associated bilateral pulmonary edema 5. dense pulmonary infiltrates on radiography.

Place the following steps for obtaining a client's ABGs in the correct order.

1. Obtain a heparinized syringe for the sample collection. 2. Explain and reinforce the procedure with the client. 3. Perform an Allen's test prior to arterial puncture to verify patent radial and ulnar circulation. 4. Perfom an arterial puncture using a surgical aseptic technique, and collect a specimen into a heparinized syringe. 5. Place the collected and capped specimen into a basin of ice and water to preserve pH levels and oxygen pressure. Ensure immediate transport to the laboratory. 6. Immediately after an arterial puncture, hold direct pressure over the site for at least 5 min. Pressure must be maintained for at least 20 min if the client is receiving anticoagulant therapy. Ensure that the bleeding has stopped prior to removing direct pressure. 7. Monitor the ABG sampling site for bleeding, loss of pulse, swelling, and changes in temperature and color. 8. Report results to the primary care provider as soon as they are available.

3 stages of inflammation:

1. Stage 1: Venoconstriction, but locally at the site you get arteriole dilation and increased cell permeability, which causes fluid leakage and swelling. Redness, warmth and edema from that dilation.

Diagnostics:

1. monitor for accessory muscle use. 2. Check abgs, pao2 and pulse ox 3. Check eosinophil levels 4. Get PFTs before and after. If doing a baseline, doing a PFT during an exacerbation gives an objective indiction of how much trouble a patient is in and what type of med to use and assess how effectively a medication is working. 5. Get a chest xray 6. Do NIH asthma staging.

A nurse is suctioning fluids from a male client via a tracheostomy tube. When suctioning, the nurse must limit the suctioning time to a maximum of: ______ seconds

10

A therapeutic theophylline level is _____ mcg/ml.

10 - 20

When suctioning secretions that are collecting in an endotracheal tube, the nurse does not apply suction for longer than:

15 seconds Applying suction for too long can result in complications such as hypoxemia and cardiac dysrhythmias. Thus the nurse is always aware of the length of time that suctioning is applied to an airway. If the suctioning time is too short, the suction catheter may not remove the secretions. If the suctioning time is too long, hypoxemia and/or cardiac dysrhythmias could result.

With relation to orthostatic hypertension, in response to the drop in BP the pulse increase _______ bpm to maintain cardiac output.

15-20

Normal O2 content

15-20 vol%

Orthostatic hypotensiohn is a drop in systolic BP greater than ___mmhg; and an increase in heart rate greater than _____ bpm in response to the drop in BP.

15; 20

A client with known chronic obstructive pulmonary disease (COPD) is admitted to the emergency department with multiple minor injuries following an automobile accident. To ensure adequate ventilation the nurse applies a nasal cannula providing oxygen at what rate and for what reason?

2 L/min to prevent elevating the arterial oxygen tension (PaO2), which would suppress the hypoxic drive

The nurse is instructing a client with COPD how to do pursed-lip breathing. In which order should the nurse explain the steps to the client? 1. "Breathe in normally through your nose for 2 counts (while counting to yourself, one, two)." 2. "Relax your neck and shoulder muscles." 3. "Pucker your lips as if you were going to whistle." 4. "Breathe out slowly through pursed lips for 4 counts (while counting to yourself, one, two, three, four)."

2, 1, 3, 4. The nurse should instruct the client to first relax the neck and shoulders and then take several normal breaths. After taking a breath in, the client should pucker the lips, and finally breathe out through pursed lips.

Which of the following is an expected outcome for an elderly client following treatment for bacterial pneumonia? 1. A respiratory rate of 25 to 30 breaths/ minute. 2. The ability to perform activities of daily living without dyspnea. 3. A maximum loss of 5 to 10 lb of body weight. 4. Chest pain that is minimized by splinting the rib cage.

2. An expected outcome for a client recovering from pneumonia would be the ability to perform activities of daily living without experiencing dyspnea. A respiratory rate of 25 to 30 breaths/ minute indicates the client is experiencing tachypnea, which would not be expected on recovery. A weight loss of 5 to 10 lb is undesirable; the expected outcome would be to maintain normal weight. A client who is recovering from pneumonia should experience decreased or no chest pain.

A client with bacterial pneumonia is to be started on I.V. antibiotics. Which of the following diagnostic tests must be completed before antibiotic therapy begins? 1. Urinalysis. 2. Sputum culture. 3. Chest radiograph. 4. Red blood cell count.

2. A sputum specimen is obtained for culture to determine the causative organism. After the organism is identified, an appropriate antibiotic can be prescribed. Beginning antibiotic therapy before obtaining the sputum specimen may alter the results of the test. Neither a urinalysis, a chest radiograph, nor a red blood cell count needs to be obtained before initiation of antibiotic therapy for pneumonia.

A client's arterial blood gas values are as follows: pH, 7.31; PaO2, 80 mm Hg; PaCO2, 65 mm Hg; HCO3 −, 36 mEq/ L. The nurse should assess the client for? 1. Cyanosis. 2. Flushed skin. 3. Irritability. 4. Anxiety.

2. The high PaCO2 level causes flushing due to vasodilation. The client also becomes drowsy and lethargic because carbon dioxide has a depressant effect on the central nervous system. Cyanosis is a sign of hypoxia. Irritability and anxiety are not common with a PaCO2 level of 65 mm Hg but are associated with hypoxia.

3 stages of inflammation:

2. Stage 2: Neutrophils - attack, destroy and phagocytized, exudate formation occurs - pus

Normal HCO3 range

21-28 according to the HESI book

HCO3- normal range

22 to 26 mEq/L

Normal HCO3 range

22-26 mEq/L

Percentage of oxygen a nasal cannula delivers?

28-44% (2-6L)

what fluid order should the emphysema client have?

3 liters of fluid per day

Which of the following mental status changes may occur when a client with pneumonia is first experiencing hypoxia? 1. Coma. 2. Apathy. 3. Irritability. 4. Depression.

3. Clients who are experiencing hypoxia characteristically exhibit irritability, restlessness, or anxiety as initial mental status changes. As the hypoxia becomes more pronounced, the client may become confused and combative. Coma is a late clinical manifestation of hypoxia. Apathy and depression are not symptoms of hypoxia.

3 stages of inflammation:

3. Stage 3: Cellular repair and replacement - it doesn't repair 100% all the time, which leads to scar tissue formation. This scar tissue doesn't perform as well as normal tissue does, it depends on how much inflammation occurred and for how long that inflammatory response was.

PaCO2 normal range

35 to 45 mm Hg

Normal PCO2 range

35-45

Normal PaCO2 range

35-45 mm Hg

Erb's point

3rd intercostal space, left of sternum aortic and pulmonic murmurs

Which of the following is an expected outcome of pursed-lip breathing for clients with emphysema? 1. To promote oxygen intake. 2. To strengthen the diaphragm. 3. To strengthen the intercostal muscles. 4. To promote carbon dioxide elimination.

4. Pursed-lip breathing prolongs exhalation and prevents air trapping in the alveoli, thereby promoting carbon dioxide elimination. By prolonging exhalation and helping the client relax, pursed-lip breathing helps the client learn to control the rate and depth of respiration. Pursed-lip breathing does not promote the intake of oxygen, strengthen the diaphragm, or strengthen intercostal muscles.

Which of the following is an appropriate expected outcome for an adult client with well-controlled asthma? 1. Chest X-ray demonstrates minimal hyperinflation. 2. Temperature remains lower than 100 ° F (37. 8 ° C). 3. Arterial blood gas analysis demonstrates a decrease in PaO2. 4. Breath sounds are clear.

4. Between attacks, breath sounds should be clear on auscultation with good air flow present throughout lung fields. Chest X-rays should be normal. The client should remain afebrile. Arterial blood gases should be normal.

When instructing clients on how to decrease the risk of chronic obstructive pulmonary disease (COPD), the nurse should emphasize which of the following? 1. Participate regularly in aerobic exercises. 2. Maintain a high-protein diet. 3. Avoid exposure to people with known respiratory infections. 4. Abstain from cigarette smoking.

4. Cigarette smoking is the primary cause of COPD. Other risk factors include exposure to environmental pollutants and chronic asthma. Participating in an aerobic exercise program, although beneficial, will not decrease the risk of COPD. Insufficient protein intake and exposure to people with respiratory infections do not increase the risk of COPD.

Percentage of oxygen a simple face mask delivers?

40-60%

Methylzanthines

4th line drug in COPD and asthma ex) amanophylline and theophylline (400 mg/day or LESS) narrow theraputic range (10-20) Amanophylline needs own IV line and can not mix with anything (no piggy backs)

What is an average range for PEEP settings?

5-10

What is the age range where cancer of the larynx usually occurs?

55-70

Percentage of oxygen a partial rebreather mask delivers?

60-75%

pH normal range

7.35 to 7.45

Normal pH range

7.35- 7.45

Normal pH range

7.35-7.45

While reviewing a clients lab results, the nurse recognizes that a fasting glucose value of ________________ is within normal limits.

80

PaO2 normal range

80 to 100 mm Hg

SaO2 below ____% is life-threatning.

80%; the lower the SaO2 level, the less acurate the value

Normal PO2 range

80-100

Normal PaO2 range

80-100 mm Hg

SaO2 below ____% requires emergency

86%

How long does TB therapy typically last?

9-12 months

SaO2

95 to 100%

Accepted reference range for SaO2

95% to 100%, acceptable levels may range from 91% to 100%. Some illness state may even allow for an SaO2 of 85% to 89%; may be slightly lower in older adult & those w/dark skin

Expected O2 range

95%-100%; may range from 91%-100%; some illnesses allow 85%-89%

Normal SaO2 range

95-100 %

Normal O2 saturation

95-100%

At what SaO2 level is considered "life-threatening"?

<80%

At what SaO2 level is considered an emergency?

<86%

What is considered excess drainage (for a chest tube)?

>70mL/hr Drainage will often increase with position changes or coughing.

8. To evaluate both oxygenation and ventilation in a patient with acute respiratory failure, the nurse uses the findings revealed with a. arterial blood gas (ABG) analysis. b. hemodynamic monitoring. c. chest x-rays. d. pulse oximetry.

A Rationale: ABG analysis is useful because it provides information about both oxygenation and ventilation and assists with determining possible etiologies and appropriate treatment. The other tests may also provide useful information about patient status but will not indicate whether the patient has hypoxemia, hypercapnia, or both. Cognitive Level: Comprehension Text Reference: p. 1805 Nursing Process: Assessment NCLEX: Physiological Integrity

19. Which information obtained by the nurse when assessing a patient with acute respiratory distress syndrome (ARDS) who is being treated with mechanical ventilation and high levels of positive end-expiratory pressure (PEEP) indicates a complication of ventilator therapy is occurring? a. The patient has subcutaneous emphysema. b. The patient has a sinus bradycardia, rate 52. c. The patient's PaO2 is 50 mm Hg and the SaO2 is 88%. d. The patient has bronchial breath sounds in both the lung fields.

A Rationale: Complications of positive-pressure ventilation (PPV) and PEEP include subcutaneous emphysema. Bradycardia, hypoxemia, and bronchial breath sounds are all concerns, but they are not caused by PPV and PEEP. Cognitive Level: Application Text Reference: p. 1816 Nursing Process: Assessment NCLEX: Physiological Integrity

2. The nurse will monitor for clinical manifestations of hypercapnia when a patient in the emergency department has a. chest trauma and multiple rib fractures. b. carbon monoxide poisoning after a house fire. c. left-sided ventricular failure and acute pulmonary edema. d. tachypnea and acute respiratory distress syndrome (ARDS).

A Rationale: Hypercapnia is caused by poor ventilatory effort, which occurs in chest trauma when rib fractures (or flail chest) decrease lung ventilation. Carbon monoxide poisoning, acute pulmonary edema, and ARDS are more commonly associated with hypoxemia. Cognitive Level: Application Text Reference: p. 1800 Nursing Process: Assessment NCLEX: Physiological Integrity

14. When admitting a patient in possible respiratory failure with a high PaCO2, which assessment information will be of most concern to the nurse? a. The patient is somnolent. b. The patient's SpO2 is 90%. c. The patient complains of weakness. d. The patient's blood pressure is 162/94.

A Rationale: Increasing somnolence will decrease the patient's respiratory rate and further increase the PaCO2 and respiratory failure. Rapid action is needed to prevent respiratory arrest. An SpO2 of 90%, weakness, and elevated blood pressure all require ongoing monitoring but are not indicators of possible impending respiratory arrest. Cognitive Level: Application Text Reference: p. 1804 Nursing Process: Assessment NCLEX: Physiological Integrity

22. A patient with acute respiratory distress syndrome (ARDS) has progressed to the fibrotic phase. The patient's family members are anxious about the patient's condition and are continuously present at the hospital. In addressing the family's concerns, it is important for the nurse to a. support the family and help them understand the realistic expectation that the patient's chance for survival is poor. b. inform the family that home health nurses will be able to help them maintain the mechanical ventilation at home after patient discharge. c. refer the family to social support services and case management to plan for transfer of the patient to a long-term care facility. d. provide hope and encouragement to the family because the patient's disease process has started to resolve.

A Rationale: The chance for survival is poor when the patient progresses to the fibrotic stage because permanent damage to the alveoli has occurred. Because of continued severe hypoxemia, the patient is not a candidate for home health or long-term care. The fibrotic stage indicates a poor patient prognosis, not the resolution of the ARDS process. Cognitive Level: Application Text Reference: p. 1814 Nursing Process: Implementation NCLEX: Psychosocial Integrity

5. A patient is brought to the emergency department unconscious following a barbiturate overdose. Which potential complication will the nurse include when developing the plan of care? a. Hypercapnic respiratory failure related to decreased ventilatory effort b. Hypoxemic respiratory failure related to diffusion limitations c. Hypoxemic respiratory failure related to shunting of blood d. Hypercapnic respiratory failure related to increased airway resistance

A Rationale: The patient with an opioid overdose develops hypercapnic respiratory failure as a result of the decrease in respiratory rate and depth. Diffusion limitations, blood shunting, and increased airway resistance are not the primary pathophysiology causing the respiratory failure. Cognitive Level: Application Text Reference: p. 1800 Nursing Process: Diagnosis NCLEX: Physiological Integrity

Chronic airflow limitation, Chronic obstructive lung disease, Chronic obstructive pulmonary disease

A disease state characterized by pulmonary airflow obstruction that is usually progressive, not fully reversible, and sometimes accompanied by airway hyperreactivity; airflow obstruction may be caused by chronic bronchitis and /or emphysema; in chronic hypercapnia the stimulus to breathe is a low PaO2 instead of an increased PaCO2

What are some signs of lung cancer?

A dry hacking cough that progresses to productive as disease progresses, hoarseness, hemoptysis and rust colored sputum

Chronic Airflow Limitation

A group of chronic lung diseases that include: 1. Asthma: intermittent and reversible 2. Chronic bronchitis 3. Emphysema

The physician orders 2 to 3 L/min of oxygen to be delivered to the client with COPD because:

A higher flow rate may suppress the client's drive to breathe; due to the chronic state of hypoxemia, adding oxygen now would suppress their drive to breathe.

Cyanosis, the blue discoloration of the skin and mucous membranes caused by the presence of desaturated hemoglobin in capillaries, is:

A late sign of hypoxia Cyanosis, blue discoloration of the skin and mucous membranes caused by the presence of desaturated hemoglobin in capillaries, is a late sign of hypoxia. The presence or absence of cyanosis is not a reliable measure of oxygen status.

A client arrives in the emergency department complaining of chest pain began 4 hours ago. A troponin T blood specimen is obtained, and the results indicate a level of 0.69ng/mL. The nurse interprets that this result indicates

A level that indicates angina pectoris

A nurse is caring for a client with respiratory insufficiency. The ABG results indicate a pH of 7.50 and a PC02 of mm Hg, and the nurse is told that the client is experiencing respiratory alkalosis. Which of the following additional lab values would the nurse expect to note?

A potassium level of 3.2 mEq/L

What is PEEP?

A small amount of pressure used to keep the airway open on expiration

The client has recently returned from having a thyroidectomy. The nurse should keep which of the following at the bedside?

A tracheotomy set

_____ released from the posterior pituitary, directly increases water reabosorption by the kidneys, thus increasing blood volume and blood pressure

ADH antidiureitc hormone

What increases the amount of water reabsorbed by the kidney tubules, which decreases urine output?

ADH-antidiuretic hormone

______ increases the rexcretion fo sodium by the kidneys

ANP arterial natriuretic peptide

What does the pulse ox monitor?

ARTERIAL blood (infrared wave measures light absorption)

following a motor-vehicle crash a client is admitted with multiple trauma, including significant bruising of the left chest from striking the steering wheel. the client is alert and reports severe left chest pain on inspiration. The nurse should assess the client for manifestations of pneumothorax, including which of the following? Absence of breath sounds Expiratory wheezing Inspiratory stridor Rhonchi

Absence of breath sounds

ABG is a measurement of what?

Acid-base balance of the blood pH (amt of hydrogen ions in the arterial blood), PaO2 (partial pressure of oxygen), PaCO2 (partial pressure of CO2), HCO3 (concentration of bicarb in blood), SaO2 (% of oxygen bound to Hgb compared the the amt that can be carried).

Blood pH below 7.35 reflect ______.

Acidosis

Drug therapy for TB

Active disease is treated with combo of 4 drugs: INH, rifampin and 2 others. Measure treatment by AFB. if negative continue treatment for 3 months. (acid fast bacillus sputum test).

highest priority nursing intervention for patient just diagnosed with pulmonary embolism?

Administer IV heparin at 1,300 units per hour

Define an air embolism

Air enters the arterial system (can happen with catheter insertion)

What kind of precautions are TB?

Airborne

After a laryngectomy, which of the following assessments takes priority

Airway patency

At 11 p.m., a male client is admitted to the emergency department. He has a respiratory rate of 44 breaths/minute. He's anxious, and wheezes are audible. The client is immediately given oxygen by face mask and methylprednisolone (Depo-medrol) I.V. At 11:30 p.m., the client's arterial blood oxygen saturation is 86% and he's still wheezing. The nurse should plan to administer:

Albuterol (Proventil).

what test do you have to perform before taking ABGs from radial artery?

Allen's

A client is scheduled for cardiac catheterization using radiopaque dye. The nurse checks which most critical item before the procedure?

Allergies to iodine or shellfish

What is a bronchoscopy?

Allows visualization of the larynx, trachea, & bronchi through flexible or rigid scope.

Tidal Volume (Vt)

Amount of air inhale and exhaled normally with each breath. Norm: 0.5 L

Residual Volume (RV)

Amount of air remaining in lungs after forced expiration. Allows for gas exchange to continue and prevents lung collapse Norm: 1.5 L

Expiratory Reserve Volume (ERV)

Amount of air that can be forcefully exhaled after normal exhalation Norm: 1.0 L

When collecting data from a client with reactive airway disease who is experiencing an acute asthma attack, the nurse should expect to auscultate for which of the following? An expiratory wheeze. A pleural friction rub. Inspiratory stridor. Subcutaneous emphysema.

An expiratory wheeze.

What denotes a positive TB test?

An induration of 10mm in 48 hours

Which of the following symptoms would the nurse identify as a priority to report for a patient with aortic stenosis

Angina

In which area should the nurse assess a patient who is on bed rest for the presence of edema?

Ankles

2. Nurse Kim is caring for a client with a pneumothorax and who has had a chest tube inserted notes continuous gentle bubbling in the suction control chamber. What action is appropriate? a. Do nothing, because this is an expected finding. b. Immediately clamp the chest tube and notify the physician. c. Check for an air leak because the bubbling should be intermittent. d. Increase the suction pressure so that bubbling becomes vigorous.

Answer A. Continuous gentle bubbling should be noted in the suction control chamber. Option B is incorrect. Chest tubes should only be clamped to check for an air leak or when changing drainage devices (according to agency policy). Option C is incorrect. Bubbling should be continuous and not intermittent. Option D is incorrect because bubbling should be gentle. Increasing the suction pressure only increases the rate of evaporation of water in the drainage system.

A nurse teaches a male client about the use of a respiratory inhaler. Which action by the client indicates a need for further teaching? a. Inhales the mist and quickly exhales b. Removes the cap and shakes the inhaler well before use c. Presses the canister down with the finger as he breathes in d. Waits 1 to 2 minutes between puffs if more than one puff has been prescribed

Answer A. The client should be instructed to hold his or her breath for at least 10 to 15 seconds before exhaling the mist. Options B, C, and D are accurate instructions regarding the use of the inhaler.

A nurse is caring for a female client diagnosed with tuberculosis. Which assessment, if made by the nurse, is inconsistent with the usual clinical presentation of tuberculosis and may indicate the development of a concurrent problem? a. Cough b. High-grade fever c. Chills and night sweats d. Anorexia and weight loss

Answer B. The client with tuberculosis USUALLY experiences cough (productive or nonproductive), fatigue, anorexia, weight loss, dyspnea, hemoptysis, chest discomfort or pain, chills and sweats (which may occur at night), and a low-grade fever

ACTH(adrenocorticotropic hormone) and Prolactin are secreted from which gland?

Anterior pituitary gland

FSH(follicle-stimulating hormone) and LH(luteinizing hormone) are secreted from what gland?

Anterior pituitary gland

Growth hormone and TSH(thyroid stimulating hormone) are secreted from what gland?

Anterior pituitary gland

Location of LLL

Anterior= ribs 4-8 posterior= T4-T10 on exspiration T4-T12 on inspiration

Levaquin (Levofloxin)

Antibiotic Fights bacteria in the body

Streptomycin

Antibiotic Works by killing sensitive bacteria by stopping the production of essential proteins needed to survive. Monitor kidney ftn and cbc.

Spiriva (Tiotropium)

Anticholinergic Enlarges airways

Atrovent (Ipratropium)

Anticholinergic Relaxes muscles in the airways and increase airflow to the lungs.

Benedryl (Diphenhydramine)

Antihistamine Reduces the natural chemical histamine in the body. Used to treat sneezing, runny nose, itchin, watery eyes, hives, rashes, itching and other s/s of allergies and the common cold. Also used to suppress couchs to treat motion sickenss and to induce sleep and to treat mild forms of parkinsons.

Zyrtec (Cetirizine)

Antihistamine Used to treat cold/allergy symptoms such as sneezing, itching, watery eyes, and runny nose. Also used to treat hives (urticaria).

Anticholinergics

Are another group of bronchodilators

Antitussives

Are drugs that suppress couching, possibly by reducing the activity of the cough center in the brain. Tessalon (Benzonate), Delsym (Dex), Robitussin

pH, PaO2, PaCO2, HCO3, SaO2

Arterial Blood Gas

To evaluate the effectiveness of prescribed therapies for a patient with ventilatory failure, which diagnostic test will be most useful to the nurse?

Arterial blood gas (ABG) analysis rationale: ABG analysis is most useful in this setting because ventilatory failure causes problems with CO2 retention, and ABGs provide information about the PaCO2 and pH. The other tests also may be done to help in assessing oxygenation or determining the cause of the patient's ventilatory failure.

________ is the deposition of lipds in the walls o ateries' lemens and form rough surfaces that may stimulate intravascular clot formation, and decreases blood flow to the afected organ.

Artherosclerosis

Bronchoscopy Preprocedure

Assess for allergies to anticoags or anesthetics Obtain consent Remove dentures NPO (usually 8-12) to reduce risk of aspiration Administer medication (lidocaine or throat sprays)

When assessing a patient with chronic lung disease, the nurse finds a sudden onset of agitation and confusion. Which action should the nurse take first?

Assess oxygenation using pulse oximetry. rationale: Since agitation and confusion are frequently the initial indicators of hypoxemia, the nurse's initial action should be to assess oxygen saturation. The other actions also are appropriate, but assessment of oxygenation takes priority over other assessments and notification of the health care provider.

The physician has ordered a thyroid scan to confirm the diagnosis. Before the procedure, the nurse should:

Assess the client for allergies

Identify what is included during the assessment phase of the nursing process for a cardiopulmonary focus.

Assessment • In-depth history of the client's normal and present cardiopulmonary function • Past impairments in circulatory or respiratory functioning • Patient history including a review of drug, food, and other allergies • Physical examination of the client's cardiopulmonary status reveals the extent of existing signs and symptoms. • Use PQRST for pain / HPI for other symptoms • Review of laboratory and diagnostic test results

20. A male client with chronic obstructive pulmonary disease (COPD) is recovering from a myocardial infarction. Because the client is extremely weak and can't produce an effective cough, the nurse should monitor closely for:

Atelectasis.

9. A finding indicating to the nurse that a 22-year-old patient with respiratory distress is in acute respiratory failure includes a a. shallow breathing pattern. b. partial pressure of arterial oxygen (PaO2) of 45 mm Hg. c. partial pressure of carbon dioxide in arterial gas (PaCO2) of 34 mm Hg. d. respiratory rate of 32/min.

B Rationale: The PaO2 indicates severe hypoxemia and that the nurse should take immediate action to correct this problem. Shallow breathing, rapid respiratory rate, and low PaCO2 can be caused by other factors, such as anxiety or pain. Cognitive Level: Application Text Reference: p. 1806 Nursing Process: Assessment NCLEX: Physiological Integrity

13. When the nurse is caring for an obese patient with left lower-lobe pneumonia, gas exchange will be best when the patient is positioned a. on the left side. b. on the right side. c. in the high-Fowler's position. d. in the tripod position.

B Rationale: The patient should be positioned with the "good" lung in the dependent position to improve the match between ventilation and perfusion. The obese patient's abdomen will limit respiratory excursion when sitting in the high-Fowler's or tripod positions. Cognitive Level: Comprehension Text Reference: pp. 1809-1810 Nursing Process: Implementation NCLEX: Physiological Integrity

23. The nurse obtains the vital signs for a patient admitted 2 days ago with gram-negative sepsis: temperature 101.2° F, blood pressure 90/56 mm Hg, pulse 92, respirations 34. Which action should the nurse take next? a. Notify the health care provider of the patient's vital signs. b. Obtain oxygen saturation using pulse oximetry. c. Document the vital signs and continue to monitor. d. Administer PRN acetaminophen (Tylenol) 650 mg.

B Rationale: The patient's increased respiratory rate in combination with the admission diagnosis of gram-negative sepsis indicates that acute respiratory distress syndrome (ARDS) may be developing; the nurse should check for hypoxemia, a hallmark of ARDS. The health care provider should be notified after further assessment of the patient. Documentation and continued monitoring of the vital signs are needed but do not constitute an adequate response to the patient situation. Tylenol administration is appropriate but not the highest priority for this patient. Cognitive Level: Application Text Reference: pp. 1813-1814 Nursing Process: Implementation NCLEX: Physiological Integrity

11. A patient with hypercapnic respiratory failure has a respiratory rate of 8 and an SpO2 of 89%. The patient is increasingly lethargic. Which collaborative intervention will the nurse anticipate? a. Administration of 100% oxygen by non-rebreather mask b. Endotracheal intubation and positive pressure ventilation c. Insertion of a mini-tracheostomy with frequent suctioning d. Initiation of bilevel positive pressure ventilation (BiPAP)

B Rationale: The patient's lethargy, low respiratory rate, and SpO2 indicate the need for mechanical ventilation with ventilator-controlled respiratory rate. Administration of high flow oxygen will not be helpful because the patient's respiratory rate is so low. Insertion of a mini-tracheostomy will facilitate removal of secretions, but it will not improve the patient's respiratory rate or oxygenation. BiPAP requires that the patient initiate an adequate respiratory rate to allow adequate gas exchange. Cognitive Level: Application Text Reference: pp. 1807-1808, 1810 Nursing Process: Planning NCLEX: Physiological Integrity

A nurse has received a client who has had a chest tube placed. While assessing the client, the nurse notices that the client's chest tube has become dislodged. Which of the following actions should the nurse take first? A. Place the tubing into sterile water to restore the water seal. B. Apply sterile gauze to the site. C. Tape or clamp all connections. D. Assess the client's respiratory status.

B. Apply sterile gauze to the site

What physical sign of COPD might you observe due to the excessive use of accessory muscles to breath?

Barrel chest

Why are there no adventitious lung sounds in the beginning stages of ARDS?

Because the initial edema initially occurs in the interstitial spaces and not the airways.

Antihistamines

Benedryl- crosses blood brain barrier clariton, Zyrtec- doesn't cross blood brain barrier. Grapefruit juice inhibits metabolism of meds and makes pt toxic

Expected Pharmacological Action of Beta2-Adrenergic Agonists

Beta2-adrenergic agonists act by selectively activating the beta2-receptors in the bronchial smooth muscle, resulting in bronchodilation. As a result of this: Bronchospasm is relieved. Histamine release is inhibited. Ciliary motility is increased.

A 28 year old male has been found wandering around in a confused pattern. The male is sweaty and pale. Which of the following tests is most likely to be performed first?

Blood Sugar test

Nurse Joy is caring for a client after a bronchoscopy and biopsy. Which of the following signs, if noticed in the client, should be reported immediately to the physician?

Blood-streaked sputum

Formoterol and salmeterol

Both are long-acting beta2-agonist inhalers. These inhalers are used every 12 hr for long-term control and are not to be used to abort an asthma attack. A short-acting beta2-agonist should be used if clients need to treat an acute attack.

A client is admitted to the emergency department with a suspected cervical spine fracture at the C3 level. The nurse is most concerned about the client's ability to:

Breathe

Albuterol (Pro Air)

Bronchodilator Relaxes muslces in the airways and increase airflow to the lungs. Used to treat or provent bronchospasm in people with reversible obstructive airway disease.

Theophylline (Elixophyllin)

Bronchodilator Relaxtion, muslces in lungs and chest, making the lungs less sensitive to allergens and often causes of bronchospasm. Treat symptoms: wheezing, SOB caused by asthma, bronchitits, emphysema, and other breathing problems.

Medications include:

Bronchodilator agents such as beta2-adrenergic agonists, methylxanthines, inhaled anticholinergics, and anti-inflammatory agents such as glucocorticoids, mast cell stabilizers, and leukotriene modifiers.

A nurse is caring for a client following a bronchoscopy. Which of the following client findings should the nurse report to the primary care provider?

Bronchospasms *Bronchospasms may indicate difficulty maintaining a patent airway. This should be reported to the primary care provider immediately. Blood-tinged sputum, a dry, productive cough, and a sore throat are expected findings.

24. Which of these nursing actions included in the care of a mechanically ventilated patient with acute respiratory distress syndrome (ARDS) is most appropriate for the RN to delegate to an experienced LPN/LVN working in the intensive care unit? a. Placing the patient in the prone position b. Assessment of patient breath sounds c. Administration of enteral tube feedings d. Obtaining the pulmonary artery pressures

C Rationale: Administration of tube feedings is included in LPN/LVN education and scope of practice and can be safely delegated to an LPN/LVN who is experienced in caring for critically ill patients. Placing a patient who is on a ventilator in the prone position requires multiple staff and should be supervised by an RN. Assessment of breath sounds and obtaining pulmonary artery pressures require advanced assessment skills and should be done by the RN caring for a critically ill patient. Cognitive Level: Application Text Reference: pp. 1816-1818 Nursing Process: Implementation NCLEX: Safe and Effective Care Environment

17. All the following medications are ordered for a mechanically ventilated patient with acute respiratory distress syndrome (ARDS) and acute renal failure. Which medication should the nurse discuss with the health care provider before administration? a. IV ranitidine (Zantac) 50 mg IV b. sucralfate (Carafate) 1 g per nasogastric tube c. IV gentamicin (Garamycin) 60 mg d. IV methylprednisolone (Solu-Medrol) 40 mg

C Rationale: Gentamicin, which is one of the aminoglycoside antibiotics, is potentially nephrotoxic, and the nurse should clarify the drug and dosage with the health care provider before administration. The other medications are appropriate for the patient with ARDS. Cognitive Level: Application Text Reference: p. 1816 Nursing Process: Implementation NCLEX: Physiological Integrity

1. It will be most important for the nurse to check pulse oximetry for which of these patients? a. A patient with emphysema and a respiratory rate of 16 b. A patient with massive obesity who is refusing to get out of bed c. A patient with pneumonia who has just been admitted to the unit d. A patient who has just received morphine sulfate for postoperative pain

C Rationale: Hypoxemia and hypoxemic respiratory failure are caused by disorders that interfere with the transfer of oxygen into the blood, such as pneumonia. The other listed disorders are more likely to cause problems with hypercapnia because of ventilatory failure. Cognitive Level: Application Text Reference: pp. 1799-1800 Nursing Process: Assessment NCLEX: Physiological Integrity

3. When a patient is diagnosed with pulmonary fibrosis, the nurse will teach the patient about the risk for poor oxygenation because of a. too-rapid movement of blood flow through the pulmonary blood vessels. b. incomplete filling of the alveoli with air because of reduced respiratory ability. c. decreased transfer of oxygen into the blood because of thickening of the alveoli. d. mismatch between lung ventilation and blood flow through the blood vessels of the lung.

C Rationale: Pulmonary fibrosis causes the alveolar-capillary interface to become thicker, which increases the amount of time it takes for gas to diffuse across the membrane. Too-rapid pulmonary blood flow is another cause of shunt but does not describe the pathology of pulmonary fibrosis. Decrease in alveolar ventilation will cause hypercapnia. Ventilation and perfusion are matched in pulmonary fibrosis; the problem is with diffusion. Cognitive Level: Application Text Reference: p. 1802 Nursing Process: Implementation NCLEX: Physiological Integrity

16. When caring for a patient who developed acute respiratory distress syndrome (ARDS) as a result of a urinary tract infection (UTI), the nurse is asked by the patient's family how a urinary tract infection could cause lung damage. Which response by the nurse is appropriate? a. "The infection spread through the circulation from the urinary tract to the lungs." b. "The urinary tract infection produced toxins that damaged the lungs." c. "The infection caused generalized inflammation that damaged the lungs." d. "The fever associated with the infection led to scar tissue formation in the lungs."

C Rationale: The pathophysiologic changes that occur in ARDS are thought to be caused by inflammatory and immune reactions that lead to changes at the alveolar-capillary membrane. ARDS is not directly caused by infection, toxins, or fever. Cognitive Level: Application Text Reference: p. 1813 Nursing Process: Implementation NCLEX: Physiological Integrity

21. When prone positioning is used in the care of a patient with acute respiratory distress syndrome (ARDS), which information obtained by the nurse indicates that the positioning is effective? a. The skin on the patient's back is intact and without redness. b. Sputum and blood cultures show no growth after 24 hours. c. The patient's PaO2 is 90 mm Hg, and the SaO2 is 92%. d. Endotracheal suctioning results in minimal mucous return.

C Rationale: The purpose of prone positioning is to improve the patient's oxygenation as indicated by the PaO2 and SaO2. The other information will be collected but does not indicate whether prone positioning has been effective. Cognitive Level: Application Text Reference: pp. 1817-1818 Nursing Process: Evaluation NCLEX: Physiological Integrity

A nurse is caring for a client who has secretions in the airway. Which of the following is the most effective method for clearing the secretions? A. Endotracheal suction B. Oropharyngeal suction C. Deep breathing and coughing D. Nasopharyngeal suction

C. Deep breathing and coughing

A nurse should measure a clients airway depth for nasopharyngeal and nasotracheal suctioning is by A. determining the distance from the nares to the sternum. B. determining the distance from the corner of the mouth to the earlobe. C. determining the distance from the tip of the nose to the earlobe. D. inserting the catheter until resistance is met.

C. determining the distance from the tip of the nose to the earlobe

In which patients do you hear crackles?

CHF, pulmonary edema

The lung chemical(s) are/is?

CO2

Hypoxic Drive

COPDers drive to breathe. It is their lack of oxygen that keeps them trying to breathe. Can kill a COPDer by giving them too much oxygen b/c causes them to lose their hypoxic drive.

The nurse is caring for a client who has undergone cardiac catheterization. The client says to the nurse, "The doctor said my cardiac output was 5.5 L/min. What is normal cardiac output?" Which of the following is the nurse's best response? A) "It is best to ask your doctor." B) "Did the test make you feel upset?" C) "The normal cardiac output for an adult is 4 to 6 L/min." D) "Are you able to explain why are you asking this question?

CORRECT ANSWER IS C The client asked a direct question that the nurse should be able to answer. Normal cardiac output for an adult is 4 to 6 L/min. Questions regarding diagnosis and prognosis may be referred to physicians. There is no harm in answering this question. When using therapeutic communication, the nurse should never ask a client to justify his or her feelings by inquiring why a question was asked. There is no evidence that this client is upset

Symptoms associated with anemia include which of the following? (Select all that apply.) A) Increased breathlessness B) Decreased breathlessness C) Increased activity tolerance D) Decreased activity tolerance

CORRECT ANSWERS ARE A & D Clients with anemia have fatigue, decreased activity tolerance, and increased breathlessness, as well as pallor (especially seen in the conjunctiva of the eye) and an increased heart rate.

Which of the following assessment data indicate that the client's airway needs suctioning? (Select all that apply.)

CORRECT ANSWERS ARE A, C & E Suctioning is necessary when the client is unable to clear respiratory secretions from the airways. Signs that a client's airway needs suctioning include a change in respiratory rate or adventitious sounds, nasal secretions, gurgling, drooling, restlessness, gastric secretions or vomitus in the mouth, and coughing without clearance of secretions from the airway.

A client with chronic obstructive pulmonary disease (COPD) is experiencing dyspnea and anxiety. The nurse helps the client to breathe better by doing which of the following? (Select all that apply.) A) Implementing guided imagery B) Instructing the client to perform pursed-lip breathing C) Elevating the head of the bed to semi-Fowler's or Fowler's position

CORRECT ANSWERS ARE B & C A) Implementing guided imagery B) Instructing the client to perform pursed-lip breathing C) Elevating the head of the bed to semi-Fowler's or Fowler's position Elevating the head of the bed to Fowler's position (45-degree angle) or semi-Fowler's position (30- to 45-degree angle) causes the diaphragm to lower from gravity and thus increases the space for lung expansion. Pursed-lip breathing prolongs exhalation and maintains the alveoli open longer, thus extending the period of oxygen and carbon dioxide exchange. Too high an elevation of the head of the bed could force the diaphragm into the thorax and reduce lung expansion. Fluids could help liquify the pulmonary secretions in the future, but right now the client needs more acute care. Guided imagery may help in the future, but now is not the time to implement this intervention.

The nurse suspects left-sided heart failure in a newly admitted client when the nurse notes which of the following symptoms? (Select all that apply.) A) Distended neck veins B) Bilateral crackles in the lungs C) Weight gain of 2 lb in past 2 days D) Shortness of breath, especially at night

CORRECT ANSWERS ARE B & D Left-sided heart failure results in ineffective ejection of blood from the left ventricle. This causes a backup of blood into the lungs. Thus, symptoms of left-sided heart failure are usually related to the lungs.

Diagnostic studies

CT Scan, V/Q scan, D-dimer, pulmonary angiography

Diagnostics for Pneumonia

CXR, physical exam and history. Sputum cultures. Blood cultures for seriously ill pts

A nurse is caring for a client following a tracheotomy 2 days ago. The nurse enters the room and notices that the tracheostomy tube is no longer in place. Describe the actions the nurse should take.

Call for assistance Assess respiratory rate/effort/color/Sa02 Obtain obturator and a spare tracheostomy tube and reinsert, if possible

Which medication would the nurse teach a patient to wear protective clothing, sunglasses, and a hat.

Captopril (Capoten)

Fever increases the tissues' need for oxygen, and as a result:

Carbon dioxide production increases Fever increases the tissues' need for oxygen, and as a result, carbon dioxide production increases. When fever persists, the metabolic rate remains high and the body begins to break down protein stores, which results in muscle wasting and decreased muscle mass.

Respiratory Alkalosis info

Caused by hyperventilation.

Where lung cancer is most likely found

Cell growth is slow. can take 8-10 years for tumor to reach 1cm which is detectable by Xray. Mostly found in Upper lobes.

Following a bronchoscopy, a client is sleepy but asks for a drink. Which of the following should be the nurse's first action? Auscultate the client's bowel sounds. Find out if a diet has been prescribed. Check the client's gag reflex. Ask the client which clear liquid she prefers.

Check the client's gag reflex.

What part of the cardiac cycle does the "lubb" of the characteristic lubb-dupp" heart sound indicate?

Closure of the AV valves

What is a sign of chronic cyanosis?

Clubbing of the fingers

What is a pneumothorax? Nursing Actions...

Collapsed lung; can occur due to injury Monitor for s/s of pneumothorax: diminshed breath sounds; CXR results

Interactions with Prednisone

Concurrent use of potassium-depleting diuretics increases the risk of hypokalemia. Concurrent use of NSAIDs increases the risk of GI ulceration. Concurrent use of glucocorticoids and hypoglycemic agents (oral and insulin) will counteract the effects.

What are some S&S of pneumonia in older adults?

Confusion, lethargy, anorexia, rapid respiratory rate

What cardiac condition can result from COPD?

Cor Pulmonale

The nurse is doing an admission assessment on a new residne to an extended care facility. The patients face and shoulders seem to have a lot of fat, but the patients arms and legs are thin. Excess of which hormone might be involved. T

Cortisol

5 Cardinal Signs for Pulmonary disorder (taken in history)

Cough, Phlem, Wheeze, SOB, Chest pain

Explain the water seal

Created by adding sterile fluid to a chamber (up to 2cm). The water seal allows air to exit from the pleural space on exhalation and stop air from entering with inhalation.

A client is admitted to the emergency department following a motorcycle crash. The nurse notes a crackling sensation when palpating the skin on the client's right chest. The nurse notifies the charge nurse and documents the presence of which of the following? A friction rub Crackles Crepitus Tactle fremitus

Crepitus

Which of the following are indications that a nurse should suction a client? (Select all that apply.) Spontaneous cough Cyanosis SaO2 greater than 95% Tachypnea Visualization of secretions

Cyanosis Tachypnea Visualization of secretions

7. A patient with chronic obstructive pulmonary disease (COPD) arrives in the emergency department complaining of acute respiratory distress. When monitoring the patient, which assessment by the nurse will be of most concern? a. The patient is sitting in the tripod position. b. The patient has bibasilar lung crackles. c. The patient's pulse oximetry indicates an O2 saturation of 91%. d. The patient's respiratory rate has decreased from 30 to 10/min.

D Rationale: A decrease in respiratory rate in a patient with respiratory distress suggests the onset of fatigue and a high risk for respiratory arrest; therefore, the nurse will need to take immediate action. Patients who are experiencing respiratory distress frequently sit in the tripod position because it decreases the work of breathing. Crackles in the lung bases may be the baseline for a patient with COPD. An oxygen saturation of 91% is common in patients with COPD and will provide adequate gas exchange and tissue oxygenation. Cognitive Level: Application Text Reference: p. 1804 Nursing Process: Assessment NCLEX: Physiological Integrity

4. A patient is diagnosed with a large pulmonary embolism. When explaining to the patient what has happened to cause respiratory failure, which information will the nurse include? a. "Oxygen transfer into your blood is slow because of thick membranes between the small air sacs and the lung circulation." b. "Thick secretions in your small airways are blocking air from moving into the small air sacs in your lungs." c. "Large areas of your lungs are getting good blood flow but are not receiving enough air to fill the small air sacs." d. "Blood flow though some areas of your lungs is decreased even though you are taking adequate breaths."

D Rationale: A pulmonary embolus limits blood flow but does not affect ventilation, leading to a ventilation-perfusion mismatch. The response beginning, "Oxygen transfer into your blood is slow because of thick membranes" describes a diffusion problem. The remaining two responses describe ventilation-perfusion mismatch with adequate blood flow but poor ventilation. Cognitive Level: Application Text Reference: p. 1802 Nursing Process: Implementation NCLEX: Physiological Integrity

20. Which statement by the nurse when explaining the purpose of positive end-expiratory pressure (PEEP) to the family members of a patient with ARDS is correct? a. "PEEP will prevent fibrosis of the lung from occurring." b. "PEEP will push more air into the lungs during inhalation." c. "PEEP allows the ventilator to deliver 100% oxygen to the lungs." d. "PEEP prevents the lung air sacs from collapsing during exhalation."

D Rationale: By preventing alveolar collapse during expiration, PEEP improves gas exchange and oxygenation. PEEP will not prevent the fibrotic changes that occur with ARDS, push more air into the lungs, or change the fraction of inspired oxygen (FIO2) delivered to the patient. Cognitive Level: Comprehension Text Reference: p. 1817 Nursing Process: Planning NCLEX: Physiological Integrity

10. While caring for a patient who has been admitted with a pulmonary embolism, the nurse notes a change in the patient's arterial oxyhemoglobin saturation (SpO2) from 94% to 88%. The nurse will a. assist the patient to cough and deep-breathe. b. help the patient to sit in a more upright position. c. suction the patient's oropharynx. d. increase the oxygen flow rate.

D Rationale: Increasing oxygen flow rate will usually improve oxygen saturation in patients with ventilation-perfusion mismatch, as occurs with pulmonary embolism. Because the problem is with perfusion, actions that improve ventilation, such as deep-breathing and coughing, sitting upright, and suctioning, are not likely to improve oxygenation. Cognitive Level: Application Text Reference: pp. 1802, 1807 Nursing Process: Implementation NCLEX: Physiological Integrity

18. After prolonged cardiopulmonary bypass, a patient develops increasing shortness of breath and hypoxemia. To determine whether the patient has acute respiratory distress syndrome (ARDS) or pulmonary edema caused by left ventricular failure, the nurse will anticipate assisting with a. positioning the patient for a chest radiograph. b. drawing blood for arterial blood gases. c. obtaining a ventilation-perfusion scan. d. inserting a pulmonary artery catheter.

D Rationale: Pulmonary artery wedge pressure will remain at normal levels in the patient with ARDS because the fluid in the alveoli is caused by increased permeability of the alveolar-capillary membrane rather than by the backup of fluid from the lungs (as occurs in cardiogenic pulmonary edema). The other tests will not help in differentiating cardiogenic from noncardiogenic pulmonary edema. Cognitive Level: Application Text Reference: p. 1815 Nursing Process: Implementation NCLEX: Physiological Integrity

6. When assessing a patient with chronic lung disease, the nurse finds a sudden onset of agitation and confusion. Which action should the nurse take first? a. Monitor the patient every 10 to 15 minutes. b. Notify the patient's health care provider immediately. c. Attempt to calm and reassure the patient. d. Assess vital signs and pulse oximetry.

D Rationale: The nurse needs to collect additional clinical data to share with the health care provider and to start interventions quickly if appropriate (e.g., increased oxygen flow if hypoxic). The change in the patient's neurologic status may indicate deterioration in respiratory function, and the health care provider should be notified immediately but only after some additional information is obtained. Monitoring the patient and attempting to calm the patient are appropriate actions, but they will not prevent further deterioration of the patient's clinical status and may delay care. Cognitive Level: Application Text Reference: pp. 1804-1805 Nursing Process: Assessment NCLEX: Physiological Integrity

12. A patient in acute respiratory failure as a complication of COPD has a PaCO2 of 65 mm Hg, rhonchi audible in the right lung, and marked fatigue with a weak cough. The nurse will plan to a. allow the patient to rest to help conserve energy. b. arrange for a humidifier to be placed in the patient's room. c. position the patient on the right side with the head of the bed elevated. d. assist the patient with augmented coughing to remove respiratory secretions.

D Rationale: The patient's assessment indicates that assisted coughing is needed to help remove secretions, which will improve PaCO2 and will also help to correct fatigue. If the patient is allowed to rest, the PaCO2 will increase. Humidification may help loosen secretions, but the weak cough effort will prevent the secretions from being cleared. The patient should be positioned with the good lung down to improve gas exchange. Cognitive Level: Application Text Reference: p. 1809 Nursing Process: Planning NCLEX: Physiological Integrity

15. The nurse is caring for a patient who was hospitalized 2 days earlier with aspiration pneumonia. Which assessment information is most important to communicate to the health care provider? a. The patient has a cough that is productive of blood-tinged sputum. b. The patient has scattered crackles throughout the posterior lung bases. c. The patient's temperature is 101.5° F after 2 days of IV antibiotic therapy. d. The patient's SpO2 has dropped to 90%, although the O2 flow rate has been increased.

D Rationale: The patient's dropping SpO2 despite having an increase in FIO2 indicates the possibility of acute respiratory distress syndrome (ARDS). The patient's blood-tinged sputum and scattered crackles are not unusual in a patient with pneumonia, although they do require continued monitoring. The continued temperature elevation indicates a possible need to change antibiotics, but this is not as urgent a concern as the progression toward hypoxemia despite an increase in O2 flow rate. Cognitive Level: Application Text Reference: p. 1815 Nursing Process: Assessment NCLEX: Physiological Integrity

A nurse is caring for a client following a bronchoscopy. Which of the following client findings should the nurse report to the primary care provider? A. Blood-tinged sputum B. Dry, nonproductive cough C. Sore throat D. Bronchospasms

D. Bronchospasms

A nurse is assisting a provider with the removal of a chest tube. Which of the following should the nurse instruct the client to do? A. Lie on his left side during removal. B. Hold his breath. C. Inhale deeply during removal. D. Perform the Valsalva maneuver during removal.

D. Perform the Valsalva maneuver during removal

What are the causes of metabolic acidosis? (4)

DKA (d/t ketone production), starvation (d/t ketone production), renal failure, severe diarrhea (intestines contain a lot of bicarb)

Afrin (Oxymetazoline)

Decongestant Relieving nasal congestion due to the common cold, hay fever, other upper resp tract allergies or sinus infection.

Sudafed (Pseudophedrine)

Decongestant Stimulates blood vessels in the nasal passages. Used to treat nasal and sinus congestion of eustachain tubes.

If it take longer than 3 seconds for the color to return when assessing capillary refill, which of the following may be indicated?

Decreased arterial flow to the extremity

An older adult client is admitted with respiratory acidosis as a complication of chronic obstruction pulmonary disease (COPD). The nurse suspects that this is related to which of the following? Increased mucous secretions Decreased exhalation of carbon dioxide. Increased respiratory rate. Recent vomiting and diarrhea.

Decreased exhalation of carbon dioxide.

A female client must take streptomycin for tuberculosis. Before therapy begins, the nurse should instruct the client to notify the physician if which health concern occurs?

Decreased hearing acuity

Nurse Ronn is assessing a client with possible Cushing's syndrome. In a client with Cushing's syndrome, the nurse would expect to find:

Deposits of adipose tissue in the trunk and dorsocervical area

A thirty five year old male has been an insulin-dependent diabetic for five years and now is unable to urinate. Which of the following would you most likely suspect?

DiabeticNeuropathy

Side/Adverse effect of Beclomethasone dipropionate

Difficulty speaking, hoarseness, and candidiasis • Advise clients to use a spacer with MDI. • Advise clients to rinse mouth or gargle with water or salt water after use. • Advise clients to monitor for redness, sores, or white patches and to report to provider if they occur. Candidiasis may be treated with nystatin oral suspension

What drug levels are raised in the presence of isoniazid?

Dilantin

Which kind of drainage system is now most commonly used?

Disposable

The nurse is caring for a client with a chest tube in the right thorax. On first assessment the nurse notes that there is bubbling in the water-seal chamber. This client is scheduled to undergo a chest x-ray examination, and the transporters have arrived to take him by wheelchair to the radiology department. The nurse considers whether the chest tube should be clamped or not during the trip to the radiology department. The nurse makes the which correct decision?

Do not clamp the chest tube and disconnect it from the wall suction. A bubbling chest tube (in the water-seal portion) should never be clamped because it provides the only exit for air accumulating in the pleural space. If the tube is clamped, tension pneumothorax could occur, which could be fatal. There is no advantage to clamping the chest tube but venting the system. Clamping of the chest tube prevents communication of the chest tube with the venting system or with the wall suction. There is no such thing as "temporary suction" for a chest tube system.

Accurate teaching for a Cardiac Holter monitor is:

Do not take a shower or bath while on the monitor.

Acid-fast smear and culture

Done on sputum for acid-fast bacilli. 3 early morning specimens are used.

Antihistamine

Drugs that treat allergy symptoms

Client presentation for possible chest tube insertion

Dyspnea Distended neck veins Poor circulation Cough

Which of the following are causes for concern following a thoracentesis? (Select all that apply.) Dyspnea Localized bloody drainage contained on the dressing Fever Hypotension SaO2 of 95% Soreness around puncture site

Dyspnea Fever Hypotension

What are some symptoms of respiratory failure in adults?

Dyspnea/ tachypnea, cyanosis, intercostal or sternal retractions

What is a nursing priority when administering Captopril (Capoten)?

Edema with HF and decreased BP with HTN.

To decrease the risk for ventilator-associated pneumonia, which action will the nurse include in the plan of care for a patient who requires intubation and mechanical ventilation?

Elevate head of bed to 30 to 45 degrees. rationale: Elevation of the head decreases the risk for aspiration. PEEP is frequently needed to improve oxygenation in patients receiving mechanical ventilation. Suctioning should be done only when the patient assessment indicates that it is necessary. Enteral feedings should provide adequate calories for the patient's high energy needs.

What TB drug requires a vision check prior to starting and may need to be taken for 1-2 years?

Ethambutol

A 25-year-old client with Grave's disease is admitted to the unit. What would the nurse expect the admitting assessment to reveal?

Exophthalmos

Purpose of Glucocorticoids

Expected Pharmacological Action ◯ These medications prevent inflammation, suppress airway mucus production, and promote responsiveness of beta2 receptors in the bronchial tree. ◯ The use of glucocorticoids does not provide immediate effects, but rather promotes decreased frequency and severity of exacerbations and acute attacks.

Mucinex (Guaifenesin)

Expectorant USed to relieve the S/S of cough and mucus in the chest due to colds, flu or hay fever. Works by phelgm in the lungs and making it less stiky and easier to cough up. This reduces chest congestion making coughs more effective.

Pulmonary Function Test (PFT)

FEV1= 80% of estimated for a normal person. COPDers have a significantly lower percentage

Initiates growth of ova in ovarian follicles and increases secretion of estrogen by follicle cells?

FSH(follicle stimulating hormone)

Initiates sperm production in the testes?

FSH(follicle stimulating hormone)

Kussamual

Fast and deep respirations. seen in pts with ketone acidosis- diabetics

Which of these assessment findings is an atypical symptom of myocardial infarction when chest pain is not present

Fatigue

3 Constitutional symptoms (taken in history)

Fever, Sweat, Loss of weight

While observing a client who is unconscious following major trauma, the nurse notes that a portion of the client's chest pulls inward on inspiration. On expiration, the same portion expands outward. The nurse documents the presence of which of the following? Symmetrical chest movement. Intercostal retractions. Flail chest. Cheyne-Stokes respirations

Flail chest.

Tamiflu (Oseltamivir) Relenza (Zanamivir)

Flue Med Antiviral medication that blocks the actions of influenza. 2 wks and older who had flu S/S >2days. Side effects: N/V, eye discomfort, insomnia, cough, diarrhea, dizziness, h/a

Carbon monoxide (CO) is a toxic inhalant that decreases the oxygen-carrying capacity of blood by:

Forming a strong bond with hemoglobin CO is the most common toxic inhalant and decreases the oxygen-carrying capacity of blood. In CO toxicity, hemoglobin strongly binds with carbon monoxide, creating a functional anemia. Because of the strength of the bond, carbon monoxide does not easily dissociate from hemoglobin, which makes hemoglobin unavailable for oxygen transport.

What does FIO2 stand for?

Fraction of Inspired oxygen concentration

A simple and cost-effective method for reducing the risks of stasis of pulmonary secretions and decreased chest wall expansion is:

Frequent change of position Changing the client's position frequently is a simple and cost-effective method for reducing the risk of pneumonia associated with stasis of pulmonary secretions and decreased chest wall expansion. Oxygen humidification, chest physiotherapy, and use of antiinfectives are all helpful, but are not cost effective.

Which medication is an example of an oral hypoglycemic agent?

Glypizide (Glucotrol)

Increases all of the following, cell division in tissues, rate of mitosis, transport rate of amino acids into cells, rate of protein synthesis, and use of fats for energy?

Growth hormone(GH)

What are the s/s of respiratory acidosis? (4)

HA, confusion, sleepy, hypoxia

Describe a single chamber drainage system

Has a water seal & a drainage collection in the same chamber

Describe a two chamber drainage system

Has a water seal and a drainage collection in separate chambers; allows for a larger collection of drainage.

Describe a three chamber drainage system

Has a water seal, drainage collections, and suction control in separate chambers.

An autoimmmune disorder that evenatually destroys thyroid tissue, leading to hypothyroidism?

Hashimoto's thyroiditis

A nurse is caring for a client who is admitted in an extremely anxious state. The client's arterial blood glass (ABG) values are pH 7.47, PO2 94, PCO2 30, and bicarbonate (HCO3)25. What should the nurse do? Give supplemental oxygen via nasal cannula. Monitor the client's fluid and electrolyte balance closely. Have the client breathe slowly into a paper bag. Administer sodium bicarbonate.

Have the client breathe slowly into a paper bag.

A nurse is preparing to obtain a sputum specimen from a male client. Which of the following nursing actions will facilitate obtaining the specimen?

Having the client take deep breaths

The nurse is planning care for a paient with cardiomyopathy. For which of the following complications of cardiomyopathy should the nurse collect data?

Heart Failure

Possible complications of an ABG

Hematoma; arterial occlusion; air embolism

How to position client with emphysema

High-Fowler's with arms supported by overbed table

A nurse is caring for a client who is being admitted following a tracheotomy. What equipment should a nurse make sure is in the room?

Humidified oxygen Pulse oximeter Suction equipment and catheters Spare tracheostomy tube Extra tracheostomy ties Normal saline Split 4x4 dressings

What initiates the stimulus to breath in normal person?

Hypercapnia

Chronic bronchitis and patho:

Hypersecretion of mucus and chronic productive cough for 3 months in 2 consecutive years. Chronic irritants = chronic inflammation leads to vasodilation of local arterioles, congestion, airway mucous membrane edema or swelling and bronchospasm.

What does central cyanosis indicate?

Hypoexmia

Releasing hormones location?

Hypothalamus

<90% indicates _________

Hypoxemia

What initiates the stimulus to breath in a person with COPD?

Hypoxia

Indications for O2 monitoring

Increased breathing workload wheezing coughing cyanosis continuous opioid use

Inflammation and edema:

Increased mucous production, coughing manages all the excess mucous production.

A pulse oximeter reading from a client diagnosed with smoke inhalation is 85% with a 40% face mask. The provider prescribes an increase of the oxygen to 50%. Because the client is a high risk for adult respiratory distress syndrome (ARDS), the nurse must observe the client for which of the following? Substernal chest pain. Increased restlessness. Apnea. Oxygen saturation of 95%

Increased restlessness

Chronic inflammation:

Increases number and size of mucous glands, which leads to large amounts of thick, tenacious sputum which gets harder to manage.

Function of the hormone ACTH(adrenocorticotropic hormone)?

Increases secretion of cortisol by the adrenal cortex.

What is the function(s) of TSH(thyroid stimulating hormone)?

Increases secretion of thyroxine and T3 by the thyroid gland

A client hospitalized with MRSA (methicillin-resistant staph aureus) is placed on contact precautions. Which statement is true regarding precautions for infections spread by contact?

Infection requires skin-to-skin contact and is prevented by hand washing, gloves, and a gown.

Increasing dyspnea and sputum:

Inflammation is a key component in development.

Airway obstruction comes from 2 sources:

Inflammatory response swelling the airway shut inside the lumen and/or hyperresponsive airways = bronchoconstriction of smooth muscles in the bronchioles, effectively shutting them closed from the outside walls.

Side/Adverse effects of Beta2-Adrenergic Agonists

Inhaled agents (short and long acting) have minimal adverse effects. Oral agents can cause tachycardia and angina because of activation of alpha1 receptors in the heart Tremors caused by activation of beta2 receptors in skeletal muscle

Explain a chest tube

Inserted into the pleural space to drain fluid, blood, or air; Reestablish a negative pressure; facilitate lung expansion; restore normal intrapleural pressure.

Findings upon exam of Emphysema

Inspection: "pink puffer", polycythemia, barrel chest, tripod position, use of accessory muscles. Palpation: decreased mvmt Percussion: Hyperresonance Auscultation: wheezes, crackles, rhonchi, distant breath sounds usually secondary diagnosis to COPD

Findings upon exam of COPD

Inspection: Cyanotic, Tripod position, big belly, pursed lip breathing, barrel chest, use of accessory muscles, Blue Bloater Palpation: decreased mvmt Percussion: Hyperresonance or dull over consolidation Auscultation: Crackles, rhonchi, wheezes, distant breath sounds -cough, sputum, dyspnea and/or history of exposure of risk factors of disease (smoking)

Findings upon exam of Atelectasis

Inspection: No change unless entire segment, lobe Palpation: small= no change, large= decreased mvmt decrease fremitus Percussion: dull over affected areas Auscultation: crackles (may disappear with deep breath sounds) absent sounds if large.

Findings upon exam of Pneumonia

Inspection: Tachypnea, accessory muscles, duskiness or cyanotic Palpation: increase fremitus over affected areas Percussion: Dull over affected areas Auscultation: Early= bronchiole sounds Late= crackles, rhonchi, egophany, whispered pectoriliquy. - Fever, shaking, chills, SOB, cough productive purulent sputum. Rust colored= pneumococcal and chest pain. Confusion in elderly.

Findings upon exam of Pulmonary Edema

Inspection: Tachypnea, cyanotic, labored breathing Palpate: decreased or normal mvmt Percussion: Dull or normal depending on amt of fluid Auscultation: fine or coarse crackles at bases moving upward as condition worsens -Crackles heard in R lung b/c of Left HF. Pulmonary edema leads or Right HF

Findings upon exam of Pleural effusion

Inspection: Tachypnea, use of accessory muscles Palpation: increase fremitus above effusion and absent over effusion. Percussion: Dull Auscultation: diminished or absent over effusion; egophony over effusion. -Progressive dyspnea and decreased mvmt on affected side. sometimes pain. -Treatment= thoracentesis

Findings upon exam of Pneumothorax

Inspection: dyspnea Palpation: decreased movement Percussion: hyperresonance Auscultation: diminished or absent breath sounds on affected side - Caused by trauma to chest. Pt will present with chest pain and cough

Findings upon exam of Pulmonary Fibrosis

Inspection: tachypnea Palpation: decreased movement Percussion: Normal Auscultation: crackles or sounds like velcro being pulled apart

Inhaled NSAIDS

Intal (cromolyn sodium)-anti-inflammatory Works by preventing the release of substances in the body that cause inflammation. Used to prvent asthma attack in people with bronchiol asthma, to prevent bronchospasm (wheezing, chest tightness, trouble breathing) caused by exercise, polluntants in the air or exposure to certain chemicals.

Asthma defined:

Intermittent and reversible airflow obstruction affecting the airway, but not the alveoli. Keep in mind what surrounds the bronchi and bronchioles - smooth muscle within the walls.

Which of the following is a classic symptom of peripheral arterial occlusive disease

Intermittent claudication

Location or Lingula

Invisible left middle lobe. Place one hand up against armpit so fingers are wrapping horizontally around ribs. Place second hand pressed up against other hand. Location of second hand is where lingula is located.

What are early signs of cerebral hypoxia?

Irritability and restlessness

If there is steady bubbling in the pleuravac what is going on.

It is hooked to suction.

A patient is returning from transesophgeal echocardiography. Which of the following would the nurse include in the patient's plan of care?

Keep the patient NPO until the gag reflect has returned.,

A male client suffers adult respiratory distress syndrome as a consequence of shock. The client's condition deteriorates rapidly, and endotracheal (ET) intubation and mechanical ventilation are initiated. When the high-pressure alarm on the mechanical ventilator sounds, the nurse starts to check for the cause. Which condition triggers the high-pressure alarm?

Kinking of the ventilator tubing

Causes all of the following: ovulation, the ruptured membranes to become corpus luteum, and increases secretion of progesterone by the corpus luteum?

LH(luteinizing hormone)

Increases secretion of testosterone by the interstitial cells of the testes?

LH(luteinizing hormone)

Once it has been identified:

LIfestyle changes may be required, getting rid of the household pet.

Singulair (Montelukast)

LM Prevent asthma attacks in adults and children as young as 12 months old. Also treat S/S year round allergies,

Increased CO2 = decreased ___________.

LOC or O2

Possible complications to a bronchoscopy

Laryngospasm Aspiration

The increase of resting blood pressure with age may contribute to which of the following?

Left sided Heart Failure

The nurse is caring for the client immediately after insertion of a permanent demand pacemaker via the right subclavian vein. The nurse takes care not to dislodge the pacing catheter by:

Limiting the movement and abduction of the right arm.

Flu Vaccine

Live virus=nasal mist Dead virus=injection Not given with egg allergy Incresase risk for reaction with Beta Blockers DO give in dominant arm.

Side/Adverse effects of LEUKOTRIENE MODIFIERS

Liver injury with use of zileuton (Zyflo) and zafirlukast (Accolate)

SIDE/ADVERSE EFFECTS of Inhaled Anticholinergics

Local anticholinergic effects (dry mouth, hoarseness)

A nurse is caring for a client who has acute respiratory failure and is being treated with mechanical ventilation. The low-pressure alarm on the ventilator begins to sound continuously. Which of the following is an appropriate nursing action? Call the respiratory therapist. Look for loose connections. Suction the client's airway secretions. Increase the oxygen concentration.

Look for loose connections.

In respiratory alkalosis, which chemical is causing the problem?

Losing too much CO2-pt is breathing too fast

What is a common lab finding in ARDS?

Low PO2

A patient with acute respiratory distress syndrome (ARDS) who is intubated and receiving mechanical ventilation develops a pneumothorax. Which action will the nurse anticipate taking?

Lower the positive end-expiratory pressure (PEEP). rationale: Because barotrauma is associated with high airway pressures, the level of PEEP should be decreased. The other actions will not decrease the risk for pneumothorax.

What is the leading cause of cancer related death in the US?

Lung cancer

With respiratory acidosis, which organ is sick and which organ will compensate?

Lungs are sick and kidneys will compensate

With respiratory alkalosis, which organ is sick and which organ will compensate?

Lungs are sick and the kidneys will compensate

Hemoglobin levels

Male= 13.2-17.3 g/dL Female= 11.7-16 g/dL

Inspiratory Capacity (IC)

Max amount of air that can be inhaled after a normal expiration. Normal: 3.5 L

Inspiratory Reserve Volume (IRV)

Max amount of volume of air that can be inhaled forcefully after normal inhale Norm: 3.0 L

Vital Capacity (VC)

Max volume of air that can be exhaled after max inhale usually higher in men Norm: 4.5 L

Interactions with Beta2-Adrenergic Agonists

Medication/Food Interactions: Use of beta-adrenergic blockers (propranolol) can negate effects of both medications. MAOIs and tricyclic antidepressants can increase the risk of tachycardia and angina.

How long do the lungs take to correct pH imbalance?

Minutes. Lungs respond fast.

A client diagnosed with pneumonia is prescribed chest physiotherapy (CPT) every 4 hr. In planning the client's care, the nurse understands that the purpose of CPT is which of the following? Encourage deep breaths Mobilize secretions in the airways Dilate the bronchioles Stimulate the cough reflex

Mobilize secretions in the airways

Bronchoscopy postprocedure

Monitor VS & level of consciousness, gag reflex during recovery. Monitor for fever (mild <24hr is common), productive cough, significant hemoptysis indicative of hemrrhage, hypoexmia. Perform oral hygiene For older adults: encourage coughing, deep breathing q2hrs.

What is tidaling?

Movement of the water level with respiration. It is expected in the water seal chamber.

A client with a diagnosis of congestive heart failure is seen in the clinic. The client is being treated with a variety of medications, icnluding digoxin (alnoxin) and furosemide (Lasix). Which finding? on data collection would lead the nurse to suspect that the client is hypokalemic.

Muscle weakness and leg cramps

Responsiblities before and after Bronchoscopy

NPO before bronchoscopy NPO until the get their gag reflex back to prevent aspiration After bronch can cough up blood tinged sputum. NO SMOKING for up to 24 hrs b/c it is an irritant that will dislodge blood clot from biopsy sight. might have sore throat after.

Which of the following can cause a low pulse oximetry reading? (Select all that apply.) Nail polish Inadequate peripheral circulation Hyperthermia Increased Hgb level Edema

Nail Polish Inadequate peripheral circulation Edema

which action by nurse is most important following hypohysectomy

Neurological

Do crackles clear with coughing?

No

A nurse is caring for a client transferred from the post anesthesia care unit following a left pneumonectomy for adenocarcinoma. Which type of chest drainage should the nurse anticipate the client will have? Bilateral chest tubes One chest tube on the operative side Two chest tubes on the operative side No chest drainage

No chest drainage

Is it ok to have continuous bubbling in the WATER chamber?

No, it indicates an air leak.

A male client abruptly sits up in bed, reports having difficulty breathing and has an arterial oxygen saturation of 88%. Which mode of oxygen delivery would most likely reverse the manifestations?

Non-rebreather mask

Mask that delivers the highest percentage of oxygen? What percentage?

Non-rebreather mask over 90%

What to do if an air embolism is suspected?

Notify MD; administer O2, obtain ABGs. Continue to assess client's resp status for any deterioration

What happens as COPD worsens?

O2 in the blood decreases causing hypoxemia and CO2 in the blood increases causing hypercarbia. This causes chronic respiratory acidosis which leads to metabolic alkalosis as compensation occurs.

What should you do if a hematoma occurs?

Observe for temperature changes, swelling, color, loss of pulse, or pain; Notify MD; Apply pressure to site

The nurse obtains the vital signs for a patient admitted 2 days ago with gram-negative sepsis: temperature 101.2° F, blood pressure 90/56 mm Hg, pulse 92, respirations 34. Which action should the nurse take next?

Obtain oxygen saturation using pulse oximetry. rationale: The patient's increased respiratory rate in combination with the admission diagnosis of gram-negative sepsis indicates that acute respiratory distress syndrome (ARDS) may be developing. The nurse should check for hypoxemia, a hallmark of ARDS. The health care provider should be notified after further assessment of the patient. Administration of the scheduled antibiotic and administration of Tylenol also will be done, but they are not the highest priority for a patient who may be developing ARDS.

Why is a thoracentesis performed?

Obtain specimens for diagnostic eval; instill medication into pleural space; remove fluid (effusion) or air for therapeutic relief of pleural pressure.

What is the Allen test?

Occlude both radial and ulnar arteries, release ulnar artery only and view hand for blanching. Tests for arterial insufficiency to the hands.

A patient has a nursing diagnosis of ineffective airway clearance related to thick, secretions. Which action will be best for the nurse to include in the plan of care?

Offer the patient fluids at frequent intervals. rationale: Since the reason for the poor airway clearance is the thick secretions, the best action will be to encourage the patient to improve oral fluid intake. The use of the incentive spirometer should be more frequent in order to facilitate the clearance of the secretions. The other actions also may be helpful in improving the patient's gas exchange, but they do not address the thick secretions that are causing the poor airway clearance.

COPD: Chronic bronchitis and emphysema:

One patient with COPD might have more of the bronchitis symptoms while another patient might have more of the emphysema symptoms; some may have the majority of one or the other, but that majority have a mixture.

If a person with ARDS is on a ventilator when should you suction?

Only when secretions are present?

Where do you hear atypical bronchial breath sounds?

Over areas of consolidation

A nurse is preparing to recceive a client who has had a chest tube placed. Which of the following items should the nurse have placed in the client's room? (Select all that apply.) Oxygen Sterile water Enclosed hemostat clamps Indwelling urinary catheter Occlusive dressing Suction source Bladder scan machine

Oxygen Sterile Water Enclosed hemostate clamps Occulsive dressing Suction Source

A nurse is caring for a client who is scheduled for a thoracentesis at the bedside. Which of the following items should the nurse ensure is in client's room? (Select all that apply.) Oxygen equipment Incentive spirometer Pulse oximeter Thoracentesis tray Suture removal kit

Oxygen equipment Pulse oximeter Theracentesis tray

The nurse is caring for a patient who was hospitalized 2 days earlier with aspiration pneumonia. Which assessment information is most important to communicate to the health care provider?

Oxygen saturation (SpO2) has dropped to 90% with administration of 100% O2 by non-rebreather mask. rationale: The patient's low SpO2 despite receiving a high fraction of inspired oxygen (FIO2) indicates the possibility of acute respiratory distress syndrome (ARDS). The patient's blood-tinged sputum and scattered crackles are not unusual in a patient with pneumonia, although they do require continued monitoring. The continued temperature elevation indicates a possible need to change antibiotics, but this is not as urgent a concern as the progression toward hypoxemia despite an increase in O2 flow rate.

A patient with ARDS who is receiving mechanical ventilation using synchronized intermittent mandatory ventilation (SIMV) has settings of fraction of inspired oxygen (FIO2) 80%, tidal volume 500, rate 18, and positive end-expiratory pressure (PEEP) 5 cm. Which assessment finding is most important for the nurse to report to the health care provider?

Oxygen saturation 99% rationale: The FIO2 of 80% increases the risk for oxygen toxicity. Since the patient's O2 saturation is 99%, a decrease in FIO2 is indicated to avoid toxicity. The other patient data would be typical for a patient with ARDS and would not need to be urgently reported to the health care provider.

This causes contraction of smooth muscle in the uterus and mammory glands?

Oxytocin

What is an expected lab finding for hypercapnia?

PCO2 >45

Pulmonary Function Test

PFT measures lung volumes and airflow. Results are used to diagnose pulmonary disease, monitor disease progression, evaluate disability and evaluate response to bronchodilators. Normal= 80-120%

TB Meds

PZA (Pyrazinamide) Rifampin-possible orange urine (side effects: hepatitis, GI disturbance) INH (Ionozide)-Can cause an elevation of hepatic enzymes and hepatitis. Monitor when therapy is initiated and during the first 3 months.

Position for Thoracentesis

Patient is tripod position by sitting up and leaning on bed side table and feet suported.

ABG procedures

Perform Allens test to verify circulation to hand before arterial puncture; is + with return of circulation; - without return. Place arterial specimen in ice and water: send to lab STAT. Accessing the radial artery can be difficult in older clients (b/c of impaired circulation) Hold pressure over artery for 5 minutes (20 mins if client is on anticoag therapy) Monitor site for: bleeding, loss of pulse, swelling, change in color or temperature. Document interventions and client response

Drugs that decrease effects of Methylzanthines

Phenobarbituates, beta agonists, riphamin (TB drug), nicotine

Indications for a chest tube

Pneumothorax (collapsed lung) Hemothorax (blood in the lung) Postoperative chest drainage (thoracotomy or open heart surgery) Pleural effusion (fluid in the lungs) Lung abscess (necrotic lung tissue)

Priority risk to monitor for after thoracentesis?

Pneumothorax AEB thoracentesis

What are the lab values that denote hypoxemia in ARDS?

Po2 <50 with FiO2 >60

Bronchoscopy Intraprocedure

Position client in sitting position Monitor VS (Sedation given to older adults w/resp insufficiency may precipitate resp arrest.

ADH and oxytocin are secreted from which gland?

Posterior pituitary gland

Indication for an ABG

Potential Dx (resp, endo, neuro) Monitoring treatments

Life threatening condition:

Potential increased airway obstruction and respiratory failure, if this occurs multiple times, they can damage brain tissue.

A nurse is preparing to ambulate a postoperative client following cardiac surgery. The nurse plans to do which of he following to enable the client to best tolerate the ambulation.

Premedicate the paient with an analgesic prior to ambulating

This hormone stimulates milk production by the mammary glands?

Prolactin

A nurse is reinforcing teaching for a client with emphysema about pursed-lip breathing. The nurse reminds the client that pursed-lip breathing will help do which of the following? Promote carbon dioxide elimination. Increase oxygen intake. Use the intercostal muscles. Strengthen the diaphragm

Promote carbon dioxide elimination.

Expectorants

Promoting or facilitating the seretion or expulsion of phlegm, mucus or other matter from resp. tract.

To meet the goal of maintaining an adequate nutritional status for a client who has pneumonia, which measure should the nurse include when assisting in planning care to promote oral intake? Provide oral hygiene care after respiratory aerosol therapy treatments and before meals. Severe nutritious foods at mealtimes and discourage between-meal snacking. Schedule respiratory aerosol treatments and chest physiotherapy just before meals. Select meals for the client to ensure that the meals are well-balanced.

Provide oral hygiene care after respiratory aerosol therapy treatments and before meals

A nurse is caring for a client who has a tracheotomy. Which of the following interventions should the nurse include? (Select all that apply.) Use medical aseptic technique when performing tracheostomy care. Change tracheostomy ties each time tracheostomy care is given. Provide the client with materials for nonverbal communication. Keep pressure greater than 30 mm Hg. Clean the stoma site with half-strength hydrogen peroxide followed by 0.9% NaCl.

Provide the client with materials for nonverbal comm. Clean the stoma site with half-strength peroxide followed by NaCl

Removal of chest tube

Pt can NOT breath during procedure. Pt should be medicated before hand. if pt breaths can cause a pneumothorax.

Noncompliant pts with TB

Pts that are not compliant with TB drug therapy can be detained in a hospital for the treatment period and forcefully treated for TB.

Client diagnosed with thrombophlebitis one (1) day ago suddenly complains of chest pain and SOB and the client is visibility anxious. The nurse immediately checks the clients for signs and symptoms of _______.,

Pulmonary embolism

Complications of PE

Pulmonary infarct- death of lung tissue - occurs when 3 factors are present. occlusion o fa lg or med pulmonary vessel, insufficient collateral blood flow from bronchial circulation, preexisting lung disease. Pulmonary hypertenstion

A male client who weighs 175 lb (79.4 kg) is receiving aminophylline (Aminophyllin) (400 mg in 500 ml) at 50 ml/hour. The theophylline level is reported as 6 mcg/ml. The nurse calls the physician who instructs the nurse to change the dosage to 0.45 mg/kg/hour. The nurse should:

Question the order because it's too low.

S/E of Methylzanthines

R/T caffine S/E -nervousness -tremors -increased HR -headache -diaurisis

Airway obstruction can occur in 2 ways:

Reactive airways spasm closed. Hyperreactive airways lead to bronchoconstriction of the smooth muscle. Inflamed airways swell/narrow closed.

5 cardinal signs of inflammation :

Redness, warmth, edema, pain and loss of function. Inflammation causes excess mucous production that the swelling of the lumen - leads to wheezing and rhonchi.

While changing soiled velcro ties for a client with a tracheostomy, the client suddenly coughs, dislodging a tracheostomy tube. Which of the following is the appropriate nursing action? Provide ventilation with a manual resuscitation bag and face mask. Have a coworker call the emergency response team. Reinsert the tracheostomy tube. Cover the tracheostomy opening with a sterile dressing.

Reinsert the tracheostomy tube.

A slightly obese female client with a history of allergy-induced asthma, hypertension, and mitral valve prolapse is admitted to an acute care facility for elective surgery. The nurse obtains a complete history and performs a thorough physical examination, paying special attention to the cardiovascular and respiratory systems. When percussing the client's chest wall, the nurse expects to elicit:

Resonant sounds

Cheyne Stokes

Respirations increase then decrease until they reach a period of apnea. Seen on pts with CHF

Definition of Respiratory Failure

Respiratory system is unable to perform adequate gas exchange as evidenced by PaO2 <60mmHg and PaCO2 >50mmHg. Types: chronic, acute, hypoxemic and hypercapnic

Location of RLL

Right anterior axillary ribs 6-8. Posteriorly T4-T10 on expiration and T4-T12 on inspiration

A nurse is assessing the functioning of a client's chest drainage system. Which of the following are expected client findings? (Select all that apply.) Continuous bubbling in the water seal chamber Gentle constant bubbling in the suction control chamber Rise and fall in the level of water in the water seal chamber with inspiration and expiration Exposed sutures without dressing Drainage system is upright at chest level

Rise and fall in the level of water in the water seal chamber with inspiration and expiration Gentle constant bubbling in the suction control chamber

oxygen therapy: calculating percentage of O2 pt is receiving

Room Air=21% for first liter of O2 add 3% after 1st L add 4%. Can only give up to 6L nasal cannula.

Formoterol (Foradil Aerolizer) Salmeterol (Serevent)

Route: • Inhaled, long-acting Therapeutic uses: • Long-term control of asthma

Albuterol (Proventil, Ventolin)

Route: • Inhaled, short-acting • Oral, long-acting Therapeutic uses: • Prevention of asthma attack (exercise-induced) • Treatment for ongoing asthma attack • Long-term control of asthma

Terbutaline (Brethine)

Route: • Oral, long-acting Therapeutic uses: • Long-term control of asthma

Air embolism S/S

SOB; decrease in SaO2; chest pain; anxiety; air hunger (dyspnea)

The patient with end stage COPD has a nursing diagnosis of impaired gas exchange. Which assessment finding shows that interventions have been effective

SP02 97% on 2L of O2

Intervention is required when?

SaO2 <91%

Tissue damage is not reversible and increases in severity over time:

Scar tissue can lead to thickening of the walls of the airway, permanent changes that cause some degree of obstruction all the time.

Status asthmaticus s/s:

Severe bronchoconstriction: increasing airway obstruction, severe wheezing, extremely labored breathing and use of accessory muscles, hypoxia and respiratory acidosis.

Status asthmaticus: KNOW THIS

Severe, persistent asthma - resists common meds that would be used reverse asthma attacks.

A client is scheduled for a thoracentesis.Into what position should the nurse assist the client for the procedure? Lying flat on the affected side. Prone with the arms raised over the head. Supine with the head of the bed elevated. Sitting while leaning forward over the bedside table.

Sitting while leaning forward over the bedside table.

What is the primary cause of COPD in the US?

Smokers

What drug can help with metabolic acidosis?

Sodium bicarb (won't fix the prob, but will buy time to find the cause)

Presentation of Pneumonia

Spread by aspiration, inhalation and hematogenous spread. 4 characteristic stages: 1) congestion 2) red hepatization 3) gray hepatization 4) resolution. Sudden onset, fever, shaking, chills, SOB, cough productive of purulent sputum (rust colored in Pneumonococcal), chest pain. Confusion in elderly

Glucocortocosteroid

Steroid

Pulmicort

Steroid Prevents the release of substances in the body that cuase inflammation. Used to prvent asthma attacks. Works by decreasing irritation and swelling in the airways which helps to control or prvent asthma s/s.

Flovent (Fluticasone)

Steroid Prevents the release of substances int he body that cause inflammation. Used to prevent asthma attacks.

A nurse is suctioning fluids from a female client through an endotracheal tube. During the suctioning procedure, the nurse notes on the monitor that the heart rate is decreasing. Which if the following is the appropriate nursing intervention?

Stop the procedure and reoxygenate the client

Prednisone when used for 10 days or more can result in:

Suppression of adrenal gland function, such as a decrease in the ability of the adrenal cortex to produce glucocorticoids: Can occur with inhaled agents and oral agents Bone loss (can occur with inhaled agents and oral agents) Hyperglycemia and glucosuria Myopathy as evidenced by muscle weakness Peptic ulcer disease Infection Disturbances of fluid and electrolytes (fluid retention as evidenced by weight gain, and edema and hypokalemia as evidenced by muscle weakness)

What is a common side effect of albuterol that you have to monitor for?

Tachycardia

Findings upon exam of Exacerbation of Asthma

Tachypnea, pursed lipped breathing, prolonged exhalation, tripod position. Palpation: decrease mvmt Percussion: Hyperresonance Auscultation: wheezes, decreased breath sounds omnious sign if no improvement or severely diminished air mvmt. - Recurrent episodes of wheezing, breathlessness, ches tightness and cough (particularly at night or early morning)

A nurse is reviewing a care plan for a client admitted following chest tube placement for a spontaneous pneumothorax. Which of the following interventions should the nurse expect to see? Tape all connections between the chest tube and drainage system. Keep the water-seal drainage system at the level of the right atrium. Notify the provider if there is continuous bubbling in the suction control chamber. Empty the collection bottle and record the amount of drainage every 8 hr.

Tape all connections between the chest tube and drainage system.

A client is receiving oxygen via a nonrebreathing mask. A crucial nursing assessment the nurse performs is to be sure that:

The bag attached to the mask is inflated at all times If the bag attached to a nonrebreathing mask is deflated, the client is at risk for breathing in large amounts of exhaled carbon dioxide. The bag should be maximally inflated at all times

How do you maintain the water seal?

The chamber MUST be kept upright and below the chest tube insertion site. Monitor for evaporation Add fluid prn to maintain the 2cm level.

A nurse is reviewing the discharge teaching plan with a client hospitalized following an acute exacerbation of reactive airway disease. When reminding the client how to prevent acute asthma attacks, what should the nurse plan to discuss first? Triggers that can precipitate an attack and how to eliminate them from the client's environment. The client's perception of the disease process and what may have triggered the current attack. The client's medication regimen, including the proper use of metered-dose inhalers. Manifestations of respiratory infections and the importance of avoiding people who have infections.

The client's perception of the disease process and what may have triggered the current attack.

How do the kidneys keep pH within normal range?

The kidneys remove acid through urine, and cause the body to retain or excrete bicarb (base).

How do the lungs keep the pH within normal range?

The lungs control CO2 (acid) by exhalation. Hypoventilation = retain CO2; hyperventilation - eliminate CO2

A nurse is caring for a client who has a tracheostomy with an inflated cuff in place. Which of the following indicates that the nurse should suction the client's airway secretions? The client is unable to speak. The client's airway secretions were last suctioned 2 hr ago. The client coughs and expectorates a large mucous plug. The nurse auscultates coarse crackles in the lung fields.

The nurse auscultates coarse crackles in the lung fields.

How often should a nurse assess the skin and nares of the patient with a nasal cannula?

The nurse should assess the client's nares and ears for skin breakdown every 6 hours.

Which assessment finding by the nurse when caring for a patient with ARDS who is being treated with mechanical ventilation and high levels of positive end-expiratory pressure (PEEP) indicates that the PEEP may need to be decreased?

The patient has subcutaneous emphysema. rationale: The subcutaneous emphysema indicates barotrauma caused by positive pressure ventilation and PEEP. Bradycardia, hypoxemia, and bronchial breath sounds are all concerns and will need to be addressed, but they are not indications that PEEP should be reduced.

When admitting a patient in possible respiratory failure with a high PaCO2, which assessment information will be of most concern to the nurse?

The patient is somnolent. rationale: Increasing somnolence will decrease the patient's respiratory rate and further increase the PaCO2 and respiratory failure. Rapid action is needed to prevent respiratory arrest. An SpO2 of 90%, weakness, and elevated blood pressure all require ongoing monitoring but are not indicators of possible impending respiratory arrest.

The nurse is caring for a 22-year-old patient who came to the emergency department with acute respiratory distress. Which information about the patient requires the most rapid action by the nurse?

The patient's PaO2 is 45 mm Hg. rationale: The PaO2 indicates severe hypoxemia and respiratory failure. Rapid action is needed to prevent further deterioration of the patient. Although the shallow breathing, rapid respiratory rate, and low PaCO2 also need to be addressed, the most urgent problem is the patient's poor oxygenation.

When prone positioning is used in the care of a patient with acute respiratory distress syndrome (ARDS), which information obtained by the nurse indicates that the positioning is effective?

The patient's PaO2 is 90 mm Hg, and the SaO2 is 92%. rationale: The purpose of prone positioning is to improve the patient's oxygenation as indicated by the PaO2 and SaO2. The other information will be collected but does not indicate whether prone positioning has been effective.

A patient with chronic obstructive pulmonary disease (COPD) arrives in the emergency department complaining of shortness of breath and dyspnea. Which assessment finding by the nurse is most important to report to the health care provider?

The patient's respiratory rate has decreased from 30 to 10 breaths/min. rationale: A decrease in respiratory rate in a patient with respiratory distress suggests the onset of fatigue and a high risk for respiratory arrest. Therefore immediate action such as positive pressure ventilation is needed. Patients who are experiencing respiratory distress frequently sit in the tripod position because it decreases the work of breathing. Crackles in the lung bases may be the baseline for a patient with COPD. An oxygen saturation of 91% is common in patients with COPD and will provide adequate gas exchange and tissue oxygenation.

Periodic infections and exacerbations:

The skin works as the first line of defense. All swelling caused by the inflammation causes gaps to occur in between the cells. On top of the chronic inflammation and swelling they already have, that decreases their airflow, that inflammation and swelling lead to infections that can add to the airflow issue.

What might you observe in a patient with cancer of the larynx?

The tongue and mouth are often white, gray, dark brown, or black and patchy. There may be chronic larangytis.

mechanical ventilation

The use of a ventilator to move room air or oxygen-enriched air into and out of the lungs mechanically if a client is unable to ventilate enough on his or her own to maintain proper levels of oxygen and carbon dioxide in the blood

If there are bubbles in the pleuravac when you ask the patient to cough what could this mean?

There is an air leak

How is an ABG obtained?

Through an arterial line or arterial puncture

Thyroxine(T4), triiodothyronine(T3) and calcitonin are hormones of which gland?

Thyroid

A 84 year-old male has been loosing mobility and gaining weight over the last 2 months. The patient also has the heater running in his house 24 hours a day, even on warm days. Which of the following tests is most likely to be performed?

Thyroid Function Test

Which phrase is used to describe the volume of air inspired and expired with a normal breath?

Tidal volume

Total Lung Capacity (TLC)

Total amount of air lungs contain Normal 6L

Trachael Shifts

Trachea will shift at end of expiration. it will deviate toward a collapsed lung, but for everything else it will deviate toward lung with less pressure.

hyperresonance

Trapping of air in the lungs.

What are the treatments for metabolic acidosis?

Treat the cause, IVP sodium bicarb (won't fix problem, but will buy time to find the cause

Asthma patho:

Trigger: Allergy based asthma - triggered by an allergen; test the patient for allergen based triggers, identify and remove it.

What position is good for COPD?

Tripod

Respiratory Acidosis info

Unable to blow off enough CO2. Patients with COPD, asthma, pneumonia, PACU patients, spinal cord injuries.

What education should be done for the client using rifampin?

Use alternate form of contraception as it reduces the effectiveness of oral contraceptives. it tinges body fluids orange and can stain contacts.

The nurse is teaching a male client with chronic bronchitis about breathing exercises. Which of the following should the nurse include in the teaching?

Use diaphragmatic breathing.

The nurse assesses a male client's respiratory status. Which observation indicates that the client is experiencing difficulty breathing?

Use of accessory muscles

Leukotriene Modifiers

Used to manage allergic rhinitis or allergies as well as prevent asthma.

Decongestant

Used to relieve nasal congestion in the upper resp.tract.

S/S of PE

Varied and nonspecific. classic triad: dyspnea, chest pain and hemoptysis. mahy have moderate hypoxemia with low PaCO2. Hypotension, pallor, severe dyspnea.

Indications for a bronchoscopy

Visualize abnormalities; theraputic reasons (tumors, inflammation, strictures) Biopsy of suspicious tissue (lung cancer) Aspiration of deep sputum or lung abscess for C&S

Functional Residual Capacity (FRC)

Volume of air remaining in lungs after normal exhalation Norm: 2.5 L

A patient's chart indicates a history of ketoacidosis. Which of the following would you not expect to see with this patient if this condition were acute?

Weight Gain

______ describes a violin like sound heard on chest asuculatation

Wheezes

Proper administration of inhaled beta2-agonist and inhaled glucocorticoid

When a client is prescribed an inhaled beta2-agonist and an inhaled glucocorticoid, advise the client to inhale the beta2-agonist before inhaling the glucocorticoid. The beta2-agonist promotes bronchodilation and enhances absorption of the glucocorticoid.

When will you see "tidaling" in a pleuravac?

When it is not connected to suction

Bronchodilators

Widens air passages of the lungs and eases breathing bronchial smooth muscle.

It is ok to have gentle & constant bubbling in the SUCTION control chamber?

Yes

Medication/Food Interactions with Leukotriene Modifiers

Zileuton and zafirlukast inhibit metabolism of warfarin (Coumadin), leading to increased warfarin levels. Zileuton and Zafirlukast inhibit metabolism of theophylline, leading to increased theophylline levels.

Asthma

a chronic inflammatory disorder of the airways. It is an intermittent and reversible airflow obstruction that affects the bronchioles. The obstruction occurs either by inflammation or airway hyper-responsiveness leading to bronchoconstriction.

tuberculosis

a highly communicable disease caused by Mycobacterium tuberculosis; tuberculosis is transmitted by the airborne route via droplet infection

Pulse Oximetry

a noninvasive measurment of the oxygen saturation of the blood, but it IS NOT A REPLACEMENT FOR ABG MEASUREMENT

what is important to have in a patients room after chest tube insertion? why?

a pair of padded clamps just in case the chest tube becomes disconnected

Iatrogenic causes of pulmonary effusion

a pulmonary effusion, or any other medical condition that is caused by a doctor or a procedure. ex: central line placement, pacemake insertion, lung biopsy, thoracentesis, CPR

Mantoux test

a skin test that determines infection with tuberculosis; a small amount (0.1 ml) of intermediate-strength purified protein derivative containing 5 tuberculin units is given intradermally in the forearm; an area of induration measuring 10 mm or more in diameter, 48 to 72 hours after injection, indicates that the individual has been exposed to tuberculosis

suctioning

a sterile procedure that involves the removal of respiratory secretions that accumulate in the trachobronchial airway when the client is unable to expectorate secretions; performed to maintain a patent airway

A client with chronic obstructive pulmonary disease is admitted to an acute care facility because of an acute respiratory infection. When assessing the client's respiratory rate, the nurse notes an abnormal inspiratory-expiratory (I:E) ratio of 1:4. What is a normal I:E ratio? a) 1:2 b) 2:1 c) 1:1 d) 2:2

a) 1:2

For a client with an endotracheal (ET) tube, which nursing action is most essential? a) Auscultating the lungs for bilateral breath sounds b) Turning the client from side to side every 2 hours c) Monitoring serial blood gas values every 4 hours d) Providing frequent oral hygiene

a) Auscultating the lungs for bilateral breath sounds

A nurse is performing a respiratory assessment on a client with pneumonia. She asks the client to say "ninety-nine" several times. Through her stethoscope, she hears the words clearly over his left lower lobe. What term should the nurse use to document this finding? a) Bronchophony b) Tactile fremitus c) Crepitation d) Egophony

a) Bronchophony

A trauma victim in the intensive care unit has a tension pneumothorax. Which signs or symptoms are associated with a tension pneumothorax? a) Decreased cardiac output b) Flattened neck veins c) Tracheal deviation to the affected side d) Hypotension e) Tracheal deviation to the opposite side f) Bradypnea

a) Decreased cardiac output d) Hypotension e) Tracheal deviation to the opposite side

The home health nurse sees a client with end-stage chronic obstructive pulmonary disease. An outcome identified for this client is preventing infection. Which finding indicates that this outcome has been met? a) Decreased oxygen requirements b) Increased sputum production c) Decreased activity tolerance d) Normothermia

a) Decreased oxygen requirements

A nurse is caring for a client who has a tracheostomy and temperature of 103° F (39.4° C). Which intervention will most likely lower the client's arterial blood oxygen saturation? a) Endotracheal suctioning b) Encouragement of coughing c) Use of cooling blanket d) Incentive spirometry

a) Endotracheal suctioning

A client comes to the emergency department complaining of sudden onset of diarrhea, anorexia, malaise, cough, headache, and recurrent chills. Based on the client's history and physical findings, the physician suspects Legionnaires' disease. While awaiting diagnostic test results, the client is admitted to the facility and started on antibiotic therapy. What is the drug of choice for treating Legionnaires' disease? a) Erythromycin (Erythrocin) b) Rifampin (Rifadin) c) Amantadine (Symmetrel) d) Amphotericin B (Fungizone)

a) Erythromycin (Erythrocin)

A home health nurse is visiting a home care client with advanced lung cancer. Upon assessing the client, the nurse discovers wheezing, bradycardia, and a respiratory rate of 10 breaths/min. These signs are associated with which condition? a) Hypoxia b) Delirium c) Hyperventilation d) Semiconsciousness

a) Hypoxia

The nurse is caring for a client with chest trauma. Which nursing diagnosis takes highest priority? a) Impaired gas exchange b) Anxiety c) Decreased cardiac output d) Ineffective cardiopulmonary tissue perfusion

a) Impaired gas exchange

After receiving an oral dose of codeine for an intractable cough, the client asks the nurse, "How long will it take for this drug to work?" How should the nurse respond? a) In 30 minutes b) In 1 hour c) In 2.5 hours d) In 4 hours

a) In 30 minutes

A client with pneumococcal pneumonia is admitted to an acute care facility. The client in the next room is being treated for mycoplasmal pneumonia. Despite the different causes of the various types of pneumonia, all of them share which feature? a) Inflamed lung tissue b) Sudden onset c) Responsiveness to penicillin d) Elevated white blood cell (WBC) count

a) Inflamed lung tissue

The nurse is teaching a client with emphysema how to perform pursed-lip breathing. The client asks the nurse to explain the purpose of this breathing technique. Which explanation should the nurse provide? a) It helps prevent early airway collapse. b) It increases inspiratory muscle strength. c) It decreases use of accessory breathing muscles. d) It prolongs the inspiratory phase of respiration.

a) It helps prevent early airway collapse.

A client suffers adult respiratory distress syndrome as a consequence of shock. The client's condition deteriorates rapidly, and endotracheal (ET) intubation and mechanical ventilation are initiated. When the high-pressure alarm on the mechanical ventilator sounds, the nurse starts to check for the cause. Which condition triggers the high-pressure alarm? a) Kinking of the ventilator tubing b) A disconnected ventilator tube c) An ET cuff leak d) A change in the oxygen concentration without resetting the oxygen level alarm

a) Kinking of the ventilator tubing

A client with chronic obstructive pulmonary disease presents with respiratory acidosis and hypoxemia. He tells the nurse that he doesn't want to be placed on a ventilator. What action should the nurse take? a) Notify the physician immediately so he can determine client competency. b) Have the client sign a do-not-resuscitate (DNR) form. c) Determine whether the client's family was consulted about his decision. d) Consult the palliative care group to direct care for the client.

a) Notify the physician immediately so he can determine client competency.

A 33-year-old woman with primary pulmonary hypertension is being evaluated for a heart-lung transplant. The nurse asks her what treatments she is currently receiving for her disease. She is likely to mention which treatments? a) Oxygen b) Aminoglycosides c) Diuretics d) Vasodilators e) Antihistamines f) Sulfonamides

a) Oxygen c) Diuretics d) Vasodilators

A client has hypoxemia of pulmonary origin. What portion of arterial blood gas results is most useful in distinguishing between acute respiratory distress syndrome and acute respiratory failure? a) Partial pressure of arterial oxygen (PaO2) b) Partial pressure of arterial carbon dioxide (PaCO2) c) pH d) Bicarbonate (HCO3-)

a) Partial pressure of arterial oxygen (PaO2)

A client is admitted to the emergency department with an acute asthma attack. The physician prescribes ephedrine sulfate, 25 mg subcutaneously (S.C.). How soon should the ephedrine take effect? a) Rapidly b) In 3 minutes c) In 1 hour d) In 2 hours

a) Rapidly

The nurse administers albuterol (Proventil), as prescribed, to a client with emphysema. Which finding indicates that the drug is producing a therapeutic effect? a) Respiratory rate of 22 breaths/minute b) Dilated and reactive pupils c) Urine output of 40 ml/hour d) Heart rate of 100 beats/minute

a) Respiratory rate of 22 breaths/minute

A client with respiratory acidosis is admitted to the intensive care unit for close observation. The nurse should stay alert for which complication associated with respiratory acidosis? a) Shock b) Stroke c) Seizures d) Hyperglycemia

a) Shock

A client has been hospitalized for treatment of acute bacterial pneumonia. Which outcome indicates an improvement in the client's condition? a) The client has a partial pressure of arterial oxygen (PaO2) value of 90 mm Hg or higher. b) The client has a partial pressure of arterial carbon dioxide (PaCO2) value of 65 mm Hg or higher. c) The client exhibits restlessness and confusion. d) The client exhibits bronchial breath sounds over the affected area.

a) The client has a partial pressure of arterial oxygen (PaO2) value of 90 mm Hg or higher.

A client with chronic obstructive pulmonary disease (COPD) is admitted to the medical-surgical unit. To help this client maintain a patent airway and achieve maximal gas exchange, the nurse should: a) instruct the client to drink 2 L of fluid daily. b) maintain the client on bed rest. c) administer anxiolytics, as prescribed, to control anxiety. d) administer pain medication as prescribed.

a) instruct the client to drink 2 L of fluid daily.

A slightly obese client with a history of allergy-induced asthma, hypertension, and mitral valve prolapse is admitted to an acute care facility for elective surgery. The nurse obtains a complete history and performs a thorough physical examination, paying special attention to the cardiovascular and respiratory systems. When percussing the client's chest wall, the nurse expects to elicit: a) resonant sounds. b) hyperresonant sounds. c) dull sounds. d) flat sounds.

a) resonant sounds.

a 48-year-old client doesn't smoke cigarettes yet is demonstrating signs of lung irritation. Which of the following questions could help with the assessment of this client? a. Do you smoke or inhale marijuana or other herbal products? b. Have you had allergy testing? c. Have you received a flu or pneumonia vaccination? d. Have you tried to stop smoking?

a. Do you smoke or inhale marijuana or other herbal products?

During a physical assessment, the nurse documents eupnea on the client's medical record. What does this finding suggest? a. Normal respirations b. Slow respirations c. Irregular respirations d. Rapid respirations

a. Normal respirations

After inspecting a client's thorax, the nurse writes "AP:T 1:2, bilateral symmetrical movements, sternum midline, respiratory rate 16 and regular." What do these findings suggest? a. Nothing. These findings are normal. b. The client has pneumonia. c. The client has a respiratory illness. d. The client has allergies.

a. Nothing. These findings are normal.

23. The accumulation of fluids in the pleural space is called: a. Pleural effusion b. Hemothorax c. Hydrothorax d. Pyothorax

a. Pleural effusion

The accumulation of fluids in the pleural space is called: a. Pleural effusion b. Hemothorax c. Hydrothorax d. Pyothorax

a. Pleural effusion

Prior to listening to a client's lung sounds, the nurse palpates the sternum and feels a horizontal bump on the bone. What does this finding suggest to the nurse? a. This is the angle of Louis. b. The manubrium is damaged. c. The costal angle is greater than normal. d. The xiphoid process is misshaped.

a. This is the angle of Louis.

A 57-year-old client tells the nurse, "I need two to three pillows to sleep." How should this information be documented? a. Two to three pillow orthopnea b. Dyspnea on excursion c. Resting apnea d. Dyspnea at rest

a. Two to three pillow orthopnea

pleural friction rub (Pleuritic pain)

abnormal breath sound that is creaky and grating in nature and is heard on inspiration and expiration

sibilant wheeze

abnormal breath sound that is high pitched and musical in nature and is heard on inhalation and exhalation

sonorous wheeze

abnormal breath sound that is low pitched and snoring in nature and is louder on expiration

Abnormal bronchial breath sounds indicate what?

abnormal sound transmission bc of consolidation of lung tissue

adventitious breath sound

abnormal sound, including sibilant wheezes (formerly wheezes), sonorous wheezes (formerly rhonchi), fine and course crackles (formerly rales), pleural friction rubs, and stridor

Conditions that cause a shift to the right in the OH curve

acidosis and HYPERthermia and when PaCO2 is increased.

After a bronchoscopy, the client is not d/c from recovery room until _______

adequate cough reflex and respiratory effort are present.

Sympathomemitics

adernergic receptors - alpha 1- epinephrine: used for mucous production in asthma and stridor and nasal congestion. - Beta 1- atenolol: no use in pulmonary, increase HR and heart contractions - Beta 2- bronchiodilation: short acting- albuterol, xopenex. long acting- salmeterol ** Works on Peripheral nervous system

the older adult is at increased risk for developing orthosatic hypotension, which could precipitate a fall is often due to,

age, immobility, chronic illness, and medication

Open Pneumothorax

air enters through an opening in the chest wall. Stab or gun shot.

What is crepitus?

air leakage in SQ tissue

Thickened bronchial walls which obstructs:

airflow and gas exchange. They are CO2 retainers. Bringing o2 in and getting CO2 out. Impeded gas exchange ends up causing resp acidosis, because of these high levels of co2 on chronic level.

Disease of the:

airway not alveoli.

Medication for acute asthma attack

albuterol

pH levels above 7.45 reflect ______.

alkalosis

Conditions that cause a shift to the left in the OH curve

alkalosis, HYPOthermia, and decrease of PaCO2 in arterial blood

Ideally, the coughing make the irritant,

along with the mucous, get coughed out of the body, getting rid of the foreign body from getting into the lungs.

Nurse Oliver observes constant bubbling in the water-seal chamber of a closed chest drainage system. What should the nurse conclude?

an air leak.

Slow

and progressive obstruction of airways.

Location of LUL

anterior= Ribs 2-4 Posterior- T1-T4

Location of RML

anteriorly from ribs 4-6. ONLY found anteriorly

Treatment of PID

antibiotics, no intercourse for 3 weeks, partner needs checked and recheck necessary in 48-72 hours even if imporving.

Drug Thearpy for PE

anticoagulants for 3-6 months.

Rule of 2 with Asthma

any yes means asthma is out of control take an inhaler more thatn 2x week nightly symptoms more than 2x month refill quick relief inhaler more than 2x year

Where are bronchial breath sounds considered abnormal?

anywhere over posterior/lateral chest

ABGs can be obtained by:

arterial puncture or through arterial line

Greatest risk for client after bronchoscopy

aspiration due to depressed gag reflex

in which patients do you hear high-pitched wheezes?

asthma or emphysema (narrowed-airway diseases)

What is montelukast (singulair) and when should it be taken?

asthma prophylaxis, chronic asthma should be taken every evening

How to prevent complications on pt with chest tube

avoid dependent loops never raise drainage system above pts chest assess pt and integrity every hr.

A home care nurse visits a client with chronic obstructive pulmonary disease who requires oxygen. Which statement by the client indicates the need for additional teaching about home oxygen use? a) "I lubricate my lips and nose with K-Y jelly." b) "I make sure my oxygen mask is on tightly so it won't fall off while I nap." c) "I have a 'no smoking' sign posted at my front entry-way to remind guests not to smoke." d) "I clean my mask with water after every meal."

b) "I make sure my oxygen mask is on tightly so it won't fall off while I nap."

For a client with an acute pulmonary embolism, the physician prescribes heparin (Liquaemin), 25,000 U in 500 ml of dextrose 5% in water (D5W) at 1,100 U/hour. The nurse should administer how many milliliters per hour? a) 8 b) 22 c) 30 d) 50

b) 22

On auscultation, which finding suggests a right pneumothorax? a) Bilateral inspiratory and expiratory crackles b) Absence of breath sounds in the right thorax c) Inspiratory wheezes in the right thorax d) Bilateral pleural friction rub

b) Absence of breath sounds in the right thorax

A client with a pulmonary embolus has the following arterial blood gas (ABG) values: pH, 7.49; partial pressure of arterial oxygen (PaO2), 60 mm Hg; partial pressure of arterial carbon dioxide (PaCO2), 30 mm Hg; bicarbonate (HCO3-) 25 mEq/L. What should the nurse do first? a) Instruct the client to breathe into a paper bag. b) Administer oxygen by nasal cannula as prescribed. c) Auscultate breath sounds bilaterally every 4 hours. d) Encourage the client to deep-breathe and cough every 2 hours.

b) Administer oxygen by nasal cannula as prescribed.

The physician prescribes triamcinolone (Azmacort) and salmeterol (Serevent) for a client with a history of asthma. What action should the nurse take when administering these drugs? a) Administer the triamcinolone and then administer the salmeterol. b) Administer the salmeterol and then administer the triamcinolone. c) Allow the client to choose the order in which the drugs are administered. d) Monitor the client's theophylline level before administering the medications.

b) Administer the salmeterol and then administer the triamcinolone.

A client has a sucking stab wound to the chest. Which action should the nurse take first? a) Draw blood for a hematocrit and hemoglobin level. b) Apply a dressing over the wound and tape it on three sides. c) Prepare a chest tube insertion tray. d) Prepare to start an I.V. line.

b) Apply a dressing over the wound and tape it on three sides.

A client, confused and short of breath, is brought to the emergency department by a family member. The medical history reveals chronic bronchitis and hypertension. To learn more about the client's current respiratory problem, the physician orders a chest X-ray and arterial blood gas (ABG) analysis. When reviewing the ABG report, the nurse sees many abbreviations. What does a lowercase "a" in an ABG value represent? a) Acid-base balance b) Arterial blood c) Arterial oxygen saturation d) Alveoli

b) Arterial blood

An adult client with cystic fibrosis is admitted to an acute care facility with an acute respiratory infection. Prescribed respiratory treatment includes chest physiotherapy. When should the nurse perform this procedure? a) Immediately before a meal b) At least 2 hours after a meal c) When bronchospasms occur d) When secretions have mobilized

b) At least 2 hours after a meal

Before weaning a client from a ventilator, which assessment parameter is most important for the nurse to review? a) Fluid intake for the past 24 hours b) Baseline arterial blood gas (ABG) levels c) Prior outcomes of weaning d) Electrocardiogram (ECG) results

b) Baseline arterial blood gas (ABG) levels

During inspiration, which of the following occurs? a) Lungs recoil. b) Diaphragm descends. c) Alveolar pressure is positive. d) Inspiratory muscles relax.

b) Diaphragm descends.

After receiving the wrong medication, the client's breathing stops. The nurse initiates the code protocol, and the client is emergently intubated. As soon as the client's condition stabilizes, the nurse completes an incident report. What should the nurse do next? a) Place the incident report on the client's chart. b) Document the incident in the nurses' notes. c) Document in the nurses' notes that an incident report was completed. d) Make a copy of the incident report for the client.

b) Document the incident in the nurses' notes.

A client with chronic obstructive pulmonary disease (COPD) is being evaluated for a lung transplant. The nurse performs the initial physical assessment. Which signs and symptoms should the nurse expect to find? a) Decreased respiratory rate b) Dyspnea on exertion c) Barrel chest d) Shortened expiratory phase e) Clubbed fingers and toes f) Fever

b) Dyspnea on exertion c) Barrel chest e) Clubbed fingers and toes

The nurse is planning care for a client after a tracheostomy. One of the client's goals is to overcome verbal communication impairment. Which of the following interventions should the nurse include in the care plan? a) Make an effort to read the client's lips to foster communication. b) Encourage the client's communication attempts by allowing him time to select or write words. c) Answer questions for the client to reduce his frustration. d) Avoid using a tracheostomy plug because it blocks the airway.

b) Encourage the client's communication attempts by allowing him time to select or write words.

A nurse is completing her annual cardiopulmonary resuscitation training. The class instructor tells her that a client has fallen off a ladder and is lying on his back; he is unconscious and isn't breathing. What maneuver should the nurse use to open his airway? a) Head tilt-chin lift b) Jaw-thrust c) Heimlich d) Seldinger

b) Jaw-thrust

A client abruptly sits up in bed, reports having difficulty breathing and has an arterial oxygen saturation of 88%. Which mode of oxygen delivery would most likely reverse the manifestations? a) Simple mask b) Nonrebreather mask c) Face tent d) Nasal cannula

b) Nonrebreather mask

An elderly client with influenza is admitted to an acute care facility. The nurse monitors the client closely for complications. What is the most common complication of influenza? a) Septicemia b) Pneumonia c) Meningitis d) Pulmonary edema

b) Pneumonia

A client with myasthenia gravis is receiving continuous mechanical ventilation. When the high-pressure alarm on the ventilator sounds, what should the nurse do? a) Check for an apical pulse. b) Suction the client's artificial airway. c) Increase the oxygen percentage. d) Ventilate the client with a handheld mechanical ventilator.

b) Suction the client's artificial airway.

A client with end-stage chronic obstructive pulmonary disease requires bi-level positive airway pressure (BiPAP). While caring for the client, the nurse determines that bilateral wrist restraints are required to prevent compromised care. Which client care outcome is associated with restraint use in the client who requires BiPAP? a) The client will remain infection-free. b) The client will maintain adequate oxygenation. c) The client will maintain adequate urine output. d) The client will remain pain-free.

b) The client will maintain adequate oxygenation.

A client with acute bronchitis is admitted to the health care facility and is receiving supplemental oxygen via nasal cannula. When monitoring this client, the nurse suddenly hears a high-pitched whistling sound. What is the most likely cause of this sound? a) The water level in the humidifier reservoir is too low. b) The oxygen tubing is pinched. c) The client has a nasal obstruction. d) The oxygen concentration is above 44%.

b) The oxygen tubing is pinched.

A client with a pneumothorax receives a chest tube attached to a Pleur-evac. The nurse notices that the fluid of the second chamber of the Pleur-evac isn't bubbling. Which nursing assumption would be most invalid? a) The tubing from the client to the chamber is blocked. b) There is a leak somewhere in the tubing system. c) The client's affected lung has reexpanded. d) The tubing needs to be cleared of fluid.

b) There is a leak somewhere in the tubing system.

A nurse detects bilateral crackles when auscultating a client's lungs. Which statement about crackles is true? a) They're usually heard on expiration and may clear with a cough. b) They're usually heard on inspiration and sometimes clear with a cough. c) They're hissing or musical and are usually heard on inspiration and expiration; if severe, they may be heard without a stethoscope. d) They're creaking and grating and are usually heard over the problem area on both inspiration and expiration.

b) They're usually heard on inspiration and sometimes clear with a cough.

Prednisone (Deltasone) is prescribed to control inflammation in a client with interstitial lung disease. During client teaching, the nurse stresses the importance of taking prednisone exactly as prescribed and cautions against discontinuing the drug abruptly. A client who discontinues prednisone abruptly may experience: a) hyperglycemia and glycosuria. b) acute adrenocortical insufficiency. c) GI bleeding. d) restlessness and seizures.

b) acute adrenocortical insufficiency.

A 47-year-old male client with unresolved hemothorax is febrile, with chills and sweating. He has a nonproductive cough and chest pain. His chest tube drainage is turbid. A possible explanation for these findings is: a) lobar pneumonia. b) empyema. c) Pneumocystis carinii pneumonia. d) infected chest tube wound site.

b) empyema.

The nurse is caring for a client who recently underwent a tracheostomy. The first priority when caring for a client with a tracheostomy is: a) helping him communicate. b) keeping his airway patent. c) encouraging him to perform activities of daily living. d) preventing him from developing an infection.

b) keeping his airway patent.

The nurse is caring for a client experiencing an acute asthma attack. The client stops wheezing and breath sounds aren't audible. This change occurred because: a) the attack is over. b) the airways are so swollen that no air can get through. c) the swelling has decreased. d) crackles have replaced wheezes.

b) the airways are so swollen that no air can get through.

A client recovering from a pulmonary embolism is receiving warfarin (Coumadin). To counteract a warfarin overdose, the nurse would administer: a) heparin. b) vitamin K1 (phytonadione). c) vitamin C. d) protamine sulfate.

b) vitamin K1 (phytonadione).

The nurse is planning to assess the apex of a client's lungs. Which area of the body will the nurse be assessing? a. Left of the sternum, third intercostal space b. Above the clavicles c. Below the scapula d. Right of the sternum, sixth intercostal space

b. Above the clavicles The apex of each lung is slightly superior to the inner third of the clavicle.

While changing the tapes on a tracheostomy tube, the male client coughs and the tube is dislodged. The initial nursing action is to: a. Call the physician to reinsert the tube. b. Grasp the retention sutures to spread the opening. c. Call the respiratory therapy department to reinsert the tracheotomy. d. Cover the tracheostomy site with a sterile dressing to prevent infection.

b. Grasp the retention sutures to spread the opening.

While changing the tapes on a tracheostomy tube, the male client coughs and the tube is dislodged. The initial nursing action is to: a. Call the physician to reinsert the tube. b. Grasp the retention sutures to spread the opening. c. Call the respiratory therapy department to reinsert the tracheotomy. d. Cover the tracheostomy site with a sterile dressing to prevent infection.

b. Grasp the retention sutures to spread the opening. Rationale: We want to prevent airway collapse, the others will take too long or will block the airway.

The mother of a four-year-old child tells the nurse, "I think there's something wrong with him; his chest is round like a ball." Which of the following would be an appropriate response for the nurse to make to the mother? a. I see what you mean. That seems odd. b. The chest of a child appears round and is normal. c. I wouldn't worry about that. d. Did you tell the doctor about this?

b. The chest of a child appears round and is normal.

what is the increase in ap diameter called

barrel chest

Both can occur together,

because inflammation itself can actually trigger bronchoconstriction.

When should respiratory treatments be scheduled with patient with COPD

before meals

What blood pH levels reflect acidosis? alkalosis?

below 7.35 reflect acidosis, above 7.45 reflect alkalosis

The kidney chemical(s) are/is?

bicarb & hydrogen

What are the problem chemical(s) with metabolic acidosis or alkalosis?

bicarb & hydrogen

What are the greatest most immediate post operative risks for a patient who has undergone a larengectomy?

bleeding or occlusion of the larengectomy tube

Anticholinergics

block acetylcholine, work on CNS ex) atrovent and spireva spireva- can not be given with narrowed glaucoma b/c increased intraoccular pressure

perfusion

blood flow through an organ or body part

Where is a pleural friction rub heard: on expiration/ inspiration?

both

Medication management usually addresses . . .

both inflammation and bronchoconstriction. These same medications may be used in symptomatic treatment of chronic obstructive pulmonary disease (COPD).

A vasovagal response may occur, causing _________?.

bradycardia

What are the treatments for respiratory alkalosis?

breathe into a paper bag encourage the pt to decrease respiratory rate, treat the cause and monitor ABGs

Cells lung cancer develp from

bronchial epithelial cells (bronchogenic)

How do you know if there is an airleak in the chest tube?

bubbling in the water seal chamber

If controlled,

by removing the irritants/allergen asthma is reversible.

Some people can manage the symptoms:

by using allergy meds, but others who are more sensitive cant. Some people opt to get allergy shots, but the issue is that it can trigger asthma if the person is more sensitive. Genetic or non allergic triggers.

A client admitted to the facility for treatment for tuberculosis receives instructions about the disease. Which statement made by the client indicates the need for further instruction? a) "I will have to take the medication for up to a year." b) "This disease may come back later if I am under stress." c) "I will stay in isolation for at least 6 weeks." d) "I will always have a positive test for tuberculosis."

c) "I will stay in isolation for at least 6 weeks."

A client with pneumonia develops respiratory failure and has a partial pressure of arterial oxygen of 55 mm Hg. He's placed on mechanical ventilation with a fraction of inspired oxygen (FIO2) of 0.9. The nursing goal should be to reduce the FIO2 to no greater than: a) 0.21. b) 0.35. c) 0.5 d) 0.7

c) 0.5

Which task can be safely delegated to a licensed practical nurse (LPN)? a) Teaching a newly diagnosed diabetic about insulin administration. b) Admitting a client who underwent a thoracotomy to the nursing unit from the postanesthesia care unit. c) Changing the dressing of a client who underwent surgery two days ago. d) Administering an I.V. bolus of morphine sulfate to a client experiencing incisional pain

c) Changing the dressing of a client who underwent surgery two days ago.

A client has undergone a left hemicolectomy for bowel cancer. Which activities prevent the occurrence of postoperative pneumonia in this client? a) Administering oxygen, coughing, breathing deeply, and maintaining bed rest b) Coughing, breathing deeply, maintaining bed rest, and using an incentive spirometer c) Coughing, breathing deeply, frequent repositioning, and using an incentive spirometer d) Administering pain medications, frequent repositioning, and limiting fluid intake

c) Coughing, breathing deeply, frequent repositioning, and using an incentive spirometer

A recent immigrant from Vietnam is diagnosed with pulmonary tuberculosis (TB). Which intervention is most important for the nurse to implement with this client? a) Client teaching about the cause of TB b) Reviewing the risk factors for TB c) Developing a list of people with whom the client has had contact d) Client teaching about the importance of TB testing

c) Developing a list of people with whom the client has had contact

The physician orders a palliative care consult for a client with end-stage chronic obstructive pulmonary disease who wishes no further medical intervention. Which step should the nurse anticipate based on her knowledge of palliative care? a) Decreasing administration of pain medications b) Reducing oxygen requirements c) Increasing the need for antianxiety agents d) Decreasing the use of bronchodilators

c) Increasing the need for antianxiety agents

A client who sustained a pulmonary contusion in a motor vehicle accident develops a pulmonary embolism. Which nursing diagnosis takes priority with this client? a) Excess fluid volume related to excess sodium intake b) Acute pain related to tissue trauma c) Ineffective breathing pattern related to tissue trauma d) Activity intolerance related to insufficient energy to carry out activities of daily living

c) Ineffective breathing pattern related to tissue trauma

A client is prescribed rifampin (Rifadin), 600 mg P.O. daily. Which statement about rifampin is true? a) It's usually given alone. b) Its exact mechanism of action is unknown. c) It's tuberculocidal, destroying the offending bacteria. d) It acts primarily against resting bacteria.

c) It's tuberculocidal, destroying the offending bacteria.

A client in the emergency department is diagnosed with a communicable disease. When complications of the disease are discovered, the client is admitted to the hospital and placed in respiratory isolation. Which infection warrants respiratory isolation? a) Chickenpox b) Impetigo c) Measles d) Cholera

c) Measles

A client is receiving conscious sedation while undergoing bronchoscopy. Which assessment finding should receive the nurse's immediate attention? a) Absent cough and gag reflexes b) Blood-tinged secretions c) Oxygen saturation of 90% d) Respiratory rate of 13 breaths/min

c) Oxygen saturation of 90%

A client is receiving supplemental oxygen. When determining the effectiveness of oxygen therapy, which arterial blood gas value is most important? a) pH b) Bicarbonate (HCO3-) c) Partial pressure of arterial oxygen (PaO2) d) Partial pressure of arterial carbon dioxide (PaCO2)

c) Partial pressure of arterial oxygen (PaO2)

The nurse is developing a teaching plan for a client with asthma. Which teaching point has the highest priority? a) Avoid contact with fur-bearing animals. b) Change filters on heating and air conditioning units frequently. c) Take prescribed medications as scheduled. d) Avoid goose down pillows.

c) Take prescribed medications as scheduled.

A client with a history of type 1 diabetes is admitted to the hospital with community-acquired pneumonia. The client's blood glucose level in the emergency care unit was 576 mg/dl. The physician prescribes an I.V. containing normal saline solution, an insulin infusion, and I.V. levofloxacin (Levaquin). The nurse piggybacks the insulin infusion into the normal saline solution. She questions whether she can also piggyback the levofloxacin into the same I.V. line. Which health team member should she collaborate with to check the compatibility of these solutions? a) The physician who prescribed the medications b) The coworker with 20 years nursing experience c) The pharmacist covering the floor d) The infectious disease nurse

c) The pharmacist covering the floor

The nurse observes constant bubbling in the water-seal chamber of a closed chest drainage system. What should the nurse conclude? a) The system is functioning normally. b) The client has a pneumothorax. c) The system has an air leak. d) The chest tube is obstructed.

c) The system has an air leak.

A client in acute respiratory distress is brought to the emergency department. After endotracheal (ET) intubation and initiation of mechanical ventilation, the client is transferred to the intensive care unit. Before suctioning the ET tube, the nurse hyperventilates and hyperoxygenates the client. What is the rationale for these interventions? a) They help prevent subcutaneous emphysema. b) They help prevent pneumothorax. c) They help prevent cardiac arrhythmias. d) They help prevent pulmonary edema.

c) They help prevent cardiac arrhythmias.

A client with a suspected pulmonary disorder undergoes pulmonary function tests. To interpret test results accurately, the nurse must be familiar with the terminology used to describe pulmonary functions. Which term refers to the volume of air inhaled or exhaled during each respiratory cycle? a) Vital capacity b) Functional residual capacity c) Tidal volume d) Maximal voluntary ventilation

c) Tidal volume

The nurse is teaching a client with chronic bronchitis about breathing exercises. Which instruction should the nurse include in the teaching? a) Make inhalation longer than exhalation. b) Exhale through an open mouth. c) Use diaphragmatic breathing. d) Use chest breathing.

c) Use diaphragmatic breathing.

A client with advanced chronic obstructive pulmonary disease (COPD), which nursing action best promotes adequate gas exchange? a) Encouraging the client to drink three glasses of fluid daily b) Keeping the client in semi-Fowler's position c) Using a high-flow Venturi mask to deliver oxygen as prescribed d) Administering a sedative as prescribed

c) Using a high-flow Venturi mask to deliver oxygen as prescribed

A client admitted with multiple traumatic injuries receives massive fluid resuscitation. Later, the physician suspects that the client has aspirated stomach contents. The nurse knows that this client is at highest risk for: a) chronic obstructive pulmonary disease (COPD). b) bronchial asthma. c) adult respiratory distress syndrome (ARDS). d) renal failure.

c) adult respiratory distress syndrome (ARDS).

A client with chronic obstructive pulmonary disease (COPD) is recovering from a myocardial infarction. Because the client is extremely weak and can't produce an effective cough, the nurse should monitor closely for: a) pleural effusion. b) pulmonary edema. c) atelectasis. d) oxygen toxicity.

c) atelectasis.

After undergoing a left thoracotomy, a client has a chest tube in place. When caring for this client, the nurse must: a) report fluctuations in the water-seal chamber. b) clamp the chest tube once every shift. c) encourage coughing and deep breathing. d) milk the chest tube every 2 hours.

c) encourage coughing and deep breathing.

A client with chronic sinusitis comes to the outpatient department complaining of headache, malaise, and a nonproductive cough. When examining the client's paranasal sinuses, the nurse detects tenderness. To evaluate this finding further, the nurse should transilluminate the: a) frontal sinuses only. b) sphenoidal sinuses only. c) frontal and maxillary sinuses. d) sphenoidal and ethmoidal sinuses.

c) frontal and maxillary sinuses.

Inspiratory and expiratory stridor may be heard in a client who: a) is experiencing an exacerbation of goiter. b) is experiencing an acute asthmatic attack. c) has aspirated a piece of meat. d) has severe laryngotracheitis.

c) has aspirated a piece of meat.

A client's chest X-ray reveals bilateral white-outs, indicating adult respiratory distress syndrome (ARDS). This syndrome results from: a) cardiogenic pulmonary edema. b) respiratory alkalosis. c) increased pulmonary capillary permeability. d) renal failure.

c) increased pulmonary capillary permeability.

A client with advanced acquired immunodeficiency syndrome (AIDS) is diagnosed with active tuberculosis. Which of the following regimens would the nurse expect the physician to prescribe? a) isoniazid (Laniazid) and rifampin (Rifadin) b) ethambutol (Myambutol), pyrazinamide, and isoniazid c) isoniazid, rifampin, ethambutol, and pyrazinamide d) ethambutol, ciprofloxacin (Cipro), pyrazinamide, and streptomycin

c) isoniazid, rifampin, ethambutol, and pyrazinamide

A 21-year-old client with cystic fibrosis develops pneumonia. To decrease the viscosity of respiratory secretions, the physician prescribes acetylcysteine (Mucomyst). Before administering the first dose, the nurse checks the client's history for asthma. Acetylcysteine must be used cautiously in a client with asthma because: a) it's a respiratory depressant. b) it's a respiratory stimulant. c) it may induce bronchospasm. d) it inhibits the cough reflex.

c) it may induce bronchospasm.

A client with severe acute respiratory syndrome (SARS) privately informs the nurse that he doesn't want to be placed on a ventilator if his condition worsens. The client's wife and children have repeatedly expressed their desire that everything be done for the client. The most appropriate action by the nurse would be to: a) inform the family of the client's wishes. b) assure the family that everything possible will be done. c) support the client's decision. d) assure the client that everything possible will be done.

c) support the client's decision.

The nurse is caring for a client who has a tracheostomy tube and is undergoing mechanical ventilation. The nurse can help prevent tracheal dilation, a complication of tracheostomy tube placement, by: a) suctioning the tracheostomy tube frequently. b) using a cuffed tracheostomy tube. c) using the minimal air leak technique with cuff pressure less than 25 cm H2O. d) keeping the tracheostomy tube plugged.

c) using the minimal air leak technique with cuff pressure less than 25 cm H2O.

The client tells the nurse he sometimes coughs up "thick yellow mucous." What does this information suggest to the nurse? a. He might have an allergy. b. He might have a fungal infection. c. He might have episodic lung infections. d. He might have tuberculosis.

c. He might have episodic lung infections Rationale: The color and odor of any mucus is associated with specific diseases or problems. Green or yellow mucus often signals a lung infection.

A seven-month-pregnant female is sitting quietly in the waiting room, and her respiratory rate is 20 and shallow. What does this finding suggest to the nurse? a. She has a history of smoking. b. She is using accessory muscles to breathe. b. She is in pending respiratory failure. c. Nothing. This is normal.

c. Nothing. This is normal.

While palpating the posterior thorax of a client, the nurse notes increased fremitus. What does this finding suggest to the nurse? a. The client needs to speak up. b. The client has a thick chest wall. c. The client could either have fluid in the lungs or have an infection. d. Nothing. This is a normal finding.

c. The client could either have fluid in the lungs or have an infection.

S/S of PID

can be silent lower ab pain worsens with movement of any type. spotting after intercourse purulent cervical or vaginal discharge. fever and chills increased cramping during menses, irregular bleeding and some pain with intercourse. Usually caused by Gohorrhoeae and chlamydia.

Values obtained by pulse oximetery are unreliable in:

cardiac arrest, shock, and other states of low perfusions

Shift to left in oxygen-hemoglobin dissociation curve

caused by: - increase in pH - decreased temperature - decreased PCO2 blood picks up oxygen more readily at the lungs but delivers oxygen less readily to the tissues

Shift to the right in oxygen-hemoglobin dissociation curve

caused by: -Decrease in pH - Increase in Temp - Increase in PCO2 Blood picks up oxygen less rapidly in the lungs but delivers oxygen more readily to the tissues.

COPD is:

characterized by bronchospasm and dyspnea.

The common clinical manifestations of pulmonary embolism are tachypnea, tachycardia, dyspnea, and _________?

chest pain

Asthma not well controlled,

chronic = excessive inflammation = tissue damage.

COPD consists of 2 CAL disorders:

chronic bronchitis and pulmonary emphysema. COPD is chronic and irreversible.

preoperative procedures for vaginal surgery

cleansing douche, cleansing enema

Normal thoracentesis tap

clear and straw colored fluid

what will you observe on the hands of the client with emphysema

clubbing of the fingernail beds

atelectasis

collapse of a lung or a portion of a lung

What is atelectasis?

collapse of the alveoli in the lung prevents normal exchange of O2 and co2 hypoventilation occurs

Pneumothorax

collapsed lung

pleural effusion

collection of fluid within the pleural cavity

What can happen to the pt if respiratory acidosis is left untreated?

coma

The client with a pneumothorax experiences hypoxia due to:

compression of the lung tissue underlying the pneumothorax

pleurisy

condition arising from inflammation of the pleura, or sac, that encases the lung

asthma

condition characterized by intermittent airway obstruction due to antigen-antibody reaction

pneumothorax

condition wherein air or gas accumulates in the pleural space of the lungs, causing the lungs to collapse

hemothorax

condition wherein blood accumulates in the pleural space of the lungs

What are some causes of respiratory failure in children?

congenital heart disease, sepsis or infection, respiratory distress syndrome, aspiration, fluid overload, dehydration.

Which lung sound occurs with the sudden opening of small airways that contain fluid?

crackles

the client with emphysema is

cyanotic

A nurse caring for a client with deep vein thrombosis must be especially alert for complications such as pulmonary embolism. Which findings suggest pulmonary embolism? a) Nonproductive cough and abdominal pain b) Hypertension and lack of fever c) Bradypnea and bradycardia d) Chest pain and dyspnea

d) Chest pain and dyspnea

The physician determines that a client has been exposed to someone with tuberculosis. The nurse expects the physician to order which of the following? a) Daily oral doses of isoniazid (Nydrazid) and rifampin (Rifadin) for 6 months to 2 years b) Isolation until 24 hours after antitubercular therapy begins c) Nothing, until signs of active disease arise d) Daily doses of isoniazid, 300 mg for 6 months to 1 year

d) Daily doses of isoniazid, 300 mg for 6 months to 1 year

The nurse is assessing a client who comes to the clinic for care. Which findings in this client suggest bacterial pneumonia? a) Nonproductive cough and normal temperature b) Sore throat and abdominal pain c) Hemoptysis and dysuria d) Dyspnea and wheezing

d) Dyspnea and wheezing

The nurse is caring for a client with pneumonia. As part of prescribed therapy, the client must use a bedside incentive spirometer to promote maximal deep breathing. The nurse checks to make sure the client is using the spirometer properly. During each waking hour, the client should perform a minimum of how many sustained, voluntary inflation maneuvers? a) One to two b) Three to four c) Five to seven d) Eight to ten

d) Eight to ten

When caring for a client with acute respiratory failure, the nurse should expect to focus on resolving which set of problems? a) Hypotension, hyperoxemia, and hypercapnia b) Hyperventilation, hypertension, and hypocapnia c) Hyperoxemia, hypocapnia, and hyperventilation d) Hypercapnia, hypoventilation, and hypoxemia

d) Hypercapnia, hypoventilation, and hypoxemia

On arrival at the intensive care unit, a critically ill client suffers respiratory arrest and is placed on mechanical ventilation. The physician orders pulse oximetry to monitor the client's arterial oxygen saturation (SaO2) noninvasively. Which vital sign abnormality may alter pulse oximetry values? a) Fever b) Tachypnea c) Tachycardia d) Hypotension

d) Hypotension

A client hospitalized for treatment of a pulmonary embolism develops respiratory alkalosis. Which clinical findings commonly accompany respiratory alkalosis? a) Nausea or vomiting b) Abdominal pain or diarrhea c) Hallucinations or tinnitus d) Light-headedness or paresthesia

d) Light-headedness or paresthesia

A client has the following arterial blood gas (ABG) values: pH, 7.12; partial pressure of arterial carbon dioxide (PaCO2), 40 mm Hg; and bicarbonate (HCO3-), 15 mEq/L. These ABG values suggest which disorder? a) Respiratory alkalosis b) Respiratory acidosis c) Metabolic alkalosis d) Metabolic acidosis

d) Metabolic acidosis

A client undergoes a tracheostomy after many failed attempts at weaning him from a mechanical ventilator. Two days after tracheostomy, while the client is being weaned, the nurse detects a mild air leak in the tracheostomy tube cuff. What should the nurse do first? a) Call the physician. b) Remove the malfunctioning cuff. c) Add more air to the cuff. d) Suction the client, withdraw residual air from the cuff, and reinflate it.

d) Suction the client, withdraw residual air from the cuff, and reinflate it.

A nurse on the medical-surgical unit just received report on her client care assignment. Which client should she assess first? a) The client with anorexia, weight loss, and night sweats b) The client with crackles and fever who is complaining of pleuritic pain c) The client who had difficulty sleeping, daytime fatigue, and morning headache d) The client with petechiae over the chest who's complaining of anxiety and shortness of breath

d) The client with petechiae over the chest who's complaining of anxiety and shortness of breath

The nurse observes a new environmental services employee enter the room of a client with severe acute respiratory syndrome (SARS). Which action by the employee requires immediate intervention by the nurse? a) The employee wears a gown, gloves, N95 respirator, and eye protection when entering the room. b) The employee doesn't remove the stethoscope, blood pressure cuff, and thermometer that are kept in the room. c) The employee removes all personal protective equipment and washes her hands before leaving the client's room. d) The employee enters the room wearing a gown, gloves, and a mask.

d) The employee enters the room wearing a gown, gloves, and a mask.

At 11 p.m., a client is admitted to the emergency department. He has a respiratory rate of 44 breaths/min. He's anxious, and wheezes are audible. The client is immediately given oxygen by face mask and methylprednisolone (Depo-medrol) I.V. At 11:30 p.m., the client's arterial blood oxygen saturation is 86%, and he's still wheezing. The nurse should plan to administer: a) alprazolam (Xanax). b) propranolol (Inderal). c) morphine. d) albuterol (Proventil).

d) albuterol (Proventil).

A client is undergoing a complete physical examination as a requirement for college. When checking the client's respiratory status, the nurse observes respiratory excursion to help assess: a) lung vibrations b) vocal sounds c) breath sounds d) chest movements

d) chest movements

The nurse assessing a client for tracheal displacement should know that the trachea will deviate toward the: a) contralateral side in a simple pneumothorax. b) affected side in a hemothorax. c) affected side in a tension pneumothorax. d) contralateral side in a hemothorax.

d) contralateral side in a hemothorax.

Before administering ephedrine, the nurse assesses the client's history. Because of ephedrine's central nervous system (CNS) effects, it is not recommended for: a) clients with an acute asthma attack. b) clients with narcolepsy. c) clients under age 6. d) elderly clients.

d) elderly clients.

A client with chronic obstructive pulmonary disease tells the nurse that he feels short of breath. The client's respiratory rate is 36 breaths/min and the nurse auscultates diffuse wheezes. His arterial oxygen saturation is 84%. The nurse calls the assigned respiratory therapist to administer a prescribed nebulizer treatment. The therapist says, "I have several more nebulizer treatments to do on the unit where I am now. As soon as I'm done, I'll come assess the client." The nurse's most appropriate action is to: a) notify the primary physician immediately. b) stay with the client until the therapist arrives. c) administer the treatment by metered-dose inhaler. d) give the nebulizer treatment herself.

d) give the nebulizer treatment herself.

A client with Guillain-Barré syndrome develops respiratory acidosis as a result of reduced alveolar ventilation. Which combination of arterial blood gas (ABG) values confirms respiratory acidosis? a) pH, 7.5; PaCO2 30 mm Hg b) pH, 7.40; PaCO2 35 mm Hg c) pH, 7.35; PaCO2 40 mm Hg d) pH, 7.25; PaCO2 50 mm Hg

d) pH, 7.25; PaCO2 50 mm Hg

A client admitted with acute anxiety has the following arterial blood gas (ABG) values: pH, 7.55; partial pressure of arterial oxygen (PaO2), 90 mm Hg; partial pressure of arterial carbon dioxide (PaCO2), 27 mm Hg; and bicarbonate (HCO3-), 24 mEq/L. Based on these values, the nurse suspects: a) metabolic acidosis. b) metabolic alkalosis. c) respiratory acidosis. d) respiratory alkalosis.

d) respiratory alkalosis.

The nurse prepares to perform postural drainage. How should the nurse ascertain the best position to facilitate clearing the lungs? a) Inspection b) Chest X-ray c) Arterial blood gas (ABG) levels d)Auscultation

d)Auscultation

The most important action the nurse should do before and after suctioning a client is: a. Placing the client in a supine position b. Making sure that suctioning takes only 10-15 seconds c. Evaluating for clear breath sounds d. Hyperventilating the client with 100% oxygen

d. Hyperventilating the client with 100% oxygen

A seven-month-pregnant female is sitting quietly in the waiting room, and her respiratory rate is 20 and shallow. What does this finding suggest to the nurse? a. She has a history of smoking. b. She is using accessory muscles to breathe. c. She is in pending respiratory failure. d. Nothing. This is normal.

d. Nothing. This is normal.

After examining a 75-year-old male client, the nurse writes down "barrel chest." What does this finding suggest? a. The client has a history of smoking. b. The client has osteoporosis. c. The client has long-standing respiratory disease. d. This is a change associated with aging.

d. This is a change associated with aging.

The nurse sees that the client will breathe deeply and then stop breathing for a short while. Which of the following does this observation suggest? a. This client is hyperventilating. b. This client is in a diabetic coma. c. This client has pneumonia. d. This is seen in aging people, people with heart failure, and people who have suffered brain damage.

d. This is seen in aging people, people with heart failure, and people who have suffered brain damage.

in emphysema, the appetite __________ the weight _______ and the ap diameter of the chest __________

decreases, decreases, increases

The right atrium recieves ____ blood from the upper body by way of the superior vena cava and from the lower body by way of the inferior vena cava.

deoxygenated

What are the s/s of metabolic alkalosis? (4)

depends on the cause, watch LOC, hypokalemia (watch for muscle cramps & arrhythmias), decreased respiratory rate

pessary

designed to tighten the vaginal wall to treat cystocele

Discharge instructions for vag surgery

douches, laxitives, don't lift heavy lifting, no prolonged standing, walking or sitting. avoid intercourse until given the ok by physician. may be loss of vag sensation but it is only temporary.

What kind of precautions are rubella?

droplet

decongestants

drugs that constrict blood vessels in the respiratory tract, resulting in shrinkage of swollen mucous membranes and opened nasal airway passages

fine crackle

dry, high-pitched crackling and popping lung sounds of short duration

Describe percussion in person with lung cancer?

dull or flat sounding on percussion

A male adult client is suspected of having a pulmonary embolus. A nurse assesses the client, knowing that which of the following is a common clinical manifestation of pulmonary embolism?

dyspnea

external respiration

exchange of gases between the atmosphere and the lungs

internal respiration

exchange of oxygen and carbon dioxide at the cellular level

A nurse is assisting with the care of a client who has a tracheostomy in place. The nurse determines that the client's airway secretions require suctioning after auscultating the lung fields and prepares to apply suction using the open method. Put the following steps in order: Apply suction. Explain the procedure to the client. Increase the flow of oxygen via the tracheostomy collar. Insert the suction catheter into the tracheostomy.

explain the procedure to the client. increase the flow of oxygen via the tracheostomy collar. insert the suction catheter into the tracheostomy apply suction.

What are the treatments for respiratory acidosis?

fix the breathing problem, treat PNA, pneumothorax pts will have chest tube(s), if pt post-op, encourage CDB

What are the treatments for metabolic alkalosis? (2)

fix the problem and replace K+

Exudate

fluid from thoracentesis that has increased protein and/or LDH.

what dietary prescription is most appropriate for the client with emphysema

frequent small meals to prevent tiring

Location of RUL

from ribs 2-4. Locate manubrium and angle of louis= rib2 posterior- T1-T4

A patient with acute respiratory distress syndrome (ARDS) and acute renal failure has the following medications prescribed. Which medication should the nurse discuss with the health care provider before administration?

gentamicin (Garamycin) 60 mg IV rationale: Gentamicin, which is one of the aminoglycoside antibiotics, is potentially nephrotoxic, and the nurse should clarify the drug and dosage with the health care provider before administration. The other medications are appropriate for the patient with ARDS.

The irritant:

gets stuck in the mucous. The mucous stimulates coughing.

the persons with emphysema have ____, ______ lip, and _____ breathing

grunting, pursed lip, rapid breathing

what do you use to clean a tracheostomy cannula?

half strength peroxide and rinse with saline

what does stridor sound like?

harsh, high pitched noises on inspiration

When is nasopharyngeal suctioning contraindicated? Why?

head injury patients Can cause increased ICP

S/E leukotirene

headache, cough, nasal congestion or GI upset

epistaxis

hemorrhage of the nares or nostrils; also known as nosebleed

Diet for person with COPD

high calorie high protein

stridor

high-pitched, harsh sound heard on inspiration when the trachea or larynx is obstructed

How long do the kidneys take to correct pH imbalance?

hours to days

metabolic acidosis = hyper___________

hyperkalemia

What are the s/s of metabolic acidosis?

hyperkalemia, increased respiratory rate (eliminating CO2), s/s also depend on the cause

What are the causes of respiratory alkalosis? (3)

hyperventilation, hysterical, acute ASA overdose

metabolic alkalosis = hypo___________.

hypokalemia

What are the causes of respiratory acidosis? (7)

hypoventilation (retaining CO2), abdominal surgery, narcotics, sleeping pills, pneumothorax, collapsed lung, pneumonia

What is the time frame to send an ABG sample to the lab?

immediately after taking the sample

Positive TB skin test

immunocompromised >= 5mm health care worker 10mm normal people 15mm

Loss of weight in COPDer

in smoker this could indicate cancer, and is treated as cancer until it is ruled out. with just COPD it is an indication that the pt will not be living much longer.

Club fingers: definition and cause

increase in the angle between the base of the nail and the fingernail to 180 degrees or more, usually accompanied by an increase in the depth, bulk and sponginess of the end of the finger. Caused by prolonged hypoxemia, cystic fibrosis, lung cancer, bronchiectasis.

describe tactile fremitus in person with lung cancer?

increased because airspaces are replaced with tumor tissue or fluid

pneumonia

inflammation of the bronchioles and alveoli accompanied by consolidation, or solidification of exudate, in the lungs

A patient develops increasing dyspnea and hypoxemia 2 days after having cardiac surgery. To determine whether the patient has acute respiratory distress syndrome (ARDS) or pulmonary edema caused by left ventricular failure, the nurse will anticipate assisting with ____________

inserting a pulmonary artery catheter. rationale: Pulmonary artery wedge pressures are normal in the patient with ARDS because the fluid in the alveoli is caused by increased permeability of the alveolar-capillary membrane rather than by the backup of fluid from the lungs (as occurs in cardiogenic pulmonary edema). The other tests will not help in differentiating cardiogenic from noncardiogenic pulmonary edema.

Thoracentesis

insertion of a large-bore needle through the chest wall into the pleural space to obtain specimens for diagnostic evaluation, remove pleural fluid, or instill medication into the pleural space. DO NOT remove more then 1000cc of fluid becasue it can cause Hypovolumic shock.

Are crackles heard during inspiration or expiration?

inspiration

Asthma is characterized by:

intermittent airflow obstruction

Military TB

involvemnet of many organs simultaneously as a result of primary disease or reactivation of latent iinfection. S/S: fever dyspnea and cyanosis or systemic manifestations of wt loss, fever and gi disturbance.

PID

involves the fallopian tubes, ovaries and pelvic peritoneum.

Incentive spirometry is used to measure the amount of air that:

is inspired with one inhalation

This inflammation/mucous response:

is not supposed to be overstimulated where it causes airflow limitation. It's supposed to be a very short lived mechanism that subsides.

Chronic inflammation:

is very damaging to the body.

If an irritant gets caught in the airway,

it irritates the cilia and causes minor inflammatory response, triggering the mucous glands to secrete mucous.

What is special about the syringe that you aspirate an ABG sample into?

it must be heparinized

With respiratory acidosis, which body part will compensate?

kidneys by secreting bicarb and excreting acid (thru urine)

The swollen airway:

leads to wheezing and the mucous can lead to rhonchi.

What are the s/s of respiratory alkalosis? (4)

lightheaded, faint, peri-oral numbness, tingling in the fingers & toes

Common sites for metastatic growth

liver, brian, bones, scalene lymph nodes, and adrenal glands.

Decubitis film

looking for moving pleural effusion, free flowing fluid. Takes and xray standing and then laying to see if the fluid moves or is trapped.

V/Q scans

looks for pulmonary embolism. IV radioisotope given to assess perfusion. Pt inhales radioactive gas, which outlines the alveoli. Normal= homogeneous radioactivity. Diminished or absent= lack of perfusion. Ventilation without perfusion suggests PE.

What are the causes of metabolic alkalosis?

loss of upper GI content, too many antacids (too much base), too much IV bicarb

What do bronchial breath sounds sound like?

loud, high-pitched sounds that resemble air blowing thru a hollow pipe

bronchial sound

loud, high-pitched, hollow-sounding breath sound normally heard over the sternum. Longer on expiration than inspiration

what flow rates of o2 are appropriate for the client with emphysema?

low flow- <2.5L/min, never exceed 2.5 in COPD

emphysema

lung disease wherein air accumulates in the tissues of the lungs or pus in the pleural space

In which patients would you hear abnormal bronchial breath sounds?

lung mass, atelectasis, or pneumonia

bronchodilator

medications that relax the smooth muscles of the bronchial tree, thereby relieving bronchospasm and increasing the vital capacity of the lungs

coarse crackle

moist, low-pitched crackling and gurgling lung sound of long duration

where do high-pitched wheezes occur: expiration or inspiration?

mostly expiration but can be both

ventilation

movement of gases into and out of the lungs

A black client with asthma seeks emergency care for acute respiratory distress. Because of this client's dark skin, the nurse should assess for cyanosis by inspecting the:

mucous membranes

what do high-pitched wheezes sound like?

musical sounds that predominate in expiration

Closed pneumothorax

no associated external wound. Spontaneous pneumothorax. Can be caused by rupture of blebs on the visceral pleura.

Transudate

no protein but fluid from blood vessels is still leaking. usually found with CHF and renal failure.

In what kind of pts would you hear rhonchi?

obstruction in trachea or bronchi

Cystocele

occurs when spport btwn the vagina and bladder is weakened

Presentation of Acute bronchitis

occurs with or after upper respiratory tract infection. Symptoms: cough 10-20 days, clear, mucoid sputum, or purulent sputum, headache, malaise, SOB on exertion and low grade fever. Elevated temp, resp rate and pulse, normal or wheeze breath sounds with expiration and exertion. Differentiate btwn pneumonia= CXR Treatment: rest, fluids, antiinflammatory agents, cough suppressants or bronchdilators.

Possible reasons for low readings

older adults dark skin hypothermia poor peripheral blood flow too much light (sun or infrared lights) low Hgb levels client movement edema nail polish

When the nurse is caring for an obese patient with left lower lobe pneumonia, gas exchange will be best when the patient is positioned _______________

on the right side. rationale: The patient should be positioned with the "good" lung in the dependent position to improve the match between ventilation and perfusion. The obese patient's abdomen will limit respiratory excursion when sitting in the high-Fowler's or tripod positions.

Where are bronchovesicular breath sounds normally heard?

over the first and second intercostal spaces at the sternal border anteriorly and at the T4 level medial to the scapula posteriorly (over major bronchi)

in emphysema, the alveoli are over __________ and under________

over-enlarged and underventilated so air is trapped in the alveoli

the left atrium receives _____ blood from the lungs by the way of the four pulmonary veins

oxygenated

An ABG measures:

pH - amount of free hydrogen ions in the arterial blood (H+). PaO2 - partial pressure of oxygen PaCO2 - partial pressure of carbon dioxide HCO3 - the concentration of bicarbonate in arterial blood. SaO2 - percentage of oxygen bound to Hgb as compared to the total amount that can be possibly carried

ABG values

pH: 7.35-7.45 PaO2: 80-100% >60 for COPD pt PaCO2: 35-45 (acidic= 45, alkalotic= 35) HCO3: 22-26

ABG interpretations

pH: 7.35-7.45 (<7.35=acidosis; >7.45=alkalosis) PaO2: 80-100 PaCO2: 35-45 HCO3: 22-26 SaO2: 95-100%

Activity regimen for patient with COPD

pacing activities short rest periods

Pack year

packs of day x number of years = pack year

Bronchoscopy

permits visualization of the larynx, trachea, and bronchi through either a flexible fiberoptic bronchoscope or a rigid bronchoscope.

status asthmaticus

persistent, intractable asthma attack Severe life threatening form of asthma

surfactant

phospholipids that are present in the lungs and lowers surface tension to prevent collapse of the airways

What lung sound sounds like a superficial, low-pitched, coarse grating sound that sounds like 2 rough surfaces moving against each other?

pleural friction rub

In which patients would you hear a pleural friction rub?

pleural inflammation often associated with pleurisy, pneumonia, or pleural infarction

complications

pleurisy, pleural effusion, atelectasis, empyema, meningitis, endocarditis

diagnosis for hyperresonance

pneumothorax, emphysema and COPD

hemopneumothorax

presence of blood and air within the pleural space

post op care for vag surgery

prevent wound infection and pressure on vag suture line. Peri care bid and after urination and defication. ice and sitz bath foley cath, stool softener.

respiration

process of exchanging oxygen and carbon dioxide

diffusion

process whereby a substance moves from an area of higher concentration to an area of lower concentration

Mucous is a:

protective measure.

Empymea

puss filled fluid

Tension Pneumothorax

rapid accumulation of air in the pleural space can cause high intrapleural pressures. causes compression of the lung on the affected side and pressure on the heart. can result from either open or closed pneumothorax.

What are Kussamal's respirations?

rapid, deep & sighing breathing air hunger commonly seen with metabolic acidosis or DKA

What are Kussamal's respirations?

rapid, deep & sighing breathing, air hunger, commonly seen with metabolic acidosis or DKA

Hematocrit Levels

reflects ratio of RBC's to plasma. Chronic hypoxemia= increased hematocrit male= 39-50% female= 35-47%

ABG sample

reports the status of oxygenation and acid-base balance of the blood

Describe Tachypnea

respirations > 35 clinical significance/contributing factors: - respiratory failure - response to fever - anemia - pain - respiratory infection - anxiety (emergencies SNS system kicks in)

What are the early s/s of hypoxia?

restlessness & tachycardia

Flail chest

results from the fracture of 2 or more ribs in two or more separate locations causin gan unstable segment. Pt has rapid shallow respirations and tachycardia. Chest moves in with inspiration and out with expiration.

What lung sound is low-pitched, coarse, loud snoring/ moaning?

rhonchi

Bronchoscopy

scope of airway throught tright and left mainstem bronchus. -flexible scope can extend further -washing and brushing -biopsy -removal of obstructions - muscous plug, foreign bodies -ridgid- usually used to remove foreign bodies

what position is best for clients with emphysema under normal circumstances?

semi-fowlers or higher

Complications of PID

septic shock, Pitz-Hugh-Curtis syndrome- spreads to liver and causes acute perihepatitis. adheasions and strictures in fallopian tubes ectopic pregnancy. infertility

Oxygen-Hemoglobin Dissociation Curve

shows shifts in whether oxygen is being picked up by lungs and delievered to tisues.

Leukotriene modifiers

singulair, acalaide

pacemaker of the heart

sinoatrial (SA) node

Signs and symptoms of Pneumothorax

small mild tachycardia and dyspnea. Resp distress for lg pneumothorax along with shallow rapid respirations, dyspnea, air hunger and o2 desaturation.

the development of emphysema is msot associated with a history of

smoking

S/S of uterine prolapse

something heavy, coming down. dyspareunia, hachache, and bowel or bladder problems, stress incontinence

A client is admitted to the hospital with possible rheumatic endocarditis. The nurse would chekc the client for signs and symptoms of concrrent _________

streptococcal infection

Thoracentesis

surgical perforation of the chest wall and pleural space with a large-bore needle. It is performed to obtain speciments for diag. eval., instill meds into the pleural space, & remove fluid (effusion) or air from the pleural space fro therapeutic relief of pleural pressure.

That means there's less distance to go completely:

swell shut if they come into contact with irritants or triggers.

Epinephrine secreated by the adrenal medulla in stressful situations is ________

sympathomimetic

Which of the following interventions is most appropriate for the patient with an ineffective breathing pattern

teach the patient controlled diaphragmatic breathing

In metabolic acidosis & alkalosis, what organ is sick and what organ is compensating?

the kidneys are sick and the lungs will compensate

With chronic inflammation:

the lumen of the airway becomes very swollen and on top of it, filled with mucous.

Inflammations triggers:

the mucous glands to produce more mucous.

what do crackles sound like?

the sound of a lock of hair being rubbed between the thumb and forefinger

The client with a history of diabetes insipidus is admitted with polyuria, polydipsia, and mental confusion. The priority intervention for this client is:

the vital signs

So even on a good day,

they aren't ever going to have bronchi and bronchiole that are nice and clear and not swollen.

Treatment of pneumothorax

thoracentesis or chest tube

Which way will the trachea deviate in a tension pneumothorax?

to the unaffected side

With respiratory acidosis, what chemical is causing the problem?

too much CO2

Where are bronchial breath sounds normally heard in the resp tract?

trachea and immediately above the manubrium

Goblet cells

trap foreign bodies. Found in respitory system. similar function as fly paper

if a client with emphysema has severe dyspneic episode what position is best

tripod

Biots

unpredictable periods of apnea usually following specific surgeries

COPDers can have:

up to a liter of mucous a day.

Serum K+ will go ______ in metabolic acidosis and go _______ in metabolic alkalosis.

up, down

when should you apply suction to a tracheostomy?

upon withdrawal, not while inserting

In what kind of pts would you hear stridor?

upper airway obstruction, airway edema

Uterine prolapse

uterus moves into the vaginal canal. 1st degree- cervix rests in lower part of vagina 2nd degree- cervix is at the vaginal opening. 3rd degree- uterus protrudes through the introitus.

rectocele

weakening between vagina and rectum

Corina

wehre trachea splits to R&L bronchus. @ 4th intercostal space

Describe the clinical signs of RIGHT sided heart failure.

weight gain distended neck veins hepatomegaly and splenomegaly dependent peripheral edema

audible wheeze

wheeze that can be heard without the aid of a stethoscope

Blebs or Bula

when alveoli turn into balloons instead of grapes. happens in COPD

When MUST O2 be humidified?

when deliver is >4L or directly to the trachea

When is a chest tube removed?

when the lungs have reexpanded and/or there is no more fluid drainage

what do you have to teach a patient about Rifampin? What is Rifampin used for?

will turn urine and other secretions orange pulmonary TB

If the trigger isn't removed,

you will be in a constant state of inflammation and the asthma will become chronic rather than being reversible.

Nursing Interventions/Client Education for Beta2-Adrenergic Agonists

• Advise clients to observe for signs and symptoms of tachcardia and angina (chest, jaw, or arm pain or palpitations) and to notify the provider if they occur. • Instruct clients on how to check pulse and to report an increase of greater than 20 to 30 beats/min. • Advise clients to avoid caffeine. • Dosage may need to be lowered. • Tremors usually resolve with continued medication use. • Dosage may need to be reduced.

Interactions with Methylxanthines

• Caffeine increases CNS and cardiac adverse effects of theophylline. • Caffeine can also increase theophylline levels. • Phenobarbital and phenytoin decrease theophylline levels. • Cimetidine (Tagamet), ciprofloxacin (Cipro), and other fluoroquinolone antibiotics increase theophylline levels.

Side/Adverse Effects of Methylxanthines

• Mild toxicity reaction may include GI distress and restlessness. • More severe reactions can occur with higher therapeutic levels and can include dysrhythmias and seizures

Nursing Interventions/Client Education of Methylxanthines

• Monitor theophylline serum levels to keep within therapeutic range (5 to 15 mcg/mL). Side effects are unlikely to occur at levels less than 20 mcg/mL. • If symptoms occur, stop the medication. If necessary, activated charcoal can be used to decrease absorption, lidocaine can be used to treat dysrhythmias, and diazepam can be used to control seizures. • Instruct client that periodic blood levels will be needed. Advise client to report any symptoms of nausea, diarrhea, or restlessness that may indicate toxicity.

Purpose of Mast Cell Stabilizers (Anti-inflammatories)

● Expected Pharmacological Action ◯ Anti-inflammatory action ■ These medications stabilize mast cells, which inhibits the release of histamine and other inflammatory mediators. ■These medications suppress inflammatory cells (eosinophils, macrophages). ● Therapeutic Uses ◯ Management of chronic asthma ◯ Prophylaxis of exercise-induced asthma ◯ Prevention of allergen-induced attack ◯ Allergic rhinitis by intranasal route ◯ Route of administration: inhalation

Purpose of LEUKOTRIENE MODIFIERS

● Expected Pharmacological Action ◯ Leukotriene modifiers prevent the effects of leukotrienes, thereby suppressing inflammation, bronchoconstriction, airway edema, and mucus production. ● Therapeutic Uses ◯ Leukotriene modifiers are used for long-term therapy of asthma in adults and children 15 years and older and to prevent exercise-induced bronchospasm. ◯ Route of administration: oral

Contraindications/Precautions of Methylxanthines

● Pregnancy Risk Category C ● Use cautiously in clients who have heart disease, hypertension, liver and renal dysfunction, and diabetes. ● Use cautiously in children and older adults.

Contraindications/Precautions of Glucocorticoids

● Pregnancy risk category C ● Contraindicated in clients who have received a live virus vaccine ●Contraindicated in clients with systemic fungal infections ●Use cautiously in children, and in clients who have diabetes, hypertension, peptic ulcer disease, and/or renal dysfunction. ● Use cautiously in clients taking NSAIDs.

Complications of Mast Cell Stabilizers (Anti-inflammatories)

● Safest of all asthma medications ● Safe to use for children

Beta2-Adrenergic Agonists

● Select Prototype Medication: albuterol (Proventil, Ventolin) ●Other Medications: Formoterol (Foradil Aerolizer) Salmeterol (Serevent) Terbutaline (Brethine)

Medication Classification: Mast Cell Stabilizers (Anti-inflammatories)

● Select Prototype Medication: cromolyn sodium (Intal) ● Other Medication: nedocromil sodium (Tilade)

MEDICATION CLASSIFICATION: INHALED ANTICHOLINERGICS

● Select Prototype Medication: ipratropium (Atrovent) ●Other Medications: tiotropium (Spiriva) ● Expected Pharmacological Action: ◯These medications block muscarinic receptors of the bronchi, resulting in bronchodilation. ●Therapeutic Uses ◯ These medications are used to relieve bronchospasm associated with chronic obstructive pulmonary disease. ◯ These medications are used for allergen-induced and exercise-induced asthma. ◯ Route of administration: inhalation

MEDICATION CLASSIFICATION: LEUKOTRIENE MODIFIERS

● Select Prototype Medication: montelukast (Singulair) ● Other Medication: zileuton (Zyflo), zafirlukast (Accolate)

Medication Classification: Methylxanthines

● Select Prototype Medication: theophylline ● Expected Pharmacological Action ◯ Theophylline causes relaxation of bronchial smooth muscle, resulting in bronchodilation ● Therapeutic Uses ◯ Oral theophylline is used for long-term control of chronic asthma. ◯Route of administration: oral or IV (emergency use only)

Contraindications/Precautions of Mast Cell Stablizers (Anti-inflammatories)

● These agents are Pregnancy Risk Category B. ● Fluorocarbons in aerosols make this medication contraindicated for clients who have coronary artery disease, dysrhythmias, and status asthmaticus. ● Use cautiously in clients with liver and kidney impairment.

Contraindications/Precautions Beta2-Adrenergic Agonists

●Beta2-adrenergic agonists are Pregnancy Risk Category C. ●These agents are contraindicated in clients with tachydysrhythmia. ●Use cautiously in clients who have diabetes, hyperthyroidism, heart disease, hypertension, and angina.

Contraindications/Precautions of Inhaled Anticholinergics

●Inhaled anticholinergics are Pregnancy Risk Category B. ●These agents are contraindicated in clients who have an allergy to peanuts because the medication preparations may contain soy lecithin. ● Use cautiously in clients who have narrow-angle glaucoma and benign prostatic hypertrophy (due to anticholinergic effects).

Medication Classification: Glucocorticoids

●Select Prototype Medication: ◯ Inhalation: beclomethasone dipropionate (QVAR) ◯ Oral: prednisone (Deltasone) ●Other Medications: ◯Inhalation: ■ Budesonide (Pulmicort Flexhaler) ■ Fluticasone propionate (Advair, Flovent) ■ Triamcinolone acetonide (Azmacort) ◯ Oral: Prednisolone (Prelone) ◯ IV: ■ Hydrocortisone sodium succinate (Solu-Cortef) ■ Methylprednisolone sodium succinate (Solu-Medrol)

Therapeutic Uses of Glucocorticoids

◯ Short-term IV agents are used for status asthmaticus. ◯Inhaled agents are used for long-term prophylaxis of asthma. ◯ Short-term oral therapy is used to treat symptoms following an acute asthma attack. ◯ Long-term oral therapy is used to treat chronic asthma. ◯ Replacement therapy is used for primary adrenocortical insufficiency. ◯ Promote lung maturity and decrease respiratory distress in fetuses at risk for preterm birth.

Mg

1.5-2.5 mEq/L

Na

135-145 mEq/L

HCO3

22-26 mEq/L

Phosphate

3.0-4.5 mg/dL

K+

3.5-5.0 mEq/L

PaCO2

35-45 mm Hg

Ca (ionized serum)

4.5-5.1 mg/dL

pH

7.35-7.45

Ca (total serum)

8.5-10.5 mg/dL

PaO2

80-100 mm Hg

Cl

95-105 mEq/L

SaO2

95-98%

Hypermagnesemia (assessment)

CNS depression, hypotension, facial flushing, musc. weakness, absent deep tendon reflexes, paralysis, shallow respirations, depressed cardiac impulse transmission

Hyperkalemia (assessment)

EKG changes, cardiac arrest, paralysis, diarrhea, muscle weakness, nausea, dysrhthmias

Hypocalcemia (assessment)

NS excitability, tetany, Trouseau's/Chvostek's signs, seizures, confusion, parasthesias, irritability

Hypercalcemia (assessment)

NS sedation, muscle weakness, lack of coordination, constipation, abd. pain/distension, confusion, depressed/absent tendon reflexes, dysrhythmias

Chvostek's Sign

Tap facial nerve 2cm anterior to ear lobe below zygomatic arch; twitching of facial muscles indicates tetany

Hyponatremia (causes)

Vomitting, diuretics, excessive admin. of D5 & water, prolonged low Na diet, excessive H2O intake

Hypomagnesemia (causes)

alcoholism, GI suctioning, diarrhea, intestinal fistulas, diuretic/laxativr abuse, **usually seen w/other lyte imbalances

Hypokalemia (assessment)

anorexia, parasthesias, vomitting, increased sensitivity to digitalis, muscle weakness, dysrhythmias, nausea

Hypernatremia (assessment)

elev. temp., weakness, disorientation, hypotension, tachycardia, delusion and hallucinations, thirst, swollen tongue, sticky mucous membranes

Hypercalcemia (causes)

hyperparathyroidism, malignant neoplastic disease, immobility, excessive intake of Ca or Ca carbonate antacids

Hypernatremia (causes)

hypertonic tube feedings w/out H2O supp., diarrhea, hyperventilation, DI, alka-seltzer, near drowning (salt water), inad. H2O consumption.

Hypocalcemia (causes)

hypoparathyroidism, pancreatitis, renal failure, steroids/loop diuretics, inadequate intake, post-thyroid surgery

Hypomagnesemia (assessment)

increased neuromusc. irritability, tremors, tetany, seizures, dysrhythmias, depression, confusion, dysphagia

Trouseau's Sign

inflate BP cuff on upper arm to 20 mm Hg above systolic pressure; carpal spasms w/in 2-5 min. indicates tetany

Hyponatremia (assessment)

nausea, muscle cramps, muscle twitching, increased ICP, confusion, convulsions

Hypermagnesemia (causes)

renal failure, excessive Mg administration (antacids/cathartics)

Hyperkalemia (causes)

renal failure, use of supplements, burns, crushing injuries

Hypokalemia (causes)

vomiting, gastric suctioning, diarrhea, diuretics and steroids, inadequate intake

Which of the following is significant data to gather from a client who has been diagnosed with pneumonia? Select all that apply. 1. Quality of breath sounds. 2. Presence of bowel sounds. 3. Occurrence of chest pain. 4. Amount of peripheral edema. 5. Color of nail beds.

1, 3, 5. A respiratory assessment, which includes auscultating breath sounds and assessing the color of the nail beds, is a priority for clients with pneumonia. Assessing for the presence of chest pain is also an important respiratory assessment as chest pain can interfere with the client's ability to breathe deeply. Auscultating bowel sounds and assessing for peripheral edema may be appropriate assessments, but these are not priority assessments for the client with pneumonia.

The nurse administers two 325 mg aspirin every 4 hours to a client with pneumonia. The nurse should evaluate the outcome of administering the drug by assessing which of the following? Select all that apply. 1. Decreased pain when breathing. 2. Prolonged clotting time. 3. Decreased temperature. 4. Decreased respiratory rate. 5. Increased ability to expectorate secretions.

1, 3. Aspirin is administered to clients with pneumonia because it is an analgesic that helps control chest discomfort and an antipyretic that helps reduce fever. Aspirin has an anticoagulant effect, but that is not the reason for prescribing it for a client with pneumonia, and the use of the drug will be short term. Aspirin does not affect the respiratory rate, and does not facilitate expectoration of secretions.

What are the treatments for respiratory alkalosis?

1. breathe into a paper bag 2. encourage the pt to decrease respiratory rate 3. treat the cause 4. monitor ABGs

What are the treatments for respiratory acidosis?

1. fix the breathing problem 2. treat PNA 3. pneumothorax pts will have chest tube(s) 4. if pt post-op, encourage CDB

What are the causes of respiratory alkalosis? (3)

1. hyperventilation 2. hysterical 3. ASA overdose (acute)

What are the causes of respiratory acidosis? (7)

1. hypoventilation (retaining CO2) 2. abdominal surgery 3. narcotics 4. sleeping pills 5. pneumothorax 6. collapsed lung 7. pneumonia

What are the s/s of respiratory alkalosis? (4)

1. lightheaded 2. faint 3. peri-oral numbness 4. tingling in the fingers & toes

When performing postural drainage, which of the following factors promotes the movement of secretions from the lower to the upper respiratory tract? 1. Friction between the cilia. 2. Force of gravity. 3. Sweeping motion of cilia. 4. Involuntary muscle contractions.

2. The principle behind using postural drainage is that gravity will help move secretions from smaller to larger airways. Postural drainage is best used after percussion has loosened secretions. Coughing or suctioning is then used to remove secretions. Movement of cilia is not sufficient to move secretions. Muscle contractions do not move secretions within the lungs.

A client with chronic obstructive pulmonary disease (COPD) is experiencing dyspnea and has a low PaO2 level. The nurse plans to administer oxygen as ordered. Which of the following statements is true concerning oxygen administration to a client with COPD? 1. High oxygen concentrations will cause coughing and dyspnea. 2. High oxygen concentrations may inhibit the hypoxic stimulus to breathe. 3. Increased oxygen use will cause the client to become dependent on the oxygen. 4. Administration of oxygen is contraindicated in clients who are using bronchodilators.

2. Clients who have a long history of COPD may retain carbon dioxide (CO2). Gradually the body adjusts to the higher CO2 concentration, and the high levels of CO2 no longer stimulate the respiratory center. The major respiratory stimulant then becomes hypoxemia. Administration of high concentrations of oxygen eliminates this respiratory stimulus and leads to hypoventilation. Oxygen can be drying if it is not humidified, but it does not cause coughing and dyspnea. Increased oxygen use will not create an oxygen dependency; clients should receive oxygen as needed. Oxygen is not contraindicated with the use of bronchodilators.

The nurse reviews an arterial blood gas report for a client with chronic obstructive pulmonary disease (COPD). pH 7.35; PC02 62; PO2 70; HCO3 34 The nurse should: 1. Apply a 100% non-rebreather mask. 2. Assess the vital signs. 3. Reposition the client. 4. Prepare for intubation.

2. Clients with chronic COPD have CO2 retention and the respiratory drive is stimulated when the PO2 decreases. The heart rate, respiratory rate, and blood pressure should be evaluated to determine if the client is hemodynamically stable. Symptoms, such as dyspnea, should also be assessed. Oxygen supplementation, if indicated, should be titrated upward in small increments. There is no indication that the client is experiencing respiratory distress requiring intubation.

When teaching a client with chronic obstructive pulmonary disease to conserve energy, the nurse should teach the client to lift objects: 1. While inhaling through an open mouth. 2. While exhaling through pursed lips. 3. After exhaling but before inhaling. 4. While taking a deep breath and holding it.

2. Exhaling requires less energy than inhaling. Therefore, lifting while exhaling saves energy and reduces perceived dyspnea. Pursing the lips prolongs exhalation and provides the client with more control over breathing. Lifting after exhaling but before inhaling is similar to lifting with the breath held. This should not be recommended because it is similar to the Valsalva maneuver, which can stimulate cardiac arrhythmias.

A client with pneumonia has a temperature of 102.6 ° F (39.2 ° C), is diaphoretic, and has a productive cough. The nurse should include which of the following measures in the plan of care? 1. Position changes every 4 hours. 2. Nasotracheal suctioning to clear secretions. 3. Frequent linen changes 4. Frequent offering of a bedpan.

3. Frequent linen changes are appropriate for this client because of the diaphoresis. Diaphoresis produces general discomfort. The client should be kept dry to promote comfort. Position changes need to be done every 2 hours. Nasotracheal suctioning is not indicated with the client's productive cough. Frequent offering of a bedpan is not indicated by the data provided in this scenario.

When caring for the client who is receiving an aminoglycoside antibiotic, the nurse should monitor which of the following laboratory values? 1. Serum sodium. 2. Serum potassium. 3. Serum creatinine. 4. Serum calcium.

3. It is essential to monitor serum creatinine in the client receiving an aminoglycoside antibiotic because of the potential of this type of drug to cause acute tubular necrosis. Aminoglycoside antibiotics do not affect serum sodium, potassium, or calcium levels.

The nurse teaches a client with chronic obstructive pulmonary disease (COPD) to assess for signs and symptoms of right-sided heart failure. Which of the following signs and symptoms should be included in the teaching plan? 1. Clubbing of nail beds. 2. Hypertension. 3. Peripheral edema. 4. Increased appetite.

3. Right-sided heart failure is a complication of COPD that occurs because of pulmonary hypertension. Signs and symptoms of right-sided heart failure include peripheral edema, jugular venous distention, hepatomegaly, and weight gain due to increased fluid volume. Clubbing of nail beds is associated with conditions of chronic hypoxemia. Hypertension is associated with left-sided heart failure. Clients with heart failure have decreased appetites.

A client with pneumonia is experiencing pleuritic chest pain. The nurse should assess the client for: 1. A mild but constant aching in the chest. 2. Severe midsternal pain. 3. Moderate pain that worsens on inspiration. 4. Muscle spasm pain that accompanies coughing.

3. Chest pain in pneumonia is generally caused by friction between the pleural layers. It is more severe on inspiration than on expiration, secondary to chest wall movement. Pleuritic chest pain is usually described as sharp, not mild or aching. Pleuritic chest pain is not localized to the sternum, and it is not the result of a muscle spasm.

The nurse administers theophylline (Theo-Dur) to a client. To evaluate the effectiveness of this medication, which of the following drug actions should the nurse anticipate? 1. Suppression of the client's respiratory infection. 2. Decrease in bronchial secretions. 3. Relaxation of bronchial smooth muscle. 4. Thinning of tenacious, purulent sputum.

3. Theophylline (Theo-Dur) is a bronchodilator that is administered to relax airways and decrease dyspnea. Theophylline is not used to treat infections and does not decrease or thin secretions.

A client who has been taking flunisolide (AeroBid), two inhalations a day, for treatment of asthma.has painful, white patches in his mouth. Which response by the nurse would be most appropriate? 1. "This is an anticipated adverse effect of your medication. It should go away in a couple of weeks." 2. "You are using your inhaler too much and it has irritated your mouth." 3. "You have developed a fungal infection from your medication. It will need to be treated with an antifungal agent." 4. "Be sure to brush your teeth and floss daily. Good oral hygiene will treat this problem."

3. Use of oral inhalant corticosteroids such as flunisolide (AeroBid) can lead to the development of oral thrush, a fungal infection. Once developed, thrush must be treated by antifungal therapy; it will not resolve on its own. Fungal infections can develop even without overuse of the corticosteroid inhaler. Although good oral hygiene can help prevent development of a fungal infection, it cannot be used alone to treat the problem.

The cyanosis that accompanies bacterial pneumonia is primarily caused by which of the following? 1. Decreased cardiac output. 2. Pleural effusion. 3. Inadequate peripheral circulation. 4. Decreased oxygenation of the blood.

4. A client with pneumonia has less lung surface available for the diffusion of gases because of the inflammatory pulmonary response that creates lung exudate and results in reduced oxygenation of the blood. The client becomes cyanotic because blood is not adequately oxygenated in the lungs before it enters the peripheral circulation. Decreased cardiac output may be a comorbid condition in some clients with pneumonia; however, it is not the cause of cyanosis. Pleural effusions are a potential complication of pneumonia but are not the primary cause of decreased oxygenation. Inadequate peripheral circulation is also not the cause of the cyanosis that develops with bacterial pneumonia.

The nurse assesses the respiratory status of a client who is experiencing an exacerbation of chronic obstructive pulmonary disease (COPD) secondary to an upper respiratory tract infection. Which of the following findings would be expected? 1. Normal breath sounds. 2. Prolonged inspiration. 3. Normal chest movement. 4. Coarse crackles and rhonchi.

4. Exacerbations of COPD are commonly caused by respiratory infections. Coarse crackles and rhonchi would be auscultated as air moves through airways obstructed with secretions. In COPD, breath sounds are diminished because of an enlarged anteroposterior diameter of the chest. Expiration, not inspiration, becomes prolonged. Chest movement is decreased as lungs become overdistended.

A 34-year-old female with a history of asthma is admitted to the emergency department. The nurse notes that the client is dyspneic, with a respiratory rate of 35 breaths/ minute, nasal flaring, and use of accessory muscles. Auscultation of the lung fields reveals greatly diminished breath sounds. Based on these findings, which action should the nurse take to initiate care of the client? 1. Initiate oxygen therapy and reassess the client in 10 minutes. 2. Draw blood for an arterial blood gas analysis and send the client for a chest X-ray. 3. Encourage the client to relax and breathe slowly through the mouth. 4. Administer bronchodilators.

4. In an acute asthma attack, diminished or absent breath sounds can be an ominous sign indicating lack of air movement in the lungs and impending respiratory failure. The client requires immediate intervention with inhaled bronchodilators, I.V. corticosteroids and, possibly, I.V. theophylline (Theo-Dur). Administering oxygen and reassessing the client 10 minutes later would delay needed medical intervention, as would drawing blood for an arterial blood gas analysis and obtaining a chest X-ray. It would be futile to encourage the client to relax and breathe slowly without providing the necessary pharmacologic intervention.

Which of the following diets would be most appropriate for a client with chronic obstructive pulmonary disease (COPD)? 1. Low-fat, low-cholesterol diet. 2. Bland, soft diet. 3. Low-sodium diet. 4. High-calorie, high-protein diet.

4. The client should eat high-calorie, high-protein meals to maintain nutritional status and prevent weight loss that results from the increased work of breathing. The client should be encouraged to eat small, frequent meals. A low-fat, low-cholesterol diet is indicated for clients with coronary artery disease. The client with COPD does not necessarily need to follow a sodium-restricted diet, unless otherwise medically indicated. There is no need for the client to eat bland, soft foods.

Which of the following measures would most likely be successful in reducing pleuritic chest pain in a client with pneumonia? 1. Encourage the client to breathe shallowly. 2. Have the client practice abdominal breathing. 3. Offer the client incentive spirometry. 4. Teach the client to splint the rib cage when coughing.

4. The pleuritic pain is triggered by chest movement and is particularly severe during coughing. Splinting the chest wall will help reduce the discomfort of coughing. Deep breathing is essential to prevent further atelectasis. Abdominal breathing is not as effective in decreasing pleuritic chest pain as is splinting of the rib cage. Incentive spirometry facilitates effective deep breathing but does not decrease pleuritic chest pain.

If a pt is restless, what should the nurse think of first?

hypoxia--check O2 sat

A female client is suspected of having a pulmonary embolus. A nurse assesses the client, knowing that which of the following is a common clinical manifestation of pulmonary embolism? a. Dyspnea b. Bradypnea c. Bradycardia d. Decreased respiratory

Answer A. The common clinical manifestations of pulmonary embolism are tachypnea, tachycardia, dyspnea, and chest pain

Nurse Kim is caring for a client with a pneumothorax and who has had a chest tube inserted notes continuous gentle bubbling in the suction control chamber. What action is appropriate? a. Do nothing, because this is an expected finding. b. Immediately clamp the chest tube and notify the physician. c. Check for an air leak because the bubbling should be intermittent. d. Increase the suction pressure so that bubbling becomes vigorous.

Answer A. Continuous gentle bubbling should be noted in the suction control chamber. Option B is incorrect. Chest tubes should only be clamped to check for an air leak or when changing drainage devices (according to agency policy). Option C is incorrect. Bubbling should be continuous and not intermittent. Option D is incorrect because bubbling should be gentle. Increasing the suction pressure only increases the rate of evaporation of water in the drainage system.

An unconscious male client is admitted to an emergency room. Arterial blood gas measurements reveal a pH of 7.30, a low bicarbonate level, a normal carbon dioxide level, a normal oxygen level, and an elevated potassium level. These results indicate the presence of: a. Metabolic acidosis b. Respiratory acidosis c. Overcompensated respiratory acidosis d. Combined respiratory and metabolic acidosis

Answer A. In an acidotic condition, the pH would be low, indicating the acidosis. In addition, a low bicarbonate level along with the low pH would indicate a metabolic state. Therefore, options B, C, and D are incorrect.

A male client has been admitted with chest trauma after a motor vehicle accident and has undergone subsequent intubation. A nurse checks the client when the high-pressure alarm on the ventilator sounds, and notes that the client has absence of breathe sounds in right upper lobe of the lung. The nurse immediately assesses for other signs of: a. Right pneumothorax b. Pulmonary embolism c. Displaced endotracheal tube d. Acute respiratory distress syndrome

Answer A. Pneumothorax is characterized by restlessness, tachycardia, dyspnea, pain with respiration, asymmetrical chest expansion, and diminished or absent breath sounds on the affected side. Pneumothorax can cause increased airway pressure because of resistance to lung inflation. Acute respiratory distress syndrome and pulmonary embolism are not characterized by absent breath sounds. An endotracheal tube that is inserted too far can cause absent breath sounds, but the lack of breath sounds most likely would be on the left side because of the degree of curvature of the right and left main stem bronchi.

A nurse is assessing a male client with chronic airflow limitations and notes that the client has a "barrel chest." The nurse interprets that this client has which of the following forms of chronic airflow limitations? a. Emphysema b. Bronchial asthma c. Chronic obstructive bronchitis d. Bronchial asthma and bronchitis

Answer A. The client with emphysema has hyperinflation of the alveoli and flattening of the diaphragm. These lead to increased anteroposterior diameter, referred to as "barrel chest." The client also has dyspnea with prolonged expiration and has hyperresonant lungs to percussion.

A nurse is caring for a male client immediately after removal of the endotracheal tube. The nurse reports which of the following signs immediately if experienced by the client? a. Stridor b. Occasional pink-tinged sputum c. A few basilar lung crackles on the right d. Respiratory rate of 24 breaths/min

Answer A. The nurse reports stridor to the physician immediately. This is a high-pitched, coarse sound that is heard with the stethoscope over the trachea. Stridor indicates airway edema and places the client at risk for airway obstruction

4. The nurse caring for a male client with a chest tube turns the client to the side, and the chest tube accidentally disconnects. The initial nursing action is to: a. Call the physician. b. Place the tube in a bottle of sterile water. c. Immediately replace the chest tube system. d. Place the sterile dressing over the disconnection site.

Answer B. If the chest drainage system is disconnected, the end of the tube is placed in a bottle of sterile water held below the level of the chest. The system is replaced if it breaks or cracks or if the collection chamber is full. Placing a sterile dressing over the disconnection site will not prevent complications resulting from the disconnection. The physician may need to be notified, but this is not the initial action.

A nurse is caring for a male client hospitalized with acute exacerbation of chronic obstructive pulmonary disease. Which of the following would the nurse expect to note on assessment of this client? a. Hypocapnia b. A hyperinflated chest noted on the chest x-ray c. Increase oxygen saturation with exercise d. A widened diaphragm noted on the chest x-ray

Answer B. Clinical manifestations of chronic obstructive pulmonary disease (COPD) include hypoxemia, - hypercapnia, - dyspnea on exertion and at rest - oxygen desaturation with exercise - and the use of accessory muscles of respiration. Chest x-rays reveal a hyperinflated chest and a flattened diaphragm if the disease is advanced.

A nurse is caring for a male client with emphysema who is receiving oxygen. The nurse assesses the oxygen flow rate to ensure that it does not exceed: a. 1 L/min b. 2 L/min c. 6 L/min d. 10 L/min

Answer B. Oxygen is used cautiously and should not exceed 2 L/min. Because of the long-standing hypercapnia that occurs in emphysema, the respiratory drive is triggered by low oxygen levels rather than increased carbon dioxide levels, as is the case in a normal respiratory system.

A nurse is suctioning fluids from a male client via a tracheostomy tube. When suctioning, the nurse must limit the suctioning time to a maximum of: a. 1 minute b. 5 seconds c. 10 seconds d. 30 seconds

Answer C. Hypoxemia can be caused by prolonged suctioning, which stimulates the pacemaker cells in the heart. A vasovagal response may occur, causing bradycardia. The nurse must preoxygenate the client before suctioning and limit the suctioning pass to 10 seconds.

An emergency room nurse is assessing a female client who has sustained a blunt injury to the chest wall. Which of these signs would indicate the presence of a pneumothorax in this client? a. A low respiratory b. Diminished breathe sounds c. The presence of a barrel chest d. A sucking sound at the site of injury

Answer B. This client has sustained a blunt or a closed chest injury. Basic symptoms of a closed pneumothorax are shortness of breath and chest pain. A larger pneumothorax may cause tachypnea, cyanosis, diminished breath sounds, and subcutaneous emphysema. Hyperresonance also may occur on the affected side. A sucking sound at the site of injury would be noted with an open chest injury.

Nurse Hannah is preparing to obtain a sputum specimen from a client. Which of the following nursing actions will facilitate obtaining the specimen? a. Limiting fluids b. Having the clients take three deep breaths c. Asking the client to split into the collection container d. Asking the client to obtain the specimen after eating

Answer B. To obtain a sputum specimen, the client should rinse the mouth to reduce contamination, breathe deeply, and then cough into a sputum specimen container. The client should be encouraged to cough and not spit so as to obtain sputum. Sputum can be thinned by fluids or by a respiratory treatment such as inhalation of nebulized saline or water. The optimal time to obtain a specimen is on arising in the morning

A nurse is caring for a female client after a bronchoscope and biopsy. Which of the following signs, if noted in the client, should be reported immediately to the physicians? a. Dry cough b. Hematuria c. Bronchospasm d. Blood-streaked sputum

Answer C. If a biopsy was performed during a bronchoscopy, blood-streaked sputum is expected for several hours. Frank blood indicates hemorrhage. A dry cough may be expected. The client should be assessed for signs of complications, which would include cyanosis, dyspnea, stridor, bronchospasm, hemoptysis, hypotension, tachycardia, and dysrhythmias. Hematuria is unrelated to this procedure.

A nurse is suctioning fluids from a female client through an endotracheal tube. During the suctioning procedure, the nurse notes on the monitor that the heart rate is decreasing. Which of the following is the appropriate nursing intervention? a. Continue to suction. b. Notify the physician immediately. c. Stop the procedure and reoxygenate the client. d. Ensure that the suction is limited to 15 seconds.

Answer C. During suctioning, the nurse should monitor the client closely for side effects, including hypoxemia, cardiac irregularities such as a decrease in heart rate resulting from vagal stimulation, mucosal trauma, hypotension, and paroxysmal coughing. If side effects develop, especially cardiac irregularities, the procedure is stopped and the client is reoxygenated.

A female client has just returned to a nursing unit following bronchoscopy. A nurse would implement which of the following nursing interventions for this client? a. Administering atropine intravenously b. Administering small doses of midazolam (Versed) c. Encouraging additional fluids for the next 24 hours d. Ensuring the return of the gag reflex before offering food or fluids

Answer D. After bronchoscopy, the nurse keeps the client on NPO status until the gag reflex returns because the preoperative sedation and local anesthesia impair swallowing and the protective laryngeal reflexes for a number of hours. Additional fluids are unnecessary because no contrast dye is used that would need flushing from the system. Atropine and midazolam would be administered before the procedure, not after.

A community health nurse is conducting an educational session with community members regarding tuberculosis. The nurse tells the group that one of the first symptoms associated with tuberculosis is: a. Dyspnea b. Chest pain c. A bloody, productive cough d. A cough with the expectoration of mucoid sputum

Answer D. One of the first pulmonary symptoms is a slight cough with the expectoration of mucoid sputum. Options A, B, and C are late symptoms and signify cavitation and extensive lung involvement.

A nurse instructs a female client to use the pursed-lip method of breathing and the client asks the nurse about the purpose of this type of breathing. The nurse responds, knowing that the primary purpose of pursed-lip breathing is to: a. Promote oxygen intake. b. Strengthen the diaphragm. c. Strengthen the intercostal muscles. d. Promote carbon dioxide elimination.

Answer D. Pursed-lip breathing facilitates maximal expiration for clients with obstructive lung disease. This type of breathing allows better expiration by increasing airway pressure that keeps air passages open during exhalation. Options A, B, and C are not the purposes of this type of breathing.

A nurse is assessing the respiratory status of a male client who has suffered a fractured rib. The nurse would expect to note which of the following? a. Slow deep respirations b. Rapid deep respirations c. Paradoxical respirations d. Pain, especially with inspiration

Answer D. Rib fractures are a common injury, especially in the older client, and result from a blunt injury or a fall. Typical signs and Sx include - pain and tenderness localized at the fracture site and exacerbated by inspiration and palpation - shallow respirations - splinting or guarding the chest protectively to minimize chest movement, and possible bruising at the fracture site. Paradoxical respirations are seen with flail chest.

A nurse is assessing a female client with multiple trauma who is at risk for developing acute respiratory distress syndrome. The nurse assesses for which earliest sign of acute respiratory distress syndrome? a. Bilateral wheezing b. Inspiratory crackles c. Intercostal retractions d. Increased respiratory rate

Answer D. The earliest detectable sign of acute respiratory distress syndrome is an increased respiratory rate, which can begin from 1 to 96 hours after the initial insult to the body. T his is followed by increasing dyspnea, air hunger, retraction of accessory muscles, and cyanosis. Breath sounds may be clear or consist of fine inspiratory crackles or diffuse coarse crackles.

Presence of overdistended and non-functional alveoli is a condition called: a. Bronchitis b. Emphysema c. Empyema d. Atelectasis

Answer: B. An overdistended and non-functional alveoli is a condition called emphysema. Atelectasis is the collapse of a part or the whole lung. Empyema is the presence of pus in the lung.

The pulse oximetry for a patient with right lower lobe pneumonia indicates an oxygen saturation of 90%. The patient has rhonchi, a weak cough effort, and complains of fatigue. Which action is best for the nurse to take?

Assist the patient with staged coughing. rationale: The patient's assessment indicates that assisted coughing is needed to help remove secretions, which will improve oxygenation. A 2-hour rest period at this time may allow the oxygen saturation to drop further. Humidification will not be helpful unless the secretions can be mobilized. Positioning on the right side may cause a further decrease in oxygen saturation because perfusion will be directed more toward the more poorly ventilated lung.

The position of a conscious client during suctioning is: a. Fowler's b. Supine position c. Side-lying d. Prone

a. Fowler's Position a conscious person who has a functional gag reflex in the semi fowler's position with the head turned to one side for oral suctioning or with the neck hyper extended for nasal suctioning. If the client is unconscious place the patient a lateral position facing you.

A client with a strained trapezius muscle complains of having occasional shortness of breath. What might be the reason for this symptom? a. The strained muscle is an accessory muscle of respiration. b. The diaphragm muscle is also injured. c. There is an undiagnosed heart problem. d. There is a blood clot in his lung.

a. The strained muscle is an accessory muscle of respiration.

The nurse is assessing the client's lung bases posteriorly. At which area can the nurse assess this portion of the lung? a. Right anterior axillary line b. Scapular line c. Midsternal line d. Left midclavicular line

b. Scapular line

In planning a patient education session, the nurse sees one area of focus for Healthy People 2010 is chronic obstructive pulmonary disease (COPD). Which of the following information should the nurse include in the education session to address this focus area? a. Screening for environmental triggers b. Smoking cessation c. Develop action plans d. Identify those at risk

b. Smoking cessation

A patient with respiratory failure has a respiratory rate of 8 and an SpO2 of 89%. The patient is increasingly lethargic. The nurse will anticipate assisting with _______________

endotracheal intubation and positive pressure ventilation. rationale: The patient's lethargy, low respiratory rate, and SpO2 indicate the need for mechanical ventilation with ventilator-controlled respiratory rate. Administration of high flow oxygen will not be helpful because the patient's respiratory rate is so low. Insertion of a mini-tracheostomy will facilitate removal of secretions, but it will not improve the patient's respiratory rate or oxygenation. BiPAP requires that the patient initiate an adequate respiratory rate to allow adequate gas exchange.

With respiratory alkalosis, is the pt hyperventilating or hypoventilating?

hyperventilating (blowing off too much CO2)

While caring for a patient who has been admitted with a pulmonary embolism, the nurse notes a change in the patient's oxygen saturation (SpO2) from 94% to 88%. The nurse will ______________

increase the oxygen flow rate. rationale: Increasing oxygen flow rate usually will improve oxygen saturation in patients with ventilation-perfusion mismatch, as occurs with pulmonary embolism. Because the problem is with perfusion, actions that improve ventilation, such as deep-breathing and coughing, sitting upright, and suctioning, are not likely to improve oxygenation.

Which of the following findings would most likely indicate the presence of a respiratory infection in a client with asthma? 1. Cough productive of yellow sputum. 2. Bilateral expiratory wheezing. 3. Chest tightness. 4. Respiratory rate of 30 breaths/ minute.

1. A cough productive of yellow sputum is the most likely indicator of a respiratory infection. The other signs and symptoms- wheezing, chest tightness, and increased respiratory rate- are all findings associated with an asthma attack and do not necessarily mean an infection is present.

A client is prescribed metaproterenol (Alupent) via a metered-dose inhaler, two puffs every 4 hours. The nurse instructs the client to report adverse effects. Which of the following are potential adverse effects of metaproterenol? 1. Irregular heartbeat. 2. Constipation. 3. Pedal edema. 4. Decreased pulse rate.

1. Irregular heartbeats should be reported promptly to the care provider. Metaproterenol (Alupent) may cause irregular heartbeat, tachycardia, or anginal pain because of its adrenergic effect on beta-adrenergic receptors in the heart. It is not recommended for use in clients with known cardiac disorders. Metaproterenol does not cause constipation, pedal edema, or bradycardia.

Which of the following is a priority goal for the client with chronic obstructive pulmonary disease (COPD)? 1. Maintaining functional ability. 2. Minimizing chest pain. 3. Increasing carbon dioxide levels in the blood. 4. Treating infectious agents.

1. A priority goal for the client with COPD is to manage the signs and symptoms of the disease process so as to maintain the client's functional ability. Chest pain is not a typical symptom of COPD. The carbon dioxide concentration in the blood is increased to an abnormal level in clients with COPD; it would not be a goal to increase the level further. Preventing infection would be a goal of care for the client with COPD.

A client with deep vein thrombosis suddenly develops dyspnea, tachypnea, and chest discomfort. What should the nurse do first? 1. Elevate the head of the bed 30 to 45 degrees. 2. Encourage the client to cough and deep breathe. 3. Auscultate the lungs to detect abnormal breath sounds. 4. Contact the physician.

1. Elevating the head of the bed facilitates breathing because the lungs are able to expand as the diaphragm descends. Coughing and deep breathing do not alleviate the symptoms of a pulmonary embolus, nor does lung auscultation. The physician must be kept informed of changes in a client's status, but the priority in this case is alleviating the symptoms.

Bed rest is prescribed for a client with pneumonia during the acute phase of the illness. The nurse should determine the effectiveness of bed rest by assessing the client's: 1. Decreased cellular demand for oxygen. 2. Reduced episodes of coughing. 3. Diminished pain when breathing deeply. 4. Ability to expectorate secretions more easily.

1. Exudate in the alveoli interferes with ventilation and the diffusion of gases in clients with pneumonia. During the acute phase of the illness, it is essential to reduce the body's need for oxygen at the cellular level; bed rest is the most effective method for doing so. Bed rest does not decrease coughing or promote clearance of secretions, and it does not reduce pain when taking deep breaths.

Which of the following physical assessment findings are normal for a client with advanced chronic obstructive pulmonary disease (COPD)? 1. Increased anteroposterior chest diameter. 2. Underdeveloped neck muscles. 3. Collapsed neck veins. 4. Increased chest excursions with respiration.

1. Increased anteroposterior chest diameter is characteristic of advanced COPD. Air is trapped in the overextended alveoli, and the ribs are fixed in an inspiratory position. The result is the typical barrel-chested appearance. Overly developed, not underdeveloped, neck muscles are associated with COPD because of their increased use in the work of breathing. Distended, not collapsed, neck veins are associated with COPD as a symptom of the heart failure that the client may experience secondary to the increased workload on the heart to pump blood into the pulmonary vasculature. Diminished, not increased, chest excursion is associated with COPD.

Which of the following health promotion activities should the nurse include in the discharge teaching plan for a client with asthma? 1. Incorporate physical exercise as tolerated into the daily routine. 2. Monitor peak flow numbers after meals and at bedtime. 3. Eliminate stressors in the work and home environment. 4. Use sedatives to ensure uninterrupted sleep at night.

1. Physical exercise is beneficial and should be incorporated as tolerated into the client's schedule. Peak flow numbers should be monitored daily, usually in the morning (before taking medication). Peak flow does not need to be monitored after each meal. Stressors in the client's life should be modified but cannot be totally eliminated. Although adequate sleep is important, it is not recommended that sedatives be routinely taken to induce sleep.

A 79-year-old female client is admitted to the hospital with a diagnosis of bacterial pneumonia. While obtaining the client's health history, the nurse learns that the client has osteoarthritis, follows a vegetarian diet, and is very concerned with cleanliness. Which of the following would most likely be a predisposing factor for the diagnosis of pneumonia? 1. Age. 2. Osteoarthritis. 3. Vegetarian diet. 4. Daily bathing.

1. The client's age is a predisposing factor for pneumonia; pneumonia is more common in elderly or debilitated clients. Other predisposing factors include smoking, upper respiratory tract infections, malnutrition, immunosuppression, and the presence of a chronic illness. Osteoarthritis, a nutritionally sound vegetarian diet, and frequent bathing are not predisposing factors for pneumonia.

The nurse is planning to teach a client with chronic obstructive pulmonary disease how to cough effectively. Which of the following instructions should be included? 1. Take a deep abdominal breath, bend forward, and cough three or four times on exhalation. 2. Lie flat on the back, splint the thorax, take two deep breaths, and cough. 3. Take several rapid, shallow breaths and then cough forcefully. 4. Assume a side-lying position, extend the arm over the head, and alternate deep breathing with coughing.

1. The goal of effective coughing is to conserve energy, facilitate removal of secretions, and minimize airway collapse. The client should assume a sitting position with feet on the floor if possible. The client should bend forward slightly and, using pursed-lip breathing, exhale. After resuming an upright position, the client should use abdominal breathing to slowly and deeply inhale. After repeating this process three or four times, the client should take a deep abdominal breath, bend forward, and cough three or four times upon exhalation (" huff" cough). Lying flat does not enhance lung expansion; sitting upright promotes full expansion of the thorax. Shallow breathing does not facilitate removal of secretions, and forceful coughing promotes collapse of airways. A side-lying position does not allow for adequate chest expansion to promote deep breathing.

The nurse should teach the client with asthma that which of the following is one of the most common precipitating factors of an acute asthma attack? 1. Occupational exposure to toxins. 2. Viral respiratory infections. 3. Exposure to cigarette smoke. 4. Exercising in cold temperatures.

2. The most common precipitator of asthma attacks is viral respiratory infection. Clients with asthma should avoid people who have the flu or a cold and should get yearly flu vaccinations. Environmental exposure to toxins or heavy particulate matter can trigger asthma attacks; however, far fewer asthmatics are exposed to such toxins than are exposed to viruses. Cigarette smoke can also trigger asthma attacks, but to a lesser extent than viral respiratory infections. Some asthmatic attacks are triggered by exercising in cold weather.

A client with acute asthma is prescribed short-term corticosteroid therapy. Which is the expected outcome for the use of steroids in clients with asthma? 1. Promote bronchodilation. 2. Act as an expectorant. 3. Have an anti-inflammatory effect. 4. Prevent development of respiratory infections.

3. Corticosteroids have an anti-inflammatory effect and act to decrease edema in the bronchial airways and decrease mucus secretion. Corticosteroids do not have a bronchodilator effect, act as expectorants, or prevent respiratory infections.

A nurse notes that a client has kyphosis and generalized muscle atrophy. Which of the following problems is a priority when the nurse develops a nursing plan of care? 1. Infection. 2. Confusion. 3. Ineffective coughing and deep breathing. 4. Difficulty chewing solid foods.

3. In kyphosis, the thoracic spine bends forward with convexity of the curve in a posterior direction, making effective coughing and deep breathing difficult. Although the client may develop other problems because respiratory status deteriorates when pulmonary secretions are not adequately cleared from airways, ineffective coughing and deep breathing should receive priority attention.

A nurse is teaching a male client with chronic respiratory failure how to use a metered-dose inhaler correctly. The nurse instructs the client to: a. Inhale quickly b. Inhale through the nose c. Hold the breath after inhalation d. Take two inhalations during one breath

Answer C. Instructions for using a metered-dose inhaler include - shaking the canister, - holding it right side up, - inhaling slowly and evenly through the mouth, - delivering one spray per breath, - and holding the breath after inhalation.


Ensembles d'études connexes

Marketing Research Multiple Choice

View Set

Chapter 10 (section 10.12-10.16): microbiology mastering online HW

View Set

History of Japanese Art Final Terms

View Set